Law Decks Flash Cards - Constitutional Law - 2007-2008

239
LAW DECKS COMPREHENSIVE FLASHCARDS TO HELP YOU PREPARE FOR THE MULTISTATE BAR EXAMINATION.

Transcript of Law Decks Flash Cards - Constitutional Law - 2007-2008

Page 1: Law Decks Flash Cards - Constitutional Law - 2007-2008

LAW DECKSCOMPREHENSIVE FLASHCARDS TO HELP YOU PREPARE FOR THE MULTISTATE BAR EXAMINATION.

Page 2: Law Decks Flash Cards - Constitutional Law - 2007-2008

LAW DECKS

TOPICS INCLUDE:

Constitutional LawContracts

CRIMINAL LAWProperty Law

TortsEvidence

Page 3: Law Decks Flash Cards - Constitutional Law - 2007-2008

Thank you for purchasing Law Decks. We feel that the use of Law Decks as a supplement to astudy program for the Multistate Bar Exam will enable you to have foundation for excellent examperformance.

All rights reserved. No part of these cards may be reproduced in any way, or by any means,without permission from Law Decks, Inc. The purchaser agrees not to resell, copy, rent, ortransfer any part of this product.

This product has been prepared from sources the author believes to be accurate and reliable.However, the possibility of human and/or mechanical error does exist. The users of these cardsare encouraged to secure additional information, as needed, to assist them in reaching excellentperformance.

If you have any questions, comments, or recommendations about Law Decks, please contact us.

Law Decks, Inc."The Decks Corporation"

4065 Quakerbridge RoadPrinceton Junction, NJ 08550

(609) 919-9400 ♦ (877)-763-3257 ♦ www.LawDecks.com

GOOD LUCK ON YOUR EXAM!

Page 4: Law Decks Flash Cards - Constitutional Law - 2007-2008

NOTICE

The authors and the publisher of this volume have taken care that the information andrecommendations contained herein are accurate and compatible with the standards generallyaccepted at the time of publication. However, as in any text, some inaccuracies and ambiguitiesmay occur; therefore, if in doubt, please consult your references. The authors and the publisherdisclaim any liability, loss, or damage incurred as a consequence, directly or indirectly, of the useand application of any of the contents of this volume.

Page 5: Law Decks Flash Cards - Constitutional Law - 2007-2008

AUTHORS

Vincent P. Loccisano, JDBased on years of analyzing and forecasting the topic coverage, fact patterns and questionformats covered on the MBE, as well as the Decks Corp. approach to flashcards, Loccisano hasdeveloped this series of flashcards you have before you. Using this systematical and scientificapproach to studying, he has passed numerous professional licensing exams, including theMassachusetts Bar Exam and the Federal Patent Bar Exam, all on the first try. Mr. Loccisanocurrently practices patent law in Boston, Massachusetts. Loccisano holds a J.D. from theIntellecutal Property Law Concentration program of Suffolk University School of Law and a B.S.in Mechanical Engineering from Lehigh University.

Candace Lombardi, MFALombardi is responsible for the creative attempts at humor, satire and diversity in your questions.She holds an M.F.A. in Creative Writing from Emerson College and a B.A. in English fromLehigh University. When she isn't thinking up creative ways to grasp your attention, Lombardiis a freelance travel writer, member of the Davis Square Writers Group, screenwriter, andchildren's book author.

Page 6: Law Decks Flash Cards - Constitutional Law - 2007-2008

Constitutional LawA New York law provides state residents with a decreased state college tuition; whilenonresidents, or those students who have been residents for less than 5 years, shall paya higher nonresident tuition rate. A fourth-year resident files a class action suitchallenging the constitutionality of the law in federal district court and seeks adeclaratory judgment. The state moves to dismiss on this basis: by the time the casewould come to trial, 13 months after filed, the resident would by then be eligible for thereduced tuition rate. Should the state's motion be granted?

A. No, because there is a live controversy.

B. Yes, because the student lacks standing.

C. No, because a declaratory judgment is the improper vehicle.

D. Yes, because the student is now an eligible resident.

Page 7: Law Decks Flash Cards - Constitutional Law - 2007-2008

Answer: AIn the present case, there is a live controversy and the case is not moot. In light of this,a federal court would proceed to hear the case. As a federal court requires a livecontroversy at all stages, not just when the case is filed, it appears at first glance thatthe student's eligibility for resident tuition would render the case moot. However, the factthat this is a class action suit yields viable claims for those remaining within the class.

D is incorrect. The filing student's case may be moot, but there exist those within theclass who are still eligible.

B is incorrect as standing is simply determined at the beginning of a suit, and requiresa tangible stake in the outcome of the suit. The filing student did have such a stake;thereby, providing sufficient standing at the time of filing.

C is incorrect. Although a ruling for declaratory judgment does require a showing ofripeness, in the present case the state law is currently being enforced. Therefore, theissue is clearly ripe and a declaratory judgment is a proper vehicle.

©2007-2008 Law Decks

Page 8: Law Decks Flash Cards - Constitutional Law - 2007-2008

Constitutional LawA congressionally enacted law imposes a 50% tax on all alcohol sales, dictating that theproceeds of this tax be applied toward an anti-drinking media campaign and theestablishment of a federal alcohol awareness clinic. A local brewer files suit in theapplicable federal court, seeking to have this tax struck down as unconstitutional. Thecourt is likely to rule:

A. The tax is constitutional as the broad General Welfare Clause would condone it.

B. The tax is constitutional as it is severable from its purpose.

C. The tax is unconstitutional as it does not provide equal time for the variousmanufacturers to present their arguments.

D. The tax is unconstitutional. It infringes upon the First Amendment rights of the variousmanufacturers by forcing them to pay for a media campaign that they do not agreewith.

Page 9: Law Decks Flash Cards - Constitutional Law - 2007-2008

Answer: AThe tax is constitutional as it falls within the power of Congress under the GeneralWelfare Clause. A tax will be upheld if Congress has the power to regulate the activityand it bears a reasonable relationship to the revenue production. Here, alcohol purchaseis an area regulated by Congress due to its effects on interstate commerce, and the taxat hand does bear a reasonable relationship to the revenue production. Furthermore, theestablishment of anti-drinking clinics, for which Congress is using the proceeds of thetax, is viewed as a valid public purpose. A public purpose such as this falls within thescope of the General Welfare Clause.

B is incorrect as the tax at issue is for use in raising revenue to fund an anti-drinkingcampaign. The tax, therefore, is constitutional as outlined above and is not severable.

C is incorrect as there exists no requirement allowing a party equal time to respond.Under the Constitution, a "fairness in response time" is not provided.

D is incorrect because the brewers' freedom of speech rights are not infringed upon.The brewers are still free to promote a pro-drinking media campaign of their own shouldthey elect to. The tax does not force the manufacturers to support the anti-drinkingmessage and, therefore, does not burden their First Amendment rights.

©2007-2008 Law Decks

Page 10: Law Decks Flash Cards - Constitutional Law - 2007-2008

Constitutional LawThe President convinces Congress to pass a federal usury law setting loan interest rates at 10%per annum, or 3% above the current Federal Reserve loan rate, on the basis that this is anemergency statute passed by the President to stimulate the economy. A state bank is currentlyloaning money at 15% to commercial state clients, while the applicable Federal Reserve rate is8%. In light of the new usury law, this state bank will now be forced to loan money at a maximuminterest rate of 11%, resulting in a substantial financial loss. If the state bank challenges theusury law as unconstitutional for not being within the enumerate powers of the federalgovernment, it will likely:

A. Lose, as the commerce power permits Congress to regulate the commerce of money and,here, there exists a definitive effect on interstate commerce.

B. Win, as the federal government may not regulate interest charged by private lenders unlessthey are involved in interstate lending.

C. Lose, as the General Welfare Clause permits Congress to bolster the federal economy usingany measure that Congress deems necessary.

D. Win, as the interest rates are a local issue not to be addressed by a national usury law.

Page 11: Law Decks Flash Cards - Constitutional Law - 2007-2008

Answer: ACongress is authorized under the Commerce Clause to regulate all activities that havea substantial impact on interstate commerce. In light of this, the actions of the state'sbanks would constitute interstate commerce, thereby, allowing federal regulation.

B is incorrect. The Commerce Clause does, in fact, protect those intrastate actions thathave an effect on interstate commerce.

C is incorrect. The General Welfare Clause serves only to limit Congress's power.

D is incorrect since regulation of interest rates is well within the powers of Congress.(Modified interest rates are changed virtually daily.)

©2007-2008 Law Decks

Page 12: Law Decks Flash Cards - Constitutional Law - 2007-2008

Constitutional LawGeorge is hired by the state department of public works as a new employee. Pursuantto his contract, he is designated a probationary DPW officer who is on probation for sixmonths; during which time he cannot be fired for any reason. After this six monthprobation, he can only be fired upon a showing of cause. After three months George isfired. If true, which of the following would support this state firing practice prior to the endof his probationary period?

A. His performance had been deemed poor.

B. He had not been employed for at least six months.

C. A more competent DPW was able to replace him.

D. He had violated state regulations by speaking to a local TV station regarding theimproper disposal of waste.

Page 13: Law Decks Flash Cards - Constitutional Law - 2007-2008

Answer: BAs we are dealing with an employee at will, the state does not have to providejustification for George's termination. If George was not an employee at will, notice anda hearing would have been necessary.

A is incorrect because poor performance, while a subject that could be raised, is notabsolutely necessary.

C is incorrect as like Answer A, it is not necessary to expressly raise the subject insupport of their decision.

D is incorrect as George's comments to the local reporter are deemed public policycomments (assuming appropriate state action exists); thereby, precluding a FirstAmendment defense to his termination.

©2007-2008 Law Decks

Page 14: Law Decks Flash Cards - Constitutional Law - 2007-2008

Constitutional LawA civilian is subpoenaed to appear in front of the House of Representatives to answerseveral questions regarding a recent investigation. Upon appearing, the civilian refusesto answer any questions directed toward her, is held in contempt of Congress, and isprosecuted in accordance with a federal criminal statute. What is the civilian's bestdefense to the charge of contempt?

A. She is an employee of the Department of Defense and may not answer questionsdirected to her role as a member of the executive branch.

B. She establishes that the questions do not relate to any issues Congress is orderedto legislate upon.

C. She establishes that she holds a presidentially-appointed office and may only bequestioned by the Senate.

D. She establishes that the questions asked of her are not related to funds that theHouse had earmarked for distribution.

Page 15: Law Decks Flash Cards - Constitutional Law - 2007-2008

Answer: BCongress is granted the authority to only investigate those matters in which it isauthorized to ultimately legislate upon. In light of this, Answer B is correct.

A is incorrect. Executive Privilege is not an absolute privilege; Congress is free toinvestigate the actions of an employee of the executive branch.

C is incorrect as Congress is also free to investigate a presidentially-appointed memberof the executive branch should they deem such an investigation necessary.

D is incorrect as Congress may also investigate matters beyond those relating to themoney it has appointed. For example, Congress can investigate all matters in which ithas the authority to legislate, regardless of fiduciary appointments.

©2007-2008 Law Decks

Page 16: Law Decks Flash Cards - Constitutional Law - 2007-2008

Constitutional LawA civilian is subpoenaed to appear in front of the House of Representatives to answerseveral questions regarding a recent investigation. Upon appearing, the civilian refusesto answer any questions directed toward her, is held in contempt of Congress, and isprosecuted in accordance with a federal criminal statute. If the Attorney General refusesto prosecute the civilian, would this refusal be constitutional?

A. No. The Attorney General may not refuse to prosecute anyone who has violated afederal law.

B. No, because the Attorney General may not lawfully disobey a congressional directiveto prosecute those in contempt of Congress.

C. Yes, the executive branch has discretion as to those they wish to prosecute.

D. Yes, If the civilian was an appointed official of the executive branch, she is grantedimmunity for those acts performed in the furtherance of her duties.

Page 17: Law Decks Flash Cards - Constitutional Law - 2007-2008

Answer: CThe executive branch, in light of the Doctrine of Separation of Powers, is granted theright to choose which matters it wishes to prosecute. In light of this, the legislature is notfree to interfere with the executive branch's decisions to choose or dismiss prosecution.

A and B are, therefore, incorrect as a violation of separation of powers.

D is incorrect as executive branch officials are not granted immunity from acts relatingto their duties. Answer D is a clear misstatement of the law.

©2007-2008 Law Decks

Page 18: Law Decks Flash Cards - Constitutional Law - 2007-2008

Constitutional LawSuppose that faced with skyrocketing home prices, Massachusetts creates the House PriceStabilization Corp.(HPSC) which acts as a private corporation, building homes in selectareas and selling them to buyers for the cost of labor and materials. The HPSC is instructedto only build homes in areas where the average new home price is 25% greater than thenational average for a new home across the United States. Based upon this criteria, theHPSC begins building in Roxbury, where new homes are 30% above the national average;but none in Somerville, where new homes are only 20% above the national average. ASomerville resident of Italian heritage sues on the basis of an equal protection violation.What additional facts would assist the resident's suit the most?

A. The legislation that established the HPSC was intended to discriminate against those ofItalian heritage.

B. Citizens of Italian heritage currently have greater difficulty affording homes inMassachusetts.

C. The state had the opportunity to allow HPSC to build statewide, not in select areas.

D. There is a much higher percentage of citizens with Italian heritage in Somerville than inRoxbury.

Page 19: Law Decks Flash Cards - Constitutional Law - 2007-2008

Answer: AAny state actions which can be shown to exist solely to discriminate on the basis of raceor national origin, and are in fact discriminating, will be struck down. The neutralapplication of this legislation is meaningless upon a showing of discrimination. TheSomerville resident will require a showing of intent by the city to disqualify particularareas from funding in the establishment of the building guidelines.

B is incorrect. Difficulty in obtaining a house is meaningless when addressing the currentfacts. Other ethnic groups, for example, may be faced with the same difficulties,rendering this argument meaningless.

C is close to a good answer, but not as good as Answer A. Answer C allows a state toshow a rational basis for electing to subsidize homes in certain areas. Upon the stateshowing a rational basis for its home building selection method, the state discriminationwould be arguably permissible. Answer A is clearly a better answer, as the rational basistest is NOT applied in light of a strict scrutiny application.

D is incorrect as it merely attempts to show that there might be some evidence ofdiscrimination by the state based upon ethnic makeup. As compared to Answer A,Answer D fails to provide concrete evidence of state discrimination solely based onethnicity and/or race.

©2007-2008 Law Decks

Page 20: Law Decks Flash Cards - Constitutional Law - 2007-2008

Constitutional LawA private secular high school has a published admittance policy which states that onlywhite students are eligible for admittance. The school applies for state-offered modellesson plans for various subjects. These model lesson plans are freely distributed to anyschool district or state school that requests them. What is the best argument for theunconstitutionality of the state providing these lesson plans to the private secular highschool in question?

A. No legitimate function is advanced in providing these lesson plans to this school.

B. The state is not permitted to aid private schools, religious or secular.

C. The private school in question furthers segregation within the state.

D. Private discrimination by the school is in violation of the Constitution.

Page 21: Law Decks Flash Cards - Constitutional Law - 2007-2008

Answer: CAn equal protection violation exists as the state would be unnecessarily involving itselfin the segregationist activities of the private secular high school. State action that isdiscriminatory in nature is expressly prohibited under the Constitution. Therefore, thestate may not aid a school which in turn discriminates.

A is incorrect. The distribution of the lesson plans by the state may be found to be"legitimate" in nature; but its interactions with the secular discriminating high schoolwould still be deemed discriminatory acts by the state and, therefore, in violation of theConstitution.

B is incorrect. States are entitled to aid private schools. Yet, the interactions of the statein the present fact pattern, by distributing lesson plans to a high school known fordiscrimination; results in a violation of the Constitution. The state would be indirectlydiscriminating against a class of people since its lesson plans would be aiding aninstitution that discriminates.

D is incorrect. Although state involvement, in accordance with Answer B, is clearlyprohibited: discrimination by a private school is permitted.

©2007-2008 Law Decks

Page 22: Law Decks Flash Cards - Constitutional Law - 2007-2008

Constitutional LawGeorge is hired by the state department of public works as a new employee. Pursuantto his contract, he is designated a probationary DPW officer who is on probation for sixmonths, during which time he cannot be fired for any reason. After this six monthprobation, he can only be fired upon a showing of cause. After three months George isfired. If true, which of the following supports George's argument that he should havereceived notice before dismissal?

A. Expecting to be dutifully employed, George has just purchased a new home withinthe city.

B. There is no evidence that there are more competent replacements for George.

C. George moved from a neighboring state on the reliance of a supervisor's word thathis job would be permanent.

D. George was the only member of his hiring class not to pass the six- monthprobationary period.

Page 23: Law Decks Flash Cards - Constitutional Law - 2007-2008

Answer: CIn light of the oral promise given to George, his termination results in the loss of anexpected property right. The taking of this property right, therefore, requires a showingof a due process and a hearing, as well as an opportunity to respond to the allegationsagainst him. Therefore, C is the best answer.

A is incorrect as it fails to relate any constitutional issue to the present fact pattern.

B is incorrect. The availability of more a competent replacement is a moot point. Suchinformation fails to alter George's status as an employee at will.

D is incorrect as it fails to relate any constitutional issues and protections to the presentfact pattern.

©2007-2008 Law Decks

Page 24: Law Decks Flash Cards - Constitutional Law - 2007-2008

Constitutional LawThe President convinces Congress to pass a federal usury law setting loan interest rates at10% per annum, or 3% above the current Federal Reserve loan rate, on the basis that this isan emergency statute passed by the President to stimulate the economy. A state bank iscurrently loaning money at 15% to commercial state clients, while the applicable FederalReserve rate is 8%. In light of the new usury law, this state bank will now be forced to loanmoney at a maximum interest rate of 11 %, resulting in a substantial financial loss. If the stateseeks to enjoin the federal usury statute on the basis that the statute prevents a state bankfrom making an adequate profit at the fixed interest rate; a federal court should:

A. Rule that the federal court does not have jurisdiction, as the state is a party and thejurisdiction lies in the US Supreme Court.

B. Rule that the state usury law is preempted by the federal statute under the SupremacyClause.

C. Rule that the issue is a local one that should be addressed on a state level, and the federalcourt should enjoin enforcement.

D. Rule that the issue be forced to the state's court of appeals due to the nature of the case.

Page 25: Law Decks Flash Cards - Constitutional Law - 2007-2008

Answer: B

Assuming that Congress has not preempted interest rate regulation, a state is notallowed to establish an interest rate regulation that is not as strict (or stricter) than thefederal rule on the books.

A is incorrect as the federal court clearly has jurisdiction.

C is incorrect as Congress has preempted local control of rate regulation.

D is incorrect as forwarding the case to the state's court of appeals is clearlyinappropriate in the given situation.

©2007-2008 Law Decks

Page 26: Law Decks Flash Cards - Constitutional Law - 2007-2008

Constitutional LawAssume federal legislation exists making local police departments eligible for federalfunding as long as they agree to dedicate a portion of this funding to combatingcorporate crimes against shareholders. What is the best constitutional rationale for suchlegislation?

A. General welfare taxation and spending

B. Police power

C. The power to enforce penal status

D. The War and Defense Power

Page 27: Law Decks Flash Cards - Constitutional Law - 2007-2008

Answer: AThe federal government, when approving monies to be spent by state governments,may attach any restrictions or obligations on the monies accepted by the state.

B is incorrect.There is no general federal police power governing the federal government— i.e., that is not one of the enumerated powers.

C is incorrect. The power to enforce penal statutes is not a federally enumerated powerwithin the control of Congress.

D is incorrect as the War and Defense Power is inapplicable to the present facts. TheWar and Defense Power gives the power to essentially change the status of the nationfrom peace to war.

©2007-2008 Law Decks

Page 28: Law Decks Flash Cards - Constitutional Law - 2007-2008

Constitutional LawJake and Brett have lived together for ten years, but have never married as Jake is still marriedto his first wife (although he and his wife have been separated for some time). Over this time,Jake and Brett have accumulated property as a couple. There is a state statute addressingproperty collected by a couple, but the statute is only applicable when dealing with marriedcouples. This statute is not valid, however, when one of the couple is married to another person,as is Jake. Over the past ten years Brett has been unemployed but ran the home she sharedwith Jake: overseeing the finances, meals, housekeeper, gardener, etc...On their 11thanniversary, Jake tells Brett that he no longer wants to live with her and asks her to move out.Brett files suit, challenging the constitutionality of the statute and seeking half of all propertyaccumulated during their relationship. The state joins as a party in the suit, attempting to provethe constitutionality of the statute. Will Brett prevail?

A. Yes, if the state fails to show a rational relationship to a legitimate state purpose.

B. Yes, if Brett can prove that the statute has no compelling state interest.

C. No, if Brett fails to prove that the statute is not rationally related to a legitimate state interest.

D. No, if the state shows that the statute was necessary and proper.

Page 29: Law Decks Flash Cards - Constitutional Law - 2007-2008

Answer: CThe statute neither burdens a fundamental right nor creates a suspect or quasi-suspectclassification. Because of this, a rational basis test is appropriate.

A is incorrect as it places the burden on the wrong party. Here, the burden is on thechallenger to show that a statute is unconstitutional under a rational basis test.

B is wrong because it applies the wrong standard of review.

D is incorrect; simply because it applies to a clause that is only applicable to federalstatutes. The Necessary and Proper Clause has nothing to do with state statutes.

©2007-2008 Law Decks

Page 30: Law Decks Flash Cards - Constitutional Law - 2007-2008

Constitutional LawJake and Brett have lived together for ten years, but have never married as Jake is still marriedto his first wife (although he and his wife have been separated for some time). Over this time,Jake and Brett have accumulated property as a couple. There is a state statute addressingproperty collected by a couple, but the statute is only applicable when dealing with marriedcouples. This statute is not valid, however, when one of the couple is married to another person,as is Jake. Over the past ten years Brett has been unemployed but ran the home she sharedwith Jake: overseeing the finances, meals, housekeeper, gardener, etc... On their 11thanniversary, Jake tells Brett that he no longer wants to live with her and asks her to move out.Brett files suit, challenging the constitutionality of the statute and seeking half of all propertyaccumulated during their relationship. The state joins as a party in the suit, stating that thestatute was in fact constitutional. The state's strongest defense of the statute would be that:

A. The Constitution does not affect the state's authority over marital relationships.

B. A rational basis exists for the state's promotion of family life and the preservation of maritalproperty.

C. There is no showing of individual discrimination in the statute, as all suits of this type are notprevented.

D. The state police power is not affected by the Constitution.

Page 31: Law Decks Flash Cards - Constitutional Law - 2007-2008

Answer: BThe right to enter into agreements is not a fundamental right and, subsequently, triggersa higher standard of review. Therefore, the rational basis test is the correct standard thatthe state must show.

A and D are wrong as there exist many instances where the Constitution may affect thestate's authority over quasi-marital relationships such as this.

C is illogical. The fact that others are free to bring suits of this type does not prove thatthere is no discrimination. In this particular situation, however, the discrimination isclearly not individually directed toward Brett.

©2007-2008 Law Decks

Page 32: Law Decks Flash Cards - Constitutional Law - 2007-2008

Constitutional LawSuppose Congress, concerned about the quality of elementary school education, passes a lawrevoking elementary school teachers' tenure: this allows schools to fire teachers they deem poor atwill. As a second consideration, the law allows for a salary reduction of these no longer tenuredteachers until a new contract can be negotiated between the school system and the teacher unionrepresentatives. The proposed date of initiation of the law is two years from its passing date inCongress, allowing schools and teachers to prepare for its initiation. Suppose the Carlysle SchoolDistrict plans on abolishing all tenure positions once the law is enacted, which will clearly affectnumerous tenured teachers working within the district. The teachers' union, of which all Carlysleteachers belong, decides to file suit on behalf of its members seeking an injunction to prevent theschool board from abolishing tenured positions, and for a declaratory judgement stating that the lawis invalid. Should the federal court hear the case?

A. No, because the ruling on the law at this point is premature, the enactment of the law being twoyears away.

B. No. The union lacks standing to sue on behalf of the Carlysle teachers.

C. Yes. Because the federal law encourages improper interference with a contract, it is in violationof the Contract Clause of the Constitution.

D. Yes, because the teachers' rights and benefits are threatened by the law and the school board'sstated plans.

Page 33: Law Decks Flash Cards - Constitutional Law - 2007-2008

Answer: AThe federal court should not hear the case because it is not yet ripe for review. Here,there is not a "case and controversy," meaning that there is no plaintiff who has beenharmed or in an immediate threat of harm. There is no immediate threat of harm to theunion in this case since the law does not take effect for another two years. In the comingtwo years, Congress might change the law or repeal it, or the school board may decideto keep the old contract system. In light of this explanation, Answer D is clearly incorrect.

B is incorrect because the union would have standing. In order for an association tohave standing there must be: 1) an injury in fact to its members that would give themstanding. 2) an injury related to the organization's purpose, and 3) neither the nature ofthe claim nor the relief requested requires individual member participation in the lawsuit.Here, all three of those conditions are met.

C is incorrect because the Contract Clause only limits state power, not federal power.Here, there is an action in response to a federal law. Therefore, a Contract Clauseapplication is incorrect. As an aside, even if this was a state law at issue and a ContractClause application was permissible, it is still not clear that there would be a violation.The Contract Clause only bans substantial interference with existing contracts. Here, itis not clear that the interference meets the substantial bar necessary to yield a violation.

©2007-2008 Law Decks

Page 34: Law Decks Flash Cards - Constitutional Law - 2007-2008

Constitutional LawA scout troop of 100 members seeks to hold an overnight camping trip at a city park.They apply for a permit and learn that the park is only open during daylight hours. Thecity states that the park must be closed to facilitate cleaning and trash removal duringthe night so that they can reopen promptly the following morning. The scout leader filessuit, seeking to have the park allow this overnight camping exercise. If the courtdetermines that the city ordinance is valid, what will be the basis for its decision?

A. The ordinance is rationally related to a legitimate government interest and does notburden the First Amendment rights more than necessary.

B. The ordinance is narrowly tailored toward a significant governmental interest anddoes not unreasonably limit alternative channels.

C. The ordinance is substantially related to a legitimate government interest and doesnot burden the First Amendment rights more than is reasonably necessary.

D. The ordinance is rationally related to a legitimate government interest and does notunreasonably limit alternative channels of communication.

Page 35: Law Decks Flash Cards - Constitutional Law - 2007-2008

Answer: BThe city ordinance is both narrowly tailored and serves a significant governmentalinterest, therefore, is permissible. Here, we are dealing with a restriction of free speechthat is content neutral in time, place and manner; narrowly tailored to serve a significantgovernment interest; and leaves the alternative channels of communication open.

Answers A and D are incorrect for using the incorrect test. A rational relationship test isused for restrictions on free speech rights in nonpublic forums. Here, the park is a publicforum requiring a strict level of scrutiny.

C is incorrect because it misstates both parts of the standard.

©2007-2008 Law Decks

Page 36: Law Decks Flash Cards - Constitutional Law - 2007-2008

Constitutional LawFollowing the advice of a congressionally created committee, Congress passes a billproviding that: "It is unlawful to discriminate against minority race members in contractnegotiations whether private or public. Any person whose rights under this statute areviolated may bring a cause of action in the federal district court for up to $10,000 in thedistrict in which the violation occurred." Assuming that Congress had the power to enactthe state, the basis for this power most likely came from the:

A. Statute Contract Clause

B. Thirteenth Amendment

C. Fourteenth Amendment

D. Commerce Clause

Page 37: Law Decks Flash Cards - Constitutional Law - 2007-2008

Answer: BThe Thirteenth Amendment gives Congress the power to adopt legislation that avoidsslavery of involuntary servitude from states or via private parties. This includes anyconduct that Congress deems to be a "badge" or "incident" of slavery.

A is incorrect. The Contract Clause prevents states from modifying contractsretroactively and has no regulation of regular private contracts.

C is also wrong. Though the Fourteenth Amendment prohibits states from racialdiscrimination; it does not extend to private conduct.

D, the Commerce Clause, is a decent answer, but it is not as good as Answer B sincethe commerce power is limited to transactions that have a substantial economic effecton interstate commerce. Here, the regulations are solely intrastate transactions. TheThirteenth Amendment is a better basis for the legislation, as it does not have theinherent limitation on legislation that the Commerce Clause does.

52007-2008 Law Decks

Page 38: Law Decks Flash Cards - Constitutional Law - 2007-2008

Constitutional LawSuppose Virginia passes a statute to provide a need-based financial aid program toresidents that attend public or private colleges in the state. Pat files for a grant to attendan out-of-state university and is denied. He files suit in federal court against a stateofficer in charge of reviewing grants, challenging the constitutionality of the statute as aviolation of equal protection. Which of the following statements is most correct?

A. The suit is barred by the Eleventh Amendment.

B. Pat has standing even though he has never paid taxes in Virginia.

C. The federal court will only grant the relief sought by Pat if there is a showing of"extraordinary circumstances."

D. The Doctrine of Sovereign Immunity prevents Virginia from being sued.

Page 39: Law Decks Flash Cards - Constitutional Law - 2007-2008

Answer: BPat can prove an injury in fact and can additionally prove that a favorable decision by the court willalleviate his injury: therefore, he has standing. The fact that he has never paid taxes is a red herring,as Pat is not bringing suit as a taxpayer (which is typically not allowed), but rather bringing suit as ainjured party himself. This personal injury is a violation of his right to equal protection. As he clearlymay be relieved of this injury by a positive outcome, he is allowed to bring this suit in his name only.

A is incorrect. The Eleventh Amendment prevents a citizen of one state from bringing suit against hisstate or another state in federal court. The Eleventh Amendment, however, does not bar suits againsta state official who by following a state law allegedly violates the plaintiff's constitutional rights. Herethe suit is not brought against the state of Virginia, nor does it seek a recovery from state funds, butrather seeks prospective payment of funds that would follow the granting of his application for aid.Such a grant is not prohibited by the Eleventh Amendment.

C is incorrect because it is only in reference to criminal statutes that a party seeking to enjoin suchstatutes must show irreparable injury or exceptional circumstances. Here, there is no criminal statuteor prosecution at issue. Thus, a showing of "extraordinary circumstances" is irrelevant andunnecessary.

D is also wrong. The Doctrine of Sovereign Immunity states that a governmental entity may not besued unless it consents to be sued. In the present case, there are no facts indicating that the statehas consented to being sued. Even if there were facts indicating consent, the suit is not against thestate, but rather against the official responsible for issuing the grants. In light of this, sovereignimmunity is inapplicable.

©2007-2008 Law Decks

Page 40: Law Decks Flash Cards - Constitutional Law - 2007-2008

Constitutional LawA federal statute was enacted that provides federal funds to open child healthcarecenters in inner-city areas. The strongest constitutional basis for this enactment is the:

A. Commerce Clause

B. Equal Protection Clause

C. Taxing and Spending Clause

D. General federal police powers

Page 41: Law Decks Flash Cards - Constitutional Law - 2007-2008

Answer: C Taxing and spending for the general welfare of citizens is a power awarded to the federalgovernment.

A is almost correct, as the Commerce Clause does give Congress a wide basis of powerwhen passing socially related legislation. Here, however, the passed legislation is onlyrelated to welfare. Therefore, the legislation is more related to the Taxing and SpendingClause than to the Commerce Clause.

B is incorrect as there exists no citizen that is being deprived of a federal right.

D is incorrect as it is a false statement. There are no federal police powers.Note: Often times a trick answer like this is included on the bar exam and is alwaysincorrect.

©2007-2008 Law Decks

Page 42: Law Decks Flash Cards - Constitutional Law - 2007-2008

Constitutional LawCongress sets up a small business advisory board, appointing Karen Jacob to head thenewly established department. After five years of successful leadership by Jacob, theadvisory board is abolished following the appeal of the legislation that initially authorizedit. Jacob, a faithful employee, is offered a new job at a significantly lower salary. Shesues, claiming that Congress cannot remove a federal judge from office, or decrease hersalary, absent a showing of bad behavior. If this goes to court, the trial court should:

A. Dismiss Jacob's case, as Jacob was not an Article III judge and is not entitled to lifetenure.

B. Dismiss Jacob's case and find for the government, as Jacob lacks standing to raisethe question.

C. Rule for Jacob, as working for the advisory board has become an establishedproperty right.

D. Rule for Jacob, under the federal judiciary independence granted to judges underArticle III.

Page 43: Law Decks Flash Cards - Constitutional Law - 2007-2008

Answer: AHere, the advisory board is simply an administrative agency with limited jurisdiction.Therefore, Karen Jacob is not viewed as the judges within the meaning of Article III; andit is only Article III judges that have life tenure. In light of this, Answer D is clearly wrong.

B is incorrect because Jacob does have standing. She can establish a specific injuryviolation of a constitutional right awarded to her and can bring the claim for her ownbenefit.

C is incorrect. The Constitution does not permit a property right to be established inemployment with a federal agency. A wrongful discharge is only applicable if KarenJacob can show that she lost something considered to be a property right.

©2007-2008 Law Decks

Page 44: Law Decks Flash Cards - Constitutional Law - 2007-2008

Constitutional LawSuppose Congress passes legislation allowing for oil and natural gas drilling on federallands by private companies. To oversee these private companies, and to engage in oiland gas contracts with them, an act establishes the Federal Oil and Gas Commission.The Federal Oil and Gas Commission has also been granted power to appointadministrative judges who are allowed to rule in cases involving violations of the oil andgas contracts of private companies against the federal agency. Finally, the Federal Oiland Gas Commission is also empowered to research the means by which lowenvironmental impact drilling can advance, and pass these findings on to Congress inan effort to pass new low environmental impact laws. The authority for the establishmentof the Federal Oil and Gas Commission is most likely:

A. The Commerce Clause

B. The Taxing and Spending Power

C. Congress's authority to regulate federal lands

D. The War and Defense Power

Page 45: Law Decks Flash Cards - Constitutional Law - 2007-2008

Answer: CArticle IV of the Constitution gives Congress the power to regulate federal lands and itis only Congress who has sufficient authority to regulate these federal lands. In light ofthis, Answers B and D are clearly wrong.

A is also wrong. While Congress is given broad Commerce Clause powers, thosepowers do not relate to the matter at hand.

©2007-2008 Law Decks

Page 46: Law Decks Flash Cards - Constitutional Law - 2007-2008

Constitutional LawCongress passes legislation allowing for oil and natural gas drilling on federal lands by privatecompanies. To oversee these private companies, and to engage in oil and gas contracts with them, anact establishes the Federal Oil and Gas Commission. The Federal Oil and Gas Commission has alsobeen granted power to appoint administrative judges who are allowed to rule in cases involvingviolations of the oil and gas contracts of private companies against the federal agency. Finally, theFederal Oil and Gas Commission is also empowered to research the means by which lowenvironmental impact drilling can advance, and pass these findings on to Congress in an effort to passnew low environmental impact laws. After violating a Federal Oil and Gas Commission rule, a privatedriller is fined $25,000 for a criminal act defined as "polluting the environment around the well head."The private driller files suit in federal court, seeking to enjoin the payment of the fine. His best argumentis:

A. Regulations concerning criminal conduct can not be made by agency rules; only by federalstatute.

B. The establishment of the commission was illegal. In light of this, any rules passed by thecommission are inherently invalid.

C. Since the fine was $25,000, the private driller is entitled to a trial by jury -as opposed to a trial bya commission-appointed administrative judge —as edited by the Federal Oil and Gas Commissionrules.

D. A trial in front of an administrative law judge violates the private driller's equal protection rightsunder the Fourteenth Amendment.

Page 47: Law Decks Flash Cards - Constitutional Law - 2007-2008

Answer: BThe Appointment Clause only allows Congress to appoint inferior officers to the courts,or as heads of departments. Congress cannot appoint its own members to thecommission, as enforcement is an executive act.

A is wrong as the commission is free to pass rules and regulations over the matters inwhich it was appointed to address. However, those rules and regulations are not criminalstatutes.

C is wrong because Congress is free to establish new actions that may be adjudicatedby agencies without juries in matters such as these.

D is clearly wrong because there is no discrimination at issue.

©2007-2008 Law Decks

Page 48: Law Decks Flash Cards - Constitutional Law - 2007-2008

Constitutional LawSuppose that a joint effort between the President and the Canadian Prime Minister toreduce the over-fishing of Atlantic salmon, a special commission is appointed to addressthe countries' concerns. The commission studies the problems and passes an annualfishing ban in the Atlantic from March 1st through June 1st. Assume that New Hampshirealso passes a fishing permit statute which, for a cost of $10,000, licenses commercialfisherman to fish year long, including otherwise prohibited months. If the jointcommission files an action in district court to enjoin the state of New Hampshire's permitprocess, the New Hampshire permit statute will most likely be found:

A. Constitutional, because the conservation of wildlife is an issue best left to the stateswhere the wildlife is located.

B. Constitutional, because a state statute takes precedent over any executiveagreement.

C. Unconstitutional, as state acts can not render an executive agreement null and void.

D. Unconstitutional, as the state statute discriminates against New Hampshirefisherman.

Page 49: Law Decks Flash Cards - Constitutional Law - 2007-2008

Answer: CSupreme Court precedent states that executive agreements are the law of the land upuntil Congress enacts a statute that is inconsistent with the executive agreement. Here,the New Hampshire state statute MUST be declared unconstitutional as it conflicts withthe presidential executive agreement. In light of this, Answers A and B are clearlyincorrect as the question at hand is whether the President has the power to enter intothe executive agreement; not if the issue is best left to the states to decide.

D is also incorrect. While the conclusion is correct, the logic behind the conclusion islacking as compared to the correct Answer C. Discrimination against New Hampshirefisherman is not a valid reason to consider this the best answer in light of the moreapplicable Answer C.

©2007-2008 Law Decks

Page 50: Law Decks Flash Cards - Constitutional Law - 2007-2008

Constitutional LawIn which of the following cases is there no original jurisdiction within the United StatesSupreme Court?

A. A case involving a violation of US law by an ambassador to China.

B. A case involving a state dispute between two neighboring states.

C. A case involving maritime jurisdiction in the region off the coast of Florida.

D. A case involving the government of a state and the federal government.

Page 51: Law Decks Flash Cards - Constitutional Law - 2007-2008

Answer: C

The Supreme Court is granted original jurisdiction in all cases affecting ambassadors,public ministers, consuls, and those in which a state is a party. Therefore, Answers A, Band D are cases where the Supreme Court does have original jurisdiction. There is nooriginal jurisdiction in cases involving maritime jurisdiction in the region off the coast ofthe United States.

©2007-2008 Law Decks

Page 52: Law Decks Flash Cards - Constitutional Law - 2007-2008

Constitutional LawSuppose the New York Hairy Eagle Act is passed to protect the endangered hairy eaglepopulation. The act makes it illegal to kill, maim or possess any part of a protected hairyeagle. A local retailer who sells gifts and tourist trinkets, some of which contain hairyeagle feathers, challenges the act on the basis that it is unconstitutional. Is the act valid?

A. No, the statute violates due process as the prohibition of sale is an effective takingwithout due compensation under the Fifth Amendment Due Process Clause.

B. No, the statute is discriminatory as applied.

C. Yes, there exists a rational relationship to an interstate commerce concern.

D. Yes, the statute is designed to protect an endangered national resource.

Page 53: Law Decks Flash Cards - Constitutional Law - 2007-2008

Answer: CRegulating the possession or sale of an eagle item is certain to have an effect oncommerce, thereby, allowing Congress to act under the Commerce Clause. Here, thereis no taking of a property right without just compensation, since a person need notsurrender the trinket. Therefore, Congress's power to regulate is proper, regardless ofthe fact that it affects the profits of a hairy eagle trinket seller. In light of this, Answer Ais clearly incorrect.

B is not supported by the facts presented in the question. It is an erroneous answer.

D has the correct conclusion, but the reasoning behind it is clearly false in light of theexplanation of the correct answer.

©2007-2008 Law Decks

Page 54: Law Decks Flash Cards - Constitutional Law - 2007-2008

Constitutional LawSuppose that in a joint effort between the President and the Canadian Prime Minister toreduce the over-fishing of Atlantic salmon, a special commission is appointed to addressthe countries' concerns. The commission is instructed to evaluate the problem andpropose legislation to the President and Canadian PM. Upon receiving these draftregulations and express authorization by Congress, the President enters into anexecutive agreement with the Canadian PM granting the joint commission enforcementand adjudicative powers over the promulgated regulations. This executive agreement bythe President is:

A. Valid, the President has unlimited powers in entering into executive agreements.

B. Valid, the President has plenary powers in the area of foreign affairs.

C. Invalid, unless the Senate ratified the executive agreement by a two-thirds vote.

D. Invalid, because conservation of wildlife is not an area left solely to presidentialdiscretion.

Page 55: Law Decks Flash Cards - Constitutional Law - 2007-2008

Answer: BThe President has "plenary" powers as per the Supreme Court. In light of this, thePresident has almost unlimited power when dealing with foreign affairs. The Presidentis also free to enter into executive agreements in accordance with Supreme Courtguidelines. This power to enter into executive agreements, however, is significantly morelimited than the power of dealing with foreign affairs.

A is wrong because there are some limits on the President's powers to enter intoexecutive agreements.

C is wrong because executive agreements do not require a two-thirds vote of theSenate.

D is a false statement. While Congress does have the authority to act in order topreserve wildlife; so, too, is the President free to enter into agreements concerning theprotection of wildlife.

©2007-2008 Law Decks

Page 56: Law Decks Flash Cards - Constitutional Law - 2007-2008

Constitutional LawSuppose that New Jersey decides to lease the county-owned land surrounding HorseshoeLake to the Horseshoe Co., a private company, for development of a beach club and marinain exchange for 5% of its yearly profits. Horseshoe Co. proposes a set of bylaws that restrictaccess to members only. Such membership would be handled by Horseshoe Co.'s boardof directors. Having reviewed these bylaws, New Jersey approves. Weeks prior to openingthe club, the board of directors decides that membership will require a monthly membershipfee and is limited to those of the Jewish faith. A dedicated atheist who was deniedmembership sues, claiming that the membership policy violates his freedom of religion.Who will prevail?

A. Plaintiff, because denial of membership to atheists has been held to hinder the freeexercise of religion.

B. Plaintiff, because the membership policy is in violation of the Establishment Clause.

C. Horseshoe Co., because freedom of religion is not protected against acts of privateindividuals, groups or a privately owned corporation.

D. Horseshoe Co., because the plaintiff, an atheist, has no standing to challenge themembership standards on religious grounds.

Page 57: Law Decks Flash Cards - Constitutional Law - 2007-2008

Answer: BThe policy of the club is not for a secular purpose and, in light of this, arguably advancesreligion. Therefore, it is an Establishment Clause violation.

While close, Answer A is not as strong an answer as Answer B, because the SupremeCourt has never defined atheism as a religion that is protected by the Free ExerciseClause.

C is wrong because state action can be found due to the county's dealings withHorseshoe Co. In light of this state action, an individual cause of action can be foundagainst the state, and the private corporation is not granted immunity in its actions.

D is wrong because a person asserting a violation of the Establishment Clause does nothave to allege infringement of a particular religious freedom in order to have standing; itis enough that he is directly affected by the government action challenged.

©2007-2008 Law Decks

Page 58: Law Decks Flash Cards - Constitutional Law - 2007-2008

Constitutional LawSuppose the Federal Firearms Act is enacted by Congress to provide a uniform set oflaws for the registration of handguns across all states in the nation. Which of thefollowing is a valid basis for the Federal Firearms Act?

A. The Equal Protection Clause of the Fourteenth Amendment

B. The Second Amendment

C. The Commerce Clause

D. The Necessary and Proper Clause

Page 59: Law Decks Flash Cards - Constitutional Law - 2007-2008

Answer: CThis statute relates to activities that affect interstate commerce and, therefore, is withinthe commerce power.

A is wrong. The Equal Protection Clause of the Fourteenth Amendment can not be thebasis for federal powers by itself as it applies only to the states. Instead, Congressinvokes its Section 5, Fourteenth Amendment powers when dealing with equalprotection of the law.

B is wrong because the Second Amendment only serves to prohibit some types ofenactment. The Second Amendment does not serve as the basis of enactment.

D is wrong because the Necessary and Proper Clause does not create the power to actin Congress, but rather broadens congressional power authorized under some otherprovision.

©2007-2008 Law Decks

Page 60: Law Decks Flash Cards - Constitutional Law - 2007-2008

Constitutional LawA federal law requiring all cars driven on naval bases to have side airbags and antilockbrakes would most probably be justified by:

A. The Property Clause of Article IV, Section 3

B. The General Welfare Clause of Article I, Section 8

C. The Supremacy Clause of Article VI, Section 2

D. The Commerce Clause of Article I, Section 8

Page 61: Law Decks Flash Cards - Constitutional Law - 2007-2008

Answer: AUnder Article IV, Section 3, Congress is given the power to regulate territories andproperty belonging to the US. Under this power, requiring side airbags and antilockbrakes would be permissible.

B is incorrect. The General Welfare Clause only relates to taxing and spending byCongress, not areas such as this where we are dealing with regulation of an activity.

C is incorrect. The Supremacy Clause grants federal law the supreme law of the land ascompared to state law. Here, there is no info regarding state law at all.

D is clearly not the best answer as compared to Answer A. Congress does have powerover interstate commerce, and arguably we are dealing with an interstate commerceissue due to the involvement of automobiles. However, Answer A presents a betteranswer based on the facts available and does not require drawing an elaborateconclusion.

©2007-2008 Law Decks

Page 62: Law Decks Flash Cards - Constitutional Law - 2007-2008

Constitutional LawIn which of the following scenarios would there be a violation of the Equal ProtectionClause of the Fourteenth Amendment?

A. Bribery of a federal official by Mr. X so that the official does not distribute condomsto Asian teenagers because Mr. X hates Asians.

B. Ms. Y bribes the dean of a locally licensed private school to admit only women,because Ms. Y feels women are the superior race.

C. Mr. Z threatens the coach of a public high school rhythmic gymnastics team toexclude all Hispanics from the team because of their race.

D. Mr. A persuades his church group to deny shelter to gay men at the church'shomeless shelter because Mr. A is a homophobe.

Page 63: Law Decks Flash Cards - Constitutional Law - 2007-2008

Answer: COnly in Answer C has Mr. Z threatened a state official into denying the equal protection rightsof a particular class; this results in a Fourteenth Amendment violation as it is a denial of life,liberty or property without due process. The Equal Protection Clause is only applicable to stateactions. Therefore, the federal statute at issue must be related to a state official or agency thatdenies the rights of an individual. In Answer C the public school coach, a state official, has beencoerced to deny the rights of individuals based on race, a suspect classification, without theshowing of a compelling state reason. In light of this, Answer C is the correct answer.

A is incorrect. Although Mr. X is inducing a federal official to discriminate, there is no FourteenthAmendment violation as the Fourteenth Amendment does not address actions of federalemployees. The Fifteenth Amendment, by comparison, would be a more applicable answer asit does curtail the discrimination of individuals by the federal government.

B is incorrect. Here, we are dealing with a private discrimination which is outside the reach ofthe Fourteenth Amendment, unless there is a showing of applicable state action by the statebeing significantly involved with the private entity. The school is simply licensed. There existsno state action and, therefore, no violation of the federal law.

D is incorrect for a similar reason to Answer B. We're dealing with a church that is a privateentity and there is no mention of any conduct that would result in state action. The FourteenthAmendment Equal Protection Clause does not apply to the acts of a private person and thereis no equal protection violation.

©2007-2008 Law Decks

Page 64: Law Decks Flash Cards - Constitutional Law - 2007-2008

Constitutional LawSuppose Congress proposes a bill creating a price schedule for all trucks sold in the USbased upon their wheelbase. What is the strongest constitutional basis for the proposedlegislation?

A. All SUV sales in the US have a significant impact upon interstate commerce.

B. The commerce power allows the federal government to allow state regulation of thesales of SUVs as these are clearly commerce-related activities.

C. Congress is free to regulate transportation, of which SUV sales are clearly related.

D. Congress may use its general welfare powers to legislate in a manner that benefitspeople of the US.

Page 65: Law Decks Flash Cards - Constitutional Law - 2007-2008

Answer: AHere, we have a clear effect on interstate commerce by the sales of these vehicles. Asall SUV sales clearly have a huge impact on interstate commerce, Congress isconstitutionally empowered to regulate such transactions.

B is incorrect because the price of an SUV in one state may only involve that singlestate. If there is no relationship between this sale and another state, an interstatecommerce argument is inapplicable. Sales like these, that do not involve interstatecommerce per se, are not open to regulation by Congress under their commerce power.

C is too broad. Regulation of transportation by Congress can only occur throughregulations that exist because there is a substantial impact on interstate commerce.Simple transportation-related regulations (like the one in this answer) without aninterstate commerce tie-in, are wrong.

D is incorrect. Congressional power under the General Welfare Clause is related to actsof taxing and spending, not regulatory acts such as this that involve neither taxing, norspending.

©2007-2008 Law Decks

Page 66: Law Decks Flash Cards - Constitutional Law - 2007-2008

Constitutional LawA state has established an interstate package and mail delivery system for moving state-required items as a replacement to the United States Postal Service and has enacted astate law to protect this service. The state further initiates legislation stating that all stateemployees may use the state package delivery system within the state, regardless ofwhether their use is for state business or personal business. Is this constitutional?

A. No, it violates the monopoly granted to the United States Postal Service.

B. No, the Equal Protection Clause precludes the granting of beneficial treatment tostate employees.

C. No, the Commerce Clause prohibits the state from directly competing with otherstate-for-profit businesses.

D. Yes, since the delivery service is entirely within the state borders.

Page 67: Law Decks Flash Cards - Constitutional Law - 2007-2008

Answer: AThe United States Postal Service is granted a monopoly under Article 1, Section 8 overthe delivery of mail and packages. No other system for delivering mail may beimplemented without the consent of Congress. Here, the actions of the state would becontrary to the monopoly granted to the United States Postal Service.

B is incorrect as no suspect class or fundamental right is at issue, the application of theEqual Protection Clause is not applicable, and a rational basis test would be utilized.Under the rational basis, any legitimate state interest would be sufficient to uphold thelaw. Therefore, the state delivery service would be upheld, as a conceivable basis couldbe found and the law is rationally related to this interest.

C is incorrect as it misstates the Commerce Clause. Under the Commerce Clause astate may compete with interstate business, but it may not interfere with interstatecommerce by protecting a local interest or by unduly burdening interstate transactions.

D is incorrect as the postal monopoly applies to all locales of the United States, and thestate's interstate mail and package delivery system service would run counter to thisfederal monopoly.

©2007-2008 Law Decks

Page 68: Law Decks Flash Cards - Constitutional Law - 2007-2008

Constitutional LawSuppose Congress has appropriated money to all states on the basis that the money is usedin establishing public opera performances, so long as the tickets to these performances aredistributed to the public on a first come, first served basis. One million dollars in federalfunding was received by a state-run inner city opera foundation, which was established to addcultural diversity and exposure to poor underprivileged urban youth. The state has an existinglaw that all tickets to public opera performances organized by the state-run foundation aredistributed as follows: 10% to charitable organizations, 20% to corporate sponsors, and 70%to the public on a first come, first served basis. The state's method of distributing tickets willbe found:

A. Constitutional, as the state-run organization is protected under the Doctrine of Federalismwhich prohibits the federal government from interfering with state government.

B. Constitutional, as the state system of distribution substantially conforms with theunderlying purpose of the federal ticket distribution requirement.

C. Unconstitutional, as the Supremacy Clause prohibits the state from controlling thedistribution of tickets.

D. Unconstitutional, as the state distribution system violates equal protection.

Page 69: Law Decks Flash Cards - Constitutional Law - 2007-2008

Answer: CThe Supremacy Clause provides that valid acts of Congress supersede any state law.Here the act of Congress is valid, as they are spending for the general welfare, andCongress is additionally free to place any contingencies on the grant as it sees fit. Here,state law clearly conflicts with the proposed federal distribution. Therefore, theSupremacy Clause provides that the federal distribution should be utilized.

A is incorrect. Even if Congress may not have the power to regulate how the tickets aredistributed, they may condition grants under their spending power.

B is incorrect. The Supremacy Clause renders all state laws, regardless of how closelyenforced to an congressionally enacted regulation, invalid.

D is incorrect as the state program is valid under the equal protection guidelines of theConstitution. Programs that favor or discriminate against as suspect class, such asracial or ethnic minorities, are subject to strict scrutiny, requiring a narrow tailoringdesignated to promote a compelling state interest. Here, there is a compellinggovernmental interest (cultural diversity), and the program is narrowly tailored.Therefore, the program would survive an equal protection challenge.

©2007-2008 Law Decks

Page 70: Law Decks Flash Cards - Constitutional Law - 2007-2008

Constitutional LawAssume there is a universal treaty between the US and France in which both sidesagree not to tax citizens residing in each other's country that has been ratified by the USSenate and the French Parliament. Further assume that Will Bates relocates to France,seeking to avoid paying tax on his estate. Following the relocation of Will Bates toFrance, the French government demands that taxes be paid on Will's billions in strictopposition to the enacted treaty. The US President quickly declares the tax treaty void,and orders that each French citizen within the US be located and taxed appropriately. Issuch an act by the President constitutional?

A. Yes, under the Supremacy Clause.

B. No, unless the President receives consent from the US Senate.

C. Yes, under the foreign policy powers of the President.

D. Yes, as the President has emergency powers to protect the citizens of the US.

Page 71: Law Decks Flash Cards - Constitutional Law - 2007-2008

Answer: CThe President of the United States is granted powers to enter into treaties and thesepowers are significantly broader than those vested in the President when dealing withinternal affairs. There exists no significant judicial control over the entering of treatiesand foreign relations, thereby allowing the President's actions.

A is incorrect. The Supremacy Clause simply states that the Constitution, treaties, andlaw are the governing laws of the land, and when faced with a situation in which thereis conflicting state law, federal laws must be applied in place of the conflicting state law.The Supremacy Clause does not grant any powers to the federal government.

B is incorrect. The Constitution states that the President must only receive the adviceand consent of the Senate when entering into treaties. There are no requirementsexpressed for the President when voiding an existing treaty in regards to consent andadvice of the Senate.

D is incorrect as the emergency power of the President when dealing with a taxcollection issue such as this, is not expressly granted. Here, there is simply a suggestionthat the citizens of the United States need protection, but a definite need is lacking. Inlight of this, the President's power over foreign affairs in more applicable.

©2007-2008 Law Decks

Page 72: Law Decks Flash Cards - Constitutional Law - 2007-2008

Constitutional LawSuppose the son of a US Senator is accused of raping a woman. As he's a pseudo-celebrity, the press is anxious to report all of the details of this case. Suppose Kansasallows rape cases to be televised as long as the victim's face is blacked out. Additionally,details of the victim's identity are not to be published. A local reporter, anxious to get ascoop, finds out the name of the victim and publishes it in the paper. The paper foundthis info from a public document that failed to black out the victim's name. In accordancewith the statute, the paper is fined $100,000. If the paper decides to challenge the fine,the court should:

A. Rule for the state, because it has a compelling interest in protecting the privacy of itscitizens.

B. Rule for the paper, because the First Amendment allows the press to print anyinformation that it legally obtains.

C. Rule for the state, because inclusion of the victim's name in the public document wasintentional.

D. Rule for the paper, because the state does not have a compelling interest inprohibiting the dissemination of the lawfully obtained information here.

Page 73: Law Decks Flash Cards - Constitutional Law - 2007-2008

Answer: DThe paper should prevail as freedom of the press prohibits Congress or the states fromrestricting the powers of the press. The information printed by the paper is protected, aswould the freedom of speech by an ordinary person, and the media is free to publishlawfully obtained facts. In light of this, Answer C is obviously wrong.

A is incorrect. While a state might have a compelling interest in protecting the privacy ofits citizens, they can not prohibit publication of public information.

B is incorrect as it is too broad, since freedom of the press is not absolute. There arenumerous restrictions. For example, the press can not reprint materials protected by theLibrary of Congress copyright office, regardless of whether or not they legally obtainedconsent of the author.

©2007-2008 Law Decks

Page 74: Law Decks Flash Cards - Constitutional Law - 2007-2008

Constitutional LawSuppose that a US attorney working for the Department of Health is angered by hisboss's refusal to turn the office he works in into a no smoking zone. Angered, he makesnumerous comments to anyone who will listen to him, causing a massive disturbancewithin the office and his supervisor great distress. The supervisor seeks to fire theattorney on the spot, but wants to know his options. The attorney is not subject toremoval for cause only, and may be fired for any reason. Which of the following is thebest summary of the situation?

A. The attorney has a liberty interest in his First Amendment rights of free speechrequiring a hearing before dismissal.

B. The attorney has a property interest as a public employee which requires notice andan opportunity to respond prior to being fired.

C. The attorney has no right to a hearing because his statements were not an expressionof views on public issues.

D. The attorney has both a liberty interest and a property interest that entitles him to anevidentiary hearing before firing.

Page 75: Law Decks Flash Cards - Constitutional Law - 2007-2008

Answer: AThe attorney has a right to a hearing to determine whether his First Amendment rightswere violated. The Due Process Clause of the Fifth Amendment gives a person a libertyinterest in the exercise of specific constitutional rights, of which free speech is included.Prior to the firing of a government employee for speech-related conduct, in a setting wherethe speech involved was matter of public concern, the courts have a duty to balance theemployee's rights as a citizen against the government's interest as an employer. Mattersof public concern, such as smoking policies, are given a broad interpretation.

B is wrong as there is no property interest in the attorney's job. Only employees who aresubject to removal for "cause" have a property interest and must be given notice of thecharges and a pre-termination opportunity to respond to those charges. Here, the attorneyis not an employee entitled to removal for cause only and has no valid property interest.

C is wrong. The attorney is entitled to a hearing as long as he can claim that his speechwas on a public issue and, therefore, protected by the First Amendment.

D is wrong for two reasons. Answer A explains why the attorney does not have a propertyright. Additionally, the attorney is not awarded a due process right to a pre-terminationevidentiary hearing. For example, a post-termination evidentiary hearing is probablysufficient.

©2007-2008 Law Decks

Page 76: Law Decks Flash Cards - Constitutional Law - 2007-2008

Constitutional LawSuppose Idaho has a state constitution similar to that of the United States, including an Article VIanalog guaranteeing a speedy trial. Luke is arrested for assault in Idaho, and receives a preliminaryhearing within a day of his original arrest. He is denied bail and returned to the county jail. The statehas a statute stating that a defendant in custody must have a trial within 30 days of his arrest. Thirtydays after his arrest and still no trial, Luke files a motion for dismissal for violation of his right to aspeedy trial under the state constitution. The trial judge grants Luke's motion, on the basis that hemust file a federal procedure regarding a speedy trial. The state appeals, but the state supreme courtagrees with the trial judge. The state prosecutor seeks to take the case to the US Supreme Court.Assuming the Supreme Court feels that the Idaho court wrongly decided the matter, it should:

A. Reverse the decision and remand it to state court because the state speedy trial issue was sointertwined with the federal question, making it difficult, if not impossible, to determine on whichground the Idaho court relied.

B. Reverse the decision, the state speedy trial provision cannot be interpreted in a manner differentfrom federal interpretations.

C. Decline jurisdiction, the Eleventh Amendment prohibits a state from challenging a decision of itssupreme court in federal court.

D. Reverse the decision and remand the case to be decided on the independent state grounds only.

Page 77: Law Decks Flash Cards - Constitutional Law - 2007-2008

Answer: DThe Supreme Court has jurisdiction to hear the case, as the state court's decision wasnot based on independent state grounds. (The original state court decision was basedon federal case law interpreting an identical federal provision.) Jurisdiction is proper andthe Supreme Court could reverse the state court decision, holding that the 30-day delayis not a federal Constitution violation. Remanding the case, however, is necessary sothat the state may decide if the delay was too long under state law.

A is incorrect as it is clear that the state court was relying on federal case interpretations.However, even if the Supreme Court could not decide whether the case was based onfederal or state grounds, it would not reverse the case: it would simply dismiss the caseand remand it to the state for clarification.

B is incorrect. The state constitutional provisions do not have to be interpreted exactlythe same as federal provisions; states are given a minimum standard of rights that theymust provide, but states are free to broaden these rights at will.

C is incorrect because a state is free to appeal a ruling of its own court to the SupremeCourt. The Eleventh Amendment only prohibits federal courts from hearing actions by aprivate party or foreign government against state government.

©2007-2008 Law Decks

Page 78: Law Decks Flash Cards - Constitutional Law - 2007-2008

Constitutional LawAssume New Jersey has elected to celebrate its rich diverse heritage by sponsoring adiversity-based photography exhibit, which this year includes photographs of variousreligious symbols. The exhibit is located in a New Jersey-owned building and isadministered by the state. Additionally, New Jersey contributes approximately $2,000per year to the display's annual budget of just over $25,000. The remaining $23,000 ispaid by local donations. A local citizen, angered by the religious display, files suitchallenging the state's funding of the exhibit. The court should find that the state'sactions are:

A. Unconstitutional, as the state does not have a compelling interest in running theexhibit.

B. Unconstitutional, as the state aid in maintaining the photographed religious symbols,is in contradiction with the First Amendment.

C. Constitutional, as the amount spent yearly by the state pales in comparison to theannual operating budget of the exhibit.

D. Constitutional, as the photography exhibit also includes other non-religious displaysdepicting the diversity of the state.

Page 79: Law Decks Flash Cards - Constitutional Law - 2007-2008

Answer: D

The state may continue the display of religious symbols in the exhibit as the diversityphotography exhibit is not primarily religious, does not exhibit preference to one religionover another and is, therefore, not in contradiction with the Establishment Clause.TheEstablishment Clause, when faced with a situation where there is no preference for onereligion over another, is valid if: 1) it has a secular purpose, 2) the primary effect neitherfosters nor burdens religion, and 3) it does not involve excessive government entanglementwith religion. Here, all three criteria are duly met. The secular purpose is to promotediversity, the primary effect neither promotes nor demotes religion, and the mereadministration of the exhibit is not viewed as excessive entanglement by the state.

A is incorrect as it applies to the incorrect standard of review. Establishment Clause casesin which a preference of one sect over another are not exhibited, are resolved under thethree-part test (noted in the correct answer above) and not under a compelling stateinterest.

B is incorrect since a minimal contribution by the state in aiding the exhibit is not excessiveentanglement.

C is incorrect as the comparison of the monies spent by the state does not preclude anEstablishment Clause violation, should one of the aforementioned three requirements ofAnswer D be found to be lacking.

©2007-2008 Law Decks

Page 80: Law Decks Flash Cards - Constitutional Law - 2007-2008

Constitutional LawIf Congress enacts a statute banning any discrimination against African-Americans in allbusiness dealings within any state; the statute would most likely be held:

A. Constitutional, as the federal government has an important interest in furthering theequal protection provisions of the Fourteenth Amendment.

B. Unconstitutional, as the Commerce Clause is of insufficient scope to cover actionssuch as these by Congress.

C. Constitutional, under the Thirteenth Amendment provisions against "denotations ofbadges of slavery."

D. Unconstitutional, as privileges and immunities provided to citizens do not include allbusiness transactions.

Page 81: Law Decks Flash Cards - Constitutional Law - 2007-2008

Answer: CThe Thirteenth Amendment has been held to allow Congress to prevent any form ofprivate discrimination that can be viewed as a badge of slavery. The statute at issuebans any discrimination against African-Americans; therefore, private conduct fallswithin the scope of the statute. Statutes such as these are permissible under theThirteenth Amendment.

A is incorrect as the Fourteenth Amendment has been limited to those actions which arestate actions. Here, the statute includes private actions, which fall within the scope of theThirteenth Amendment.

B is incorrect as the statute remains enforceable under the Thirteenth Amendmentregardless of whether or not the Commerce Clause is of sufficient scope to cover theprescribed actions. Addressing the Commerce Clause for sufficient scope to cover allintestate transactions is unnecessary in light of an answer denoting an application of theThirteenth Amendment.

D is wrong. It is simply not applicable to the facts at hand. The privileges and immunitiesprovided to citizens are often directed to matters such as interstate travel. As recitedearlier in the correct answer, the Thirteenth Amendment is the applicable basis by whichto answer the present set of facts. In light of this, Answer C remains the best.

©2007-2008 Law Decks

Page 82: Law Decks Flash Cards - Constitutional Law - 2007-2008

Constitutional LawA poor teenage woman finds herself pregnant and living in a state that does not providefinancial assistance in obtaining an abortion. The neighboring states, in comparison, doprovide financial assistance for abortions for poor teenage women such as she. Wishingto obtain an abortion, the woman considers moving to a neighboring state, but fails tomeet the one year residency requirement of this neighboring state prior to obtaining anabortion. Desperate, this woman brings an action in federal court naming the governorfrom the neighboring state as the only defendant and challenging the state's residencyrequirement. Upon examination, the governor of the neighboring state proclaims thatshe feels the residency requirement is, in fact, unconstitutional. The court should:

A. Dismiss, as the poor teenage woman lacks standing.

B. Dismiss, as there is no case of controversy.

C. Abstain, as the issue of state-funded abortions is a highly political question sure toincite multiple groups.

D. Issue a declaratory judgment upholding the residency requirement in light of acompelling state interest in preventing out-of-state parties from obtaining state-fundedabortions.

Page 83: Law Decks Flash Cards - Constitutional Law - 2007-2008

Answer: BThe court should dismiss the present case as there is a lack of disparate parties on bothsides. As federal courts will not issue advisory opinions; the present case fails to includetwo disinterested parties representing both sides of the controversy. In order to preventa mere advisory opinion, the federal court should dismiss.

A is incorrect as the woman clearly has standing. In order to show standing a personmust demonstrate a definitive stake in the outcome of the controversy. Here, the plaintiffcan clearly show an injury of fact, namely: the prevention of obtaining an abortion shouldthe government fail to rule in her favor.

C is incorrect as there is no political question at issue. A political question is looselydefined as one that is inherently unable to be resolved by the political process.Residency requirements such as this are not political questions.

D is incorrect. When faced with residency requirements and waiting periods such as this,a state is typically obligated to show a compelling state interest, since the existence ofsuch a requirement impedes an individual's ability to travel within states. Regardless ofthis fact, should one assume that the state has a valid compelling state interest, the courtstill would have to decline jurisdiction in accordance with Answer B as there is no casein controversy.

©2007-2008 Law Decks

Page 84: Law Decks Flash Cards - Constitutional Law - 2007-2008

Constitutional LawThe state of Montana passes a law requiring all trucks passing through to use tire chainsduring the winter months, while neighboring states allow either tire chains or studdedtires during the winter months. The Supreme Court strikes down the statute stating thateither chains or studs are equally safe. The state of Montana then passes a new statutestating that all trucks must use studded tires during the winter months as chains areunsafe and dangerous. This statute will be struck down under which of the followingarguments?

A. The Supremacy Clause

B. The Commerce Clause

C. The basis that studs and chains are equally safe.

D. Res judicata

Page 85: Law Decks Flash Cards - Constitutional Law - 2007-2008

Answer: BThe Commerce Clause allows a state to regulate local interstate commerce so long as theregulation does not unduly burden or discriminate against interstate commerce. Here, thestatute does not discriminate against interstate commerce as it treats all trucks in a similarfashion. Regarding the burden on interstate commerce which must be balanced against thepotential benefit, here the burden is great, as all trucks passing through will be required touse a particular type of tire. As the Supreme Court has already found that the two types oftires are equally affective, there is no applicable benefit. As the burden outweighs thebenefit, the statute will be struck down.

A is incorrect as the Supremacy Clause is used to strike down state laws that conflict withfederal laws or that involve an area of law that Congress has preempted. Here, there areno facts alleging either of these conditions.

C is incorrect. The mere fact that both types of tires are found to be equally effective doesnot allow an outright ban of one type of tire to be instantly found unconstitutional. Forexample, the Due Process Clause may be applied to allege that the law is arbitrary;however, additional facts would be required of the present question.

D is incorrect. Res judicata, namely the fact that the issue has already been litigated, is notdoctrine that is constitutional in nature. A court is not required to follow its previous findingsand decision.

©2007-2008 Law Decks

Page 86: Law Decks Flash Cards - Constitutional Law - 2007-2008

Constitutional LawIn which of the following scenarios will Jonathan, a Mexican citizen who received hisPhD from Texas S & M and is still residing in Texas, likely prevail?

A. Jonathan applies to be a firefighter in Texas but is denied due to an existing cityordinance requiring all firefighters to be US citizens, resulting in a lawsuit on thegrounds of unconstitutionality.

B. Jonathan applies to be a sheriff, but is denied due to an existing city ordinancerequiring all sheriffs to be US citizens, resulting in a lawsuit on the grounds ofunconstitutionality.

C. Jonathan applies to be a professor at Texas S & M, but is denied due to an existingcity ordinance requiring all professors at the university to be US citizens, resulting ina lawsuit on the grounds of unconstitutionality.

D. Jonathan applies to be a public school teacher in Texas, but is denied due to anexisting city ordinance requiring all public school teachers to be US citizens, resultingin a lawsuit on the grounds of unconstitutionality.

Page 87: Law Decks Flash Cards - Constitutional Law - 2007-2008

Answer: CState or local laws relating to alienage are the subject of strict scrutiny evaluation.Discrimination based on alienage is typically not allowed, absent a showing of acompelling state interest. Here, no compelling state interest exists by prohibiting aliensfrom teaching at the university level.

A, B and C are incorrect on the basis that the Court has found the participation of aliensin state government matters an application of the rational basis test. The Supreme Courthas upheld statutes that prohibit aliens from teaching in elementary schools, as arguablya teacher offers significant influence in young students' understanding of state andfederal government and the benefits of citizenship; rendering the application of a rationalbasis test which may result in a finding of constitutionality for the state legislation. In lightof the above, challenging legislation relating to alienage on the professor level, whichhas a strict scrutiny test associated with it, offers the best chance at obtaining anunconstitutionality judgment.

©2007-2008 Law Decks

Page 88: Law Decks Flash Cards - Constitutional Law - 2007-2008

Constitutional LawTexas makes it a felony for anyone in any building in which court is in session to holdbanners or signs that are geared toward influencing a jury. Matthew holds a signreading: "Free Bud, or none of the jury will be safer on the steps of the courthouse. Inthe criminal proceeding:

A. Matthew can be convicted, as the statute does not violate the freedom of expressionguaranteed by the First Amendment.

B. Matthew cannot be convicted, unless he personally intended to harm the jury.

C. Matthew cannot be convicted, because the statute potentially could apply to thosewhose speech is constitutionally protected.

D. Matthew can be convicted if there existed a clear and present danger that the jurywould be influence by the sign.

Page 89: Law Decks Flash Cards - Constitutional Law - 2007-2008

Answer: AMatthew can be convicted as the statute at issue applies to a courthouse, an area specifically controlledby government, as opposed to more traditionally First Amendment protected areas, such as a publicstreet. It has been established that free speech may interfere with governmental process in an area likea courthouse, and therefore, regulation of free speech for that area has been found permissible. Inregulating free speech in these non-public areas, a showing that the regulation of the speech is relatedto the purpose served by the property must be had, in addition to a showing that the regulation is not ineffect solely to suppress a single form of speech or criticism. In the present fact pattern, regulation ofspeech in this non-public forum of the courthouse is directly related to promoting a secure, comfortableenvironment for jurors, lawyers and judges alike. Therefore, such a regulation is permissible.

B is incorrect as Matthew's intent is irrelevant. The law makes any speech that intends to disrupt a juryimpermissible, and does not take into account the intent of the speaker.

C is incorrect as it is based on an argument that the statute is overly broad. While a statute that restrictssubstantially more speech than necessary is in fact unenforceable, here the speech is simply restrictedin areas where judicial actions are undergoing. Restriction such as this is not overly broad and,therefore, permissible.

D is incorrect for misapplying the clear and present danger test. The Clear and Present Danger Doctrinedictates that a state cannot forbid advocating either the use of force or violation of the law unless suchadvocacy is directed at inciting lawless action or likely to produce such lawless action. However, astatute does not punish advocacy of force or breaking the law, but rather seeks to maintain a stablejudicial environment.Therefore, the restrictions are enforceable and the clear and present danger testin unenforceable.

02007-2008 Law Decks

Page 90: Law Decks Flash Cards - Constitutional Law - 2007-2008

Constitutional LawSuppose Michigan has an existing state statute requiring all schools, both public andprivate, to administer a state-prepared test to students in order to determine if they arecurrently meeting the existing state standards. Recently, Michigan has passed a newstatute reimbursing private schools for the cost of moderating and grading these testsfrom private funds. If reviewed by the Supreme Court, the statute should be held:

A. Constitutional, if objective grading standards are used and reimbursement coversonly secular services.

B. Constitutional, if there is a great enough state interest to justify the burden onreligion, and there exists no alternative means.

C. Unconstitutional, because direct payment to private schools is a violation of theEstablishment Clause of the First Amendment.

D. Unconstitutional, as the statewide testing procedure results in unnecessaryentanglement between private and public schools.

Page 91: Law Decks Flash Cards - Constitutional Law - 2007-2008

Answer: AThere exists a secular purpose in providing state aid. Additionally, providing aid has alegitimate interest to the state, and does not present any risk of being used to aid thetransmission of religious views. In light of this, Answer A is correct.

B misstates the legal test when dealing with state interaction with religion. The testevaluating the state's interest vs. the burden on religion, as well as whether analternative means exists, is only applicable to a Free Exercise Clause question. Here,we are faced with an Establishment Clause case.

C is incorrect because not all aid to religious schools is in violation of the EstablishmentClause.

D has no basis in the facts given.

©2007-2008 Law Decks

Page 92: Law Decks Flash Cards - Constitutional Law - 2007-2008

Constitutional LawNew Jersey decides to lease the county-owned land surrounding Horseshoe Lake to theHorseshoe Co., a private company, for development of a beach club and marina inexchange for 5% of its yearly profits. Horseshoe Co. proposes a set of bylaws thatrestrict access to members only. Such membership would be handled by the HorseshoeCo.'s board of directors. Having reviewed these bylaws, New Jersey approves. Weeksprior to opening the club, the board of directors decides membership will only be opento American citizens. A citizen of Russia, now legally living in the United States, appliesand is promptly denied admittance following his application. He sues, claiming themembership policy violates his equal protection rights. Who will win?

A. The plaintiff, because the standards promote no compelling state interest.

B. The plaintiff, unless Horseshoe Co. can show a rational basis for the denial of aRussian man's application.

C. Horseshoe Co., because the plaintiff lacks standing to raise an equal protection claimwhile he is not a US citizen.

D. Horseshoe Co., because the promulgate bylaws make no mention of national originor religion.

Page 93: Law Decks Flash Cards - Constitutional Law - 2007-2008

Answer: AState action can be found in situations where the state has affirmatively encouraged orfacilitated discriminatory acts by private groups. Here, such a situation is presentedthrough the leasing of the land and, therefore, state action is applicable. Discriminationby a state of lawful aliens is generally viewed as a suspect classification test and canonly be permissible if the state shows that it is necessary to promote a compelling stateinterest. Here, there is obviously no compelling state interest.

B is incorrect. National origin discrimination does not require a rational basis testapplication, but rather that a strict scrutiny test be applied.

C is incorrect. Resident aliens are deemed to be "persons" within the meaning of theFourteenth Amendment; thereby, allowing the legal alien to raise an equal protectionclaim.

D is also incorrect. Even though the bylaws are nondiscriminatory on their face, theeffects of the application of the bylaws are discriminatory in nature.

©2007-2008 Law Decks

Page 94: Law Decks Flash Cards - Constitutional Law - 2007-2008

Constitutional LawThe state legislature of New York passes a law making it a crime for "unmarried couplesto engage in 'crimes against nature' or other forms of perverted activities." The bestargument against constitutionality would be:

A. Violation of the Equal Protection Clause

B. Unconstitutionally vague

C. Violation of right to privacy

D. A taking without due process

Page 95: Law Decks Flash Cards - Constitutional Law - 2007-2008

Answer: BThe best argument is that the statute is unconstitutional based on vagueness. Under theDue Process Clause, a law or regulation is unconstitutional for vagueness if it fails togive persons reasonable notice of what actions are prohibited. Here, the scope of "otherforms of perverted sexual activities" is unclear, thereby rendering the clauseunconstitutionally vague.

A is incorrect. It is unclear what standard of review would be applied under the EqualProtection Clause, as the statute does not designate a suspect or quasi-suspectclassification. Additionally, it is not clear that a fundamental right is at issue.

C is incorrect as it is unclear that the explicit fundamental right to privacy is at issue.Assuming a privacy right is found, strict scrutiny review would be applied, resulting in aprobable finding of unconstitutionality of the statute. However, if a privacy right is notfound, a rational basis test would be applied and the statute would be upheld. Therefore,it is unlikely that a privacy right would be involved, but also unlikely that the statute wouldbe upheld.

D is incorrect as the Due Process Clause requires governmental compensation forgovernmental taking (typically of property) and no taking is evident in the recited facts.

©2007-2008 Law Decks

Page 96: Law Decks Flash Cards - Constitutional Law - 2007-2008

Constitutional LawA law states that all disagreements between the federal government and a state overfederal aid shall be settled in a federal district court in the effected state.The lawadditionally provides that "the judgement shall be transmitted to the head of the federalaid dispensing authority, and if said head is satisfied with the judgment, head shallexecute it according to the terms." The law is:

A. Constitutional, disagreements over federal aid involve federal questions and fallwithin the jurisdiction power of the US.

B. Constitutional, spending of federal monies includes the authority to settle disputesoriginating from said spending.

C. Unconstitutional, the statute vests authority in the federal court to determine a matterprohibited by the Eleventh Amendment.

D. Unconstitutional, the statute vests authority in a federal court to render an advisoryopinion.

Page 97: Law Decks Flash Cards - Constitutional Law - 2007-2008

Answer: DThe law gives federal district courts the authority to render advisory opinions. Federalcourts may hear cases and controversies, as interpreted by the Supreme Court, but maynot render advisory opinions. The judgment here is clearly advisory in nature, and is,therefore, beyond the power of the district court.

A is incorrect as there is no case or controversy at issue. Although there is a clearfederal question, the need for a case or controversy is also necessary.

B is incorrect in a similar vain. Although Congress clearly has the authority to attend todisputes such as this, the existence of a case or controversy is again lacking.

C is incorrect. The Eleventh Amendment only prohibits federal courts from a citizen'saction against their state, and does not bar actions between the federal government anda state.

©2007-2008 Law Decks

Page 98: Law Decks Flash Cards - Constitutional Law - 2007-2008

Constitutional LawThe state occupational health and safety board of a town issues regulations, valid underits statutory mandate, requiring all employers in the state to provide air purificationsystems for all employee work areas. These regulations replace previous guidelines foremployee air quality that were generally not mandatory and did not specify the methodof purification used. The required air purification system is least likely to be aconstitutional requirement when applied to which of the following employers?

A. Company X, a wholly owned subsidiary of Alpha electronics, a German corporationwhich provides personalized instructions for private citizens on the use of the interneton a home computer by way of its ten in-state outlets.

B. The state supreme court who recently refurbished a new courthouse complete withnon-ionizing air filtration.

C. The US Armed Forces recruiting center located within the confines of the town.

D. The town citizens' center, a privately operated community center, funded bydonations and constructed with a loan from the Veterans' Administration Agency andrepayable to said agency.

Page 99: Law Decks Flash Cards - Constitutional Law - 2007-2008

Answer: CThe US Armed Forces recruiting center is least likely to comply. The state has no powerto regulate activities of the federal government absent the consent of Congress.

A is incorrect. Although owned by a subsidiary of a foreign company, there is insufficientshowing that a foreign commerce is involved and requires congressional regulation ofsuch foreign commerce.

B is incorrect as there is not a substantial impairment of contract, which would mandatean application of the Contract Clause.

D is incorrect. Although funded by the Veterans' Administration Agency, the communitycenter is privately operated and funded by donations yielding only a slight connectionwith the federal government. Said connection is insufficient to claim immunity.

©2007-2008 Law Decks

Page 100: Law Decks Flash Cards - Constitutional Law - 2007-2008

Constitutional LawSuppose that the state of Massachusetts passes a statute requiring all employers toprovide medical insurance to full time employees. The Employers Anti-InsuranceAssociation, an organization made up of many local employers, brings suit against thestatute. The burden is on:

A. The EAIA, to prove no rational relationship.

B. The EAIA, to prove no compelling interest.

C. The state, to prove a rational relationship.

D. The state, to prove a compelling state interest.

Page 101: Law Decks Flash Cards - Constitutional Law - 2007-2008

Answer: AThe EAIA must prove that there is no rational basis. Regardless of whether the statuteis treated as an equal protection problem (all employers are singled out) or as a dueprocess challenge (law affects all employees) the same rational basis standard willapply. No fundamental right is at issue and the employers are not a suspect or quasi-suspect class so strict scrutiny is not applicable. The burden of proof under this rationalbasis application will fall on the party challenging the governmental action, namely theEAIA.

B is incorrect as the wrong standard is applied.

C and D are incorrect as the burden is placed on the wrong party.

©2007-2008 Law Decks

Page 102: Law Decks Flash Cards - Constitutional Law - 2007-2008

Constitutional LawSuppose that as part of Homeland Security, Congress establishes a federal security taskforce commission that evaluates building security in Washington, D.C. The commissionhas the authority to establish new security guidelines, investigate existing guidelines,and coordinate security procedures among agencies such as the Secret Service and theCIA. Of the ten commission members: five are appointed by the President, three by acommittee from the House of Representatives, and two by the Supreme Court. ThePresident is given veto power over any of the selections made by the House or theSupreme Court. If this legislation is challenged and found unconstitutional, it will mostlikely be because:

A. The President's veto power over the House is an improper exercise of the veto power.

B. The commission will be exercising administrative powers.

C. The Court does not have the power to make advisory commission appointments.

D. The legislation does not provide for Senate confirmation of Presidential appointees.

Page 103: Law Decks Flash Cards - Constitutional Law - 2007-2008

Answer: BArticle II, Section 2: Congress may not appoint members of a body with administrativeor enforcement powers, as these appoints are viewed as "officers of the United States"requiring Presidential appointment and Senatorial confirmation (absent a vesting ofcongressional appointment power in the President alone, in federal courts, or in headsof departments). Election of two commission members by the House of Representativesis a violation of the Appointments Clause since the commission has investigative andadministrative powers.

A is incorrect. The legislation presents a problem in that the congressional appointmentsinfringe on the executive branch's power to appoint, as opposed to the allegedinfringement by the President on the legislature's power to veto congressionalappointments.

C is incorrect since the Court has been properly granted the power by Congress toappoint members of the commission.

D is incorrect because Senate confirmation is only required for cabinet-level officers.Inferior officers (i.e. —members of this commission), may be appointed by the Presidentwithout Senate approval if provided for by Congress. Here, Congress has in fact giventhat power to the President.

©2007-2008 Law Decks

Page 104: Law Decks Flash Cards - Constitutional Law - 2007-2008

Constitutional LawWilmington decides to rezone several areas of the city following complaints fromresidents regarding adult theaters and strip clubs, which they claim have artificiallylowered the value of their property and have burdened their neighborhood withincreased late night traffic. Which of the following options are available for the city?

A. Wilmington has no power to limit adult theaters in any manner that differs fromzoning limitations of other commercial businesses.

B. Wilmington may prohibit adult theaters and strip clubs through a new zoningordinance because erotica such as this is unprotected speech.

C. Wilmington needs to show a compelling interest to rezone adult theaters and stripclubs.

D. Wilmington is free to rezone to eliminate these adult theaters and strip clubs on thebasis of controlling secondary effects of these establishments on the city residents.

Page 105: Law Decks Flash Cards - Constitutional Law - 2007-2008

Answer: DZoning restriction on adult establishments are permissible as long as the zoningordinance is designed to promote important governmental interests and does notprohibit all such entertainment in the community. Here, the important governmentalinterest may be defined as preventing the secondary effects of these establishments onresidents located nearby.

A is incorrect as Supreme Court precedent has, in fact, allowed differential treatment ofadult theaters in certain cities. This regulation has been found to be permissibleregardless of the fact that it appears on its face to be a content-based regulation of freespeech.

B is incorrect because nude dancing has been held to be a symbolic conduct and ismarginally protected speech. Therefore, governmental regulation is permissible if itserves important governmental interests unrelated to the suppression of this marginallyregulated speech.

C is incorrect. Erotica is only marginally protected speech and, therefore, may beregulated upon a showing of important interests. Though the result of Answer B iscorrect; the reasoning is wrong. There is no need to show a compelling interest, butrather a lower standard of an important interest. The permissible important interest ofprotecting property values is sufficient enough to allow this regulation.

©2007-2008 Law Decks

Page 106: Law Decks Flash Cards - Constitutional Law - 2007-2008

Constitutional LawMarcia and Barnard are a married couple and have a son two years into their marriage.Marcia works as a horticulturalist while Barnard stays home to take care of their child.A year after the birth of their child, Marcia and Barnard's relationship is in shambles andBarnard seeks a divorce. The state requires him to pay a $300 divorce application fee.Without any income, Barnard feels this is unreasonable and sues. Which of the followingstatements is correct?

A. To justify the fee, the state will have to show that its divorce fee is necessary toachieve a compelling interest in light of Barnard's poverty.

B. Barnard will have to show that requiring him to pay the divorce fee is irrational andunrelated to any state program.

C. Barnard has a compelling interest in obtaining a divorce, and the state, therefore,must show that its divorce fee is narrowly tailored to serve an important state interest.

D. The right to a divorce is fundamental. Therefore, the state must show that requiringBarnard to pay the fee is necessary to achieve a compelling state interest.

Page 107: Law Decks Flash Cards - Constitutional Law - 2007-2008

Answer: DMarriage and divorce are part of the fundamental right of privacy. Therefore, the statemust show a compelling state interest in requiring payment of the fee. Absent a showingby the state of a compelling interest, a strict scrutiny test will allow Barnard to challengethe law and win.

A gives the correct answer, but for the incorrect reason. Here the strict scrutinyapplication is not based on the fact that Barnard is poor, but rather is based on the factthat a fundamental right is at issue.

B is incorrect for applying the wrong standard of review. While this may appear to be aclassification based on wealth (i.e. —the wealth to afford a divorce) which would typicallyresult in a rational basis test, here we are first dealing with a fundamental right violationby the states; thereby, requiring a strict scrutiny application.

C is incorrect for two reasons: both the incorrect standard as well as the incorrect result.It is paramount to recognize that divorce is within the fundamental rights awarded to aperson requiring a strict scrutiny evaluation, as opposed to the intermediate standardtest set forth in this answer.

©2007-2008 Law Decks

Page 108: Law Decks Flash Cards - Constitutional Law - 2007-2008

Constitutional LawA law is passed by Congress forbidding the use of the US Postal Service for distributionof birth control advertisements. Litigation between those for and against the billeventually goes before the federal court regarding the constitutionality of the statute. Thebest argument against the constitutionality is:

A. The statute offends certain rights to privacy.

B. The statute constitutes a taking without due process.

C. The statute improperly infringes on the commercial speech protection of the FirstAmendment.

D. The statute unduly burdens interstate commerce.

Page 109: Law Decks Flash Cards - Constitutional Law - 2007-2008

Answer: CThe bill infringes upon the commercial speech protections of the First Amendment. Whilecommercial speech is protected by the First Amendment, it may be significantly moreregulated than non-commercial speech upon evaluation if the speech is misleading, orfraudulent, and if the speech concerns a legal activity. Here, the activity is in fact legal andthere is no indication of fraud. The second phase of review is if the governmental interestin regulation is substantial, the regulation advances that interest directly and the regulationis narrowly applied. Here, there is no indication that there is a valid governmental interestpromoted, nor is there an indication of direct advancement of governmental interest.

A is not the best answer as it is not as direct as Answer C. Here we are dealing withregulation of free speech. The best answer clearly relates to First Amendment restrictions.

B is incorrect as there is no unjust taking of personal property, or an unjust reduction invalue of personal property at issue. There is no indication that the ban is unjust and noindication that contraceptive makers will not be capable of making a profit in lieu of theproposed ban.

D is wrong. Congress is free to control interstate commerce as it pleases, with theexception of acts that violate other constitutional provisions. It is the states that must notunduly burden interstate commerce.

©2007-2008 Law Decks

Page 110: Law Decks Flash Cards - Constitutional Law - 2007-2008

Constitutional LawOhio passes a statute requiring all prospective parents under 30 years of age to obtaina "responsibility certificate" before having children. Those without this certificate willloose their children to the state-run adoption process. A pregnant 25 year old womanwithout this certificate challenges the law. If the action proceeds to a substantivedetermination which of the following is most accurate?

A. The state bears the burden on proving that the law is necessary to achievea compelling governmental purpose, as it has a substantial impact on fundamentalrights.

B. The state has the burden on proving that the law is necessary as it creates a suspectclassification.

C. The prospective mother has the burden on proving that the law is not necessary inorder to achieve a compelling governmental purpose, because it is presumed valid.

D. The mother bears the burden, as the law is an exercise of the state's police power.

Page 111: Law Decks Flash Cards - Constitutional Law - 2007-2008

Answer: AIf the case goes to trial the state has the burden of proof that the statute is related to acompelling governmental purpose, because it has a substantial impact on a fundamentalright, namely the right to have children which is viewed as a right to privacy. If the statuteimposes on this right to privacy, it will violate due process absent a showing by the statethat the law promotes a compelling state interest.

B is incorrect as there is no suspect class involved.

C is incorrect because statutes are only presumed to have a sufficient relationship to agovernmental interest if tested by a rational basis test. The statute here will require astrict scrutiny test since a fundamental right is at issue.

D is incorrect as an incorrect statement of law. The burden of persuasion is not placedon the mother simply because the law is within the state's police power.

©2007-2008 Law Decks

Page 112: Law Decks Flash Cards - Constitutional Law - 2007-2008

Constitutional Law

Can Congress pass a bill limiting the President's power to pardon a person convictedunder a federal criminal statute?

Page 113: Law Decks Flash Cards - Constitutional Law - 2007-2008

Answer: No

The President has exclusive power to pardon. Article II, Section 2 of the Constitutiongrants this power to the President, and this power is not subject to control by Congress.

©2007-2008 Law Decks

Page 114: Law Decks Flash Cards - Constitutional Law - 2007-2008

Constitutional LawDoes a reporter for the Times, who refuses to provide sources to her editor, have a FirstAmendment right to have an article exposing impropriety of a local elected officialpublished?

A. Yes, under the Free Speech Clause.

B. Yes, a reporter's sources are protected under the First Amendment.

C. No, the newspaper is a private entity.

D. No, the freedom of the press has a wider range than the freedom of speech.

Page 115: Law Decks Flash Cards - Constitutional Law - 2007-2008

Answer: CNo. It is Congress who cannot abridge the freedom of the press as outlined in the FirstAmendment and this provision is applicable to the states via the FourteenthAmendment, Due Process Clause. In light of this, a court cannot compel a newspaperto either print, or refrain from printing, a prospective story. The Times, however, is notcompelled to print an article by a reporter as they are a private institution.

A is incorrect as the Freedom of Speech Clause only applies to the reporter. The Timescannot be compelled to publish her story, yet the reporter is free to distribute the storyherself by any other means.

B is incorrect as the sources of a reporter's information are not awarded protectionunder the Constitution.

D is incorrect as it incorrectly draws a connection between freedom of the press andFirst Amendment free speech. Here, as noted in the correct answer, the reporter'sfreedom of speech is not at issue.

52007-2008 Law Decks

Page 116: Law Decks Flash Cards - Constitutional Law - 2007-2008

Constitutional LawFollowing the passage of a city ordinance prohibiting residents from contributing to alocal political committee that supports ballot reform, the ordinance is challenged infederal court. How should the federal court rule on the constitutionality of this ordinance?

A. Strike it down, as it violates First Amendment free speech.

B. Strike it down as a violation of due process, as no hearing mechanism is providedfor.

C. Uphold it, as the city has a legitimate interest in controlling such contributions.

D. Dismiss the case, as it involves a political question and is, therefore, a non-justiciablematter.

Page 117: Law Decks Flash Cards - Constitutional Law - 2007-2008

Answer: AThe government may limit the amount of contributions to an individual's campaign, butit may NOT limit contributions to a political group. That would be a violation of FirstAmendment free speech and the law does not have a compelling enough interest toallow restraints on free speech.

B is incorrect. Due process does not require that every law provide for a hearing, butrather only those laws relating to the deprivation of life, liberty or property of anindividual. Here the law is being applied to ALL PERSONS, not to an individual.

C is incorrect as a legitimate interest is not enough, but rather a compelling state interestwould be required.

D is incorrect as non-justiciable questions arise when the issue at hand is not capableof resolution and enforcement by the judiciary. Presently, a validity question of the lawis at hand which is clearly within reach of the judiciary.

©2007-2008 Law Decks

Page 118: Law Decks Flash Cards - Constitutional Law - 2007-2008

Constitutional LawA city ordinance prohibits adult theaters from being located within 100 feet of a schoolzone or 1,000 feet of each other. This ordinance was passed on the basis that theaterslocated in objection to this ordinance would result in a negative residential neighborhoodenvironment, and is uniformly applied to all areas of the city. XXX Theaters owns twoadult movie theaters located such that they are in contradiction with the city ordinance.XXX Theaters files suit declaring the ordinance unconstitutional. The court should holdthat:

A. The zoning is a violation of the First Amendment.

B. The ordinance is invalid as it is a form of "spot zoning."

C. The ordinance is valid as it applies to all areas of the city.

D. The ordinance is valid as material protected by the First Amendment is subject tozoning and licensing requirements.

Page 119: Law Decks Flash Cards - Constitutional Law - 2007-2008

Answer: DThe court has held that although the First Amendment allows for the commercialexploitation of matters such as adult theaters, a city is not prevented from licensing orzoning these businesses in accordance with city regulations. A city may not, however,ban a commercial endeavor completely without violating the First Amendment.

A is incorrect as explained earlier. The First Amendment protects adult theaters; yetcities are still able to zone and license these institutions accordingly.

B is incorrect. Absent a complete ban of an industry by a city, spot zoning is an allowablepractice.

C is incorrect. Any ordinance banning an industry, even a ban evenly applied across thecity, is impermissible and a violation of the First Amendment.

©2007-2008 Law Decks

Page 120: Law Decks Flash Cards - Constitutional Law - 2007-2008

Constitutional LawA civic group, tired of what it deems immoral behavior of the military, decides to relocatein masses to a remote town in Colorado. Large numbers of this group flood the town andquickly outnumber residents that have lived there for decades. This civic group alsoquickly takes control of the town legislature. In protest against the war, they pass a townordinance preventing any military person or vehicle to pass through the town. Thisproves terribly inconvenient to a neighboring military base that typically uses Main Streetthrough town for access to the base. In light of the town's new law, can the militarycontinue to pass through town on its way to base?

A. Yes, the federal right to bear arms prevents legislation such as this as it interfereswith military rights.

B. Yes, the town is prevented from interfering with federal operations by the militarysuch as these.

C. No, as long as there exists an optional route the military may take that is notsufficiently more inconvenient.

D. No. Under the town's police power, they are free to prevent the military under theguise of protecting health, morals and safety of its citizens.

Page 121: Law Decks Flash Cards - Constitutional Law - 2007-2008

Answer: BThe town is clearly not allowed to regulate the actions of the federal government(military) absent congressional consent. Therefore, preventing access to Main Street isimpermissible. Furthermore, the federal government is immune from state law whileengaged in functions encompassing their federal function.

A is incorrect. The right to bear arms only protects the rights of citizens, not the USmilitary. Therefore, the government may not utilize the right to bear arms argument togain access through the town.

C is incorrect. State regulation of the federal government is impermissible when thefederal government is engaging in actions within the scope of its powers. Access to andfrom a military base is clearly within the scope of military powers. Alternative access isirrelevant.

D is incorrect. Even if the ordinance is a police power regulation, it can not interfere withthe legitimate functions of the federal government.

©2007-2008 Law Decks

Page 122: Law Decks Flash Cards - Constitutional Law - 2007-2008

Constitutional LawSuppose the new comprehensive healthcare reform statute that Congress passesestablishes an insurance review board authorized to subpoena records of insurancecompanies in order to determine if their fee structures are fair. The board does not haveany power, however, to fix fee increases and pricing. Suppose Florida has a healthcarefee pricing plan on its books that passed before the federal plan, which sets increasesat a maximum of 10% per year as reviewed by their state insurance board. Which wouldbe the best basis for finding the Florida provision unconstitutional?

A. The federal legislation supersedes the Florida plan even though it was passed afterthe state plan.

B. The Florida insurance board has similar powers to the federal board, but the Floridainsurance board also has the power to set fees and impose fines.

C. The Florida provision is a Contract Clause violation as it impairs contracts betweenhealthcare providers and insurers.

D. Healthcare fee caps result in an interstate commerce burden regardless of whetheror not there is a federal regulation.

Page 123: Law Decks Flash Cards - Constitutional Law - 2007-2008

Answer: BFlorida granting its healthcare board the power to fix fees, while the federal law did not, indicatesthat the state law violates the Supremacy Clause. Regardless of whether or not the state law directlyconflicts with the federal statute, interference by a state law in the federal scheme or in federalregulations, occupies the entire field. The more comprehensive a federal scheme is, the more likelya finding of implied preemption. Here, we have a comprehensive plan on the federal level absent afee fixing schedule. Such lawmaking implies that Congress knowingly and purposefully elected toleave it out. In light of this, the state trying to fix prices is a Supremacy Clause violation.

A is incorrect. The time frame of legislation is not the only factor in determining when a federal lawsupercedes a state law. A state regulation could be found permissible, regardless of when it waspassed, if it does not conflict with federal legislation or is not preempted.

C is incorrect. The Contract Clause is only applicable if there are substantial impairments of existingcontracts by state legislation. Additionally, the legislation must serve an important and legitimatepublic interest and must not be a narrowly tailored means of promoting that interest. Here, the lawonly has a prospective effect, and the other Contract Clause requirements will not be met.

D is also incorrect. State regulation of interstate commerce areas that have not been regulated bythe federal government are allowed so long as that regulation is nondiscriminatory and does notunduly burden interstate commerce. Here, we have a nondiscriminatory legislation and no factsindicating that there will be an undue burden on commerce.

52007-2008 Law Decks

Page 124: Law Decks Flash Cards - Constitutional Law - 2007-2008

Constitutional LawAustin decides to tear down some dilapidated buildings within the city and build a dogpark. The city puts the project out for bid, requiring that at least 50% of the biddingcontractors' crews for the project are city residents. Frank gets the project and beginswork. Inspector Pasquale learns that only 40% of Frank's employees are city residents.Inspector Pasquale gives Frank ten days to add another 10% before the contract istaken from him. Instead of hiring more city residents, Frank sues alleging that theemployment minimum is unconstitutional. Who should the court rule in favor of?

A. Austin, as it is a "market participant" here.

B. Austin, since there is a rational basis for favoring city residents in this case.

C. Frank, as this requirement is a violation of his Privileges and Immunities Clauserights.

D. Frank, as the requirement is a Contract Clause violation.

Page 125: Law Decks Flash Cards - Constitutional Law - 2007-2008

Answer: CThe Privileges and Immunities Clause protects builders in the current situation as theregulation interferes substantially with Frank's ability to earn a living. The Privileges andImmunities Clause of Article IV prevents state discrimination of civil rights againstresidents of other states that interferes substantially with commercial activity. Frank'sright to pursue his livelihood is considered a substantial civil liberty. Therefore, AnswerC is correct.

A is incorrect. The market participation exception is only applicable to a CommerceClause situation; it is not applicable to any situation in which the privileges andimmunities analysis is advanced.

B is incorrect because a rational basis test is also inapplicable. A Privileges andImmunities Clause violation can be overcome if there is a substantial justification for thediscrimination. Here, the proposed answer is indicating an application of the wrongstandard of review.

D is incorrect as the Contract Clause limits a state's ability to modify existing contractsretroactively and has no relationship to a state's power to regulate prospective contracts.Here, we are dealing with a legislation that existed before Frank's contract.

©2007-2008 Law Decks

Page 126: Law Decks Flash Cards - Constitutional Law - 2007-2008

Constitutional LawSandy City seeks to raise revenue, and builds a new town square which offers weeklyfireworks shows organized and produced by the city. These draw thousands of people and arewell received. A local philanthropic group, seeking to raise money for land mine victims, asksthe city if they can produce a fireworks show of their own. The city agrees and People AgainstLandmines (PALM) hires Joe Prucci, a fireworks artist of great renown. Prucci designs a show,heads to the square and begins to set up. A city official instantly shuts Prucci down, stating thathe can only use city-manufactured fireworks (not those of his own design) for fear that his ownfireworks will fly too far and set a nearby factory building on fire. (The city-manufacturedfireworks are known to fly a safe distance.) PALM files suit, seeking to allow Prucci to installhis own fireworks and produce his own show. The court will most likely:

A. Find for PALM since art is protected by the First Amendment and the city rule interfereswith Prucci's freedom of expression.

B. Find for Sandy City, since the fireworks show is not speech and, therefore, is notprotected by the First Amendment.

C. Find for PALM, since the city rule is not the least restrictive method for achieving the city'sgoals.

D. Find for Sandy City, since the rule is a reasonable time, place and manner restriction.

Page 127: Law Decks Flash Cards - Constitutional Law - 2007-2008

Answer: DSandy City will prevail because its rule is a reasonable time, place and manner restriction. Herewe are dealing with protected First Amendment speech, since speech is not limited to spokenwords but includes all things that communicate an idea. Content of speech generally can not belimited, but the conduct of speech in public forums can be regulated as long as there is areasonable time, place and manner restriction. Additionally, the restriction must be content neutral,narrowly tailored and leave open alternative channels of communication. Sandy City meets theserequirements: the images displayed by the fireworks are not controlled, just the means of showingthem; the rule is narrowly tailored and does not regulate substantially more speech than isnecessary to further a significant government interest (i.e.-preventing a factory fire); andalternative channels of communication are available via the city's equipment and fireworks.

A is incorrect. While art is protected by the First Amendment, it may still be regulated by reasonabletime, place and manner regulations, as indicated above.

C is incorrect. It states the wrong standard. To be valid, a time, place and manner regulation neednot be the least restrictive means for achieving the desired result, only narrowly tailored to theresult.

B is incorrect. Art is protected by the First Amendment. As discussed above, the First Amendment'sfreedom of speech guarantee protects more than merely spoken or written words; it includesconduct and other forms of expression undertaken to communicate an idea such as this fireworksshow.

52007-2008 Law Decks

Page 128: Law Decks Flash Cards - Constitutional Law - 2007-2008

Constitutional LawSuppose that Arizona, faced with high unemployment and an ever increasing illegalimmigrant population, decides to pass a statute hiring 200,000 new state employees inan attempt to decrease both problems. The statute provides a hiring bias toward thosewho can show at least two years of experience in the customer service industry andhave been recently laid off. It additionally stipulates that aliens can only be hired as stateemployees if there are no available citizens with a service industry background. Aworkers' rights group challenges the hiring bias toward those with a minimum two yearhistory in the customer service industry. Which of the following is most relevant?

A. The Privileges and Immunities Clause of Article IV

B. The Equal Protection Clause of the Fourteenth Amendment

C. The reserved powers of the state under the Tenth Amendment

D. The Privileges and Immunities Clause of the Fourteenth Amendment

Page 129: Law Decks Flash Cards - Constitutional Law - 2007-2008

Answer: BThe Equal Protection Clause of the Fourteenth Amendment is the most relevantbecause there is an established distinction between various groups: residents vs. aliens,individuals who worked in tourism vs. those who did not, workers with two yearsexperience vs. those without, etc.. .

A is wrong. The Privileges and Immunities Clause of Article IV has no bearing: the stateis not treating residents of other states differently.

C is not the best answer. The Reserved Powers Doctrine helps the state's argumentrather than hinders it.

D is wrong because the Fourteenth Amendment privileges do not include employmentas one of the protected privileges.

©2007-2008 Law Decks

Page 130: Law Decks Flash Cards - Constitutional Law - 2007-2008

Constitutional LawSuppose that North Bend, Indiana, a conservative religious community, protests theshowing of a critically acclaimed movie, The Creeks. Citizens of the North Bendcommunity feel that the movie is explicitly graphic and protest the nudity containedwithin. The movie has shown across the rest of the country only meeting critical acclaim.The community protests the local theater and seeks an injunction to prevent the theaterfrom showing the film. The theater owner refuses to voluntarily stop showing the film anddecides to appear in court to defend himself against the proposed injunction. What is thetheater owner's best defense?

A. The town's injunction is in violation of the Establishment Clause of the FirstAmendment which prevents the enforcing of a particular set of religious beliefs.

B. The proper "community standards" should be those of the entire state rather than ofthis single conservative town.

C. The film has proven artistic merit.

D. The film has a redeeming social value.

Page 131: Law Decks Flash Cards - Constitutional Law - 2007-2008

Answer. CThe First Amendment generally protects the right to freedom of speech, of which movies areone. Therefore, in order to prevent the showing of the film, the town must prove that thespeech involved is unprotected speech. The town's best argument would be that the speechis obscene in nature, which is defined as a depiction of sexual conduct that, taken as awhole by the average person using contemporary standards: 1) appeals to the prurientinterest in sex; 2) portrays sex in a patently offensive way; and 3) using a rational reasonableperson standard, does not have serious literary, artistic, political, or scientific value. Upon ashowing that the film does in fact have an artistic merit; it cannot be held to be obscene.

A is incorrect. The Establishment Clause forbids the government from adopting a law thatestablishes religion. Here, the town is not trying to enforce a religious view, but trying toprohibit obscenity. Prohibiting obscenity is a legitimate interest, regardless of whether thisalleged obscenity coincides with the beliefs of a particular religious group.

B is not as good an argument as Answer D. While a statewide community standard may beused, it is not mandatory. A local community standard is sufficient to evaluate the "patentlyoffensive" area of the law.

D is not as good an argument as Answer C because it is not sufficient that there is someredeeming social value. There must be serious redeeming value to bypass the standards ofthe test set forth in Answer C.

©2007-2008 Law Decks

Page 132: Law Decks Flash Cards - Constitutional Law - 2007-2008

Constitutional LawA devoutly religious township board in Pennsylvania votes to allow elementary schoolteachers to inflict corporal punishment on misbehaving students, in accordance withtheir religious beliefs. Elaine, a student caught chewing gum in school, is promptly hitacross the knuckles with a wooden ruler for her disobedience. Elaine's parents, theDolacks, do not believe in corporal punishment, and are disturbed to learn of thispunishment by one of her teachers. They sue the teacher under a federal statuteproviding for damages against any government employee (i.e.—a teacher) who deprivesa person of her constitutional rights, and allege the following rights violations:

I. Her First Amendment rights under the Establishment Clause

H. Her right to procedural due process under the Fourteenth Amendment

III. Her right to be free from disproportionate punishment under the Eighth andFourteenth Amendments.

Which of the above will the Dolacks likely prevail on?

A. I and II B. I and III C. II and III D. None of the above

Page 133: Law Decks Flash Cards - Constitutional Law - 2007-2008

Answer: DNone of the Above

Statement I is inapplicable as there is no Establishment Clause violation. The clausestates that government action will be upheld if the action: serves a secular purpose; itsprimary effect neither advances nor inhibits religion; it does not excessively entanglegovernment with religion; and there is no sect preference. Here, the secular purpose ismaintaining order in the classroom regardless of the fact that it coincides with the townboard's religious beliefs. Additionally, the main purpose neither advances nor inhibitsreligion and there is no excessive entanglement. Finally, there is no sect preferenceunder the school board's corporal punishment rule, because there has been nodeprivation of procedural due process.

Statement II is incorrect as no hearing is required prior to the inflicting of corporalpunishment, regardless of the fact that the Supreme Court has held that corporalpunishment may involve a liberty interest.

Statement III is incorrect as there is no Eighth Amendment violation. Hitting students asa disciplinary action has not been found to be cruel and unusual punishment.

©2007-2008 Law Decks

Page 134: Law Decks Flash Cards - Constitutional Law - 2007-2008

Constitutional LawA chicken producing state, seeking to protect its own chicken farmers, adopts a statuterequiring any food service business operating in the state to serve chicken raised in theUnited States. Such a statute aims to prevent the sale of less expensive imported foreignchicken. A corner chicken kebab salesman faces a loss in profits following his forcedswitch to US chicken, as the foreign chicken he has been serving costs him 30% lessthan the US chicken he must now serve. He speaks to his attorney, who does a bit ofresearch, and they realize that all fertilizer purchased by the state parks is derived fromthe waste of foreign chicken farmers. Which is the best argument against theconstitutionality of the statute?

A. The statute burdens foreign commerce.

B. The statute violates equal protection guarantees since it is irrational to prohibit thesale of foreign chicken meat but not foreign chicken fertilizer.

C. The statute substantially interferes with the right of a citizen to earn a living underthe Privileges and Immunities Clause of the Fourteenth Amendment.

D. The statute constitutes a taking without due process of law.

Page 135: Law Decks Flash Cards - Constitutional Law - 2007-2008

Answer: AThe burden on foreign commerce argument is the best. Practically speaking, the powerto regulate foreign commerce lies exclusively with Congress; therefore, the stateregulation of citizens' purchases is likely a violation of state powers.

B is incorrect because the statute is a rational method of protecting local interests. Therational basis standard applied, typically results in the Supreme Court deferring to thelegislature's decision that the law is rational, regardless of the fact that the statute failsto include aspects of foreign commerce such as fertilizer sales. Statutes merely mustaddress some of the problems that prompted their passage. The fact that the statute isactually under-inclusive is irrelevant.

C is incorrect because the Fourteenth Amendment does not protect a right to earn aliving. The Privileges and Immunities Clause protects against infringement of rights ofnational citizenship.

D is incorrect because the chicken kebab salesman had all of the process that was duehim. There is no need for an individual hearing, as the statute was generally applied andnot individualized to those in the same group as the chicken kebab salesman.

©2007-2008 Law Decks

Page 136: Law Decks Flash Cards - Constitutional Law - 2007-2008

Constitutional LawThe state of Vermont requires all drivers to have liability insurance which is extremelyexpensive due to the high number of accidents. Investigating the actual accidents, thestate found that men under 21 had the largest number of accidents and directly raisedthe rates of all others in the state by more than 10%. The state changes the driving agerequirement from 18 years to 21 years for males, but leaves it at 18 years for females.Woody, an 18 year old taxi cab driver now no longer allowed to drive, is fired andsubsequently replaced with an 18 year old female driver. If Woody sues to have the lawset aside and prevails, it will most likely be because the state could not prove that thelaw was:

A. The least restrictive means of achieving a compelling government purpose.

B. Rationally related to a legitimate government purpose.

C. Necessary to achieve a compelling government purpose.

D. Substantially related to an important government interest.

Page 137: Law Decks Flash Cards - Constitutional Law - 2007-2008

Answer: DWoody will prevail if the state cannot establish that the restriction is substantially relatedto an important government interest. General classifications are tested against anintermediate standard of review, requiring the state government to show a persuasivejustification that the classification is substantially related to an important governmentinterest.

Answers A and C are substantially the same and both incorrect because they state thestandard to be applied to classifications involving a suspect class or a fundamental right.Gender is not a suspect classification, but rather exists as a quasi-suspect classification.

B is incorrect because it states the standard to be applied when there is neither afundamental right, nor a suspect or a quasi-suspect class involved.

©2007-2008 Law Decks

Page 138: Law Decks Flash Cards - Constitutional Law - 2007-2008

Constitutional LawSuppose that New Jersey decides to lease the county-owned land surrounding HorseshoeLake to the Horseshoe Co., a private company, for development of a beach club and marinain exchange for 5% of its yearly profits. Horseshoe Co. proposes a set of bylaws that restrictaccess to members only. Such membership would be handled by Horseshoe Co.'s board ofdirectors. Having reviewed these bylaws, New Jersey approves. Weeks prior to opening theclub, the board of directors decides that membership will require a $5,000 application fee anda $500 monthly membership fee. Aside from the fee requirements, people of all nationalities,ethnic backgrounds and religions are welcome to join. A resident who could not afford theapplication and monthly membership fees brings suit against the Horseshoe Co., alleging thatthe fees are in violation of the Equal Protection Clause for discriminating against the poor. Whowill prevail?

A. Plaintiff. A person cannot be deprived of a public right solely because they cannot affordto pay for it.

B. Plaintiff, because the poor are considered a protected class.

C. Horseshoe Co., because only de facto discrimination against the poor is a violation of theEqual Protection Clause.

D. Horseshoe Co., because the membership privilege does not qualify as a sufficient equalprotection deprivation.

Page 139: Law Decks Flash Cards - Constitutional Law - 2007-2008

Answer: DA violation of equal protection, in light of the denial of rights, has only been found whenthe rights denied are particularly important to those who cannot pay. Failing to join acountry club is clearly not a particularly important denial of rights. In light of this, AnswerA is clearly incorrect.

C is wrong because a number of de facto discriminations against the poor have beenfound to be violations of the Equal Protection Clause of the Fourteenth Amendment.

B is an incorrect statement of the law.

©2007-2008 Law Decks

Page 140: Law Decks Flash Cards - Constitutional Law - 2007-2008

Constitutional LawA private housing development of two and three-bedroom units has a deed restrictioncontained within the executed deed of each unit. It limits ownership and occupancy ofeach unit to groups of unrelated adults consisting of no more than three persons, orfamilies. An unmarried couple purchases a three-bedroom unit and immediately movesin along with another unmarried couple who are their friends. Their neighbors promptlybring suit seeking to prevent the new owners from having their unmarried friends livingwithin the development. Assuming judgement is in favor of the new owners, which of thefollowing was likely the pivotal question at issue in the case?

A. Did the deed restriction constitute an impermissible restraint against alienation?

B. Is enforcement of the restriction a violation of the Equal Protection Clause of theFourteenth Amendment?

C. Did the new owners give notice to their neighbors of their intention to have anotherunmarried couple live with them?

D. Can the three-bedroom unit at issue easily house four adults?

Page 141: Law Decks Flash Cards - Constitutional Law - 2007-2008

Answer: BHere we are faced with discrimination against unmarried adults. Supreme Courtprecedent teaches that restrictions in deeds based on race are not enforceable. Suchprecedent has resulted in several states striking down other discriminatory provisions inzoning laws. In light of this, a similar argument may be advanced in the present factpattern, yielding Answer B as the correct answer.

A is not the best answer, as the problem at issue is not alienation of two or more people,but rather occupancy restrictions.

The facts presented in Answers C and D are irrelevant and clearly erroneous.

©2007-2008 Law Decks

Page 142: Law Decks Flash Cards - Constitutional Law - 2007-2008

Constitutional LawSuppose that New York requires any retail business within the state to be licensed.Additionally, via an amendment to the original statute, any licensed retail store isrequired to establish that 25% of goods sold are manufactured within the state of NewYork. Which of the following is the best constitutional argument by which to attack thevalidity of the in-state manufacturing provision?

A. The Privileges and Immunities Clause of the Fourteenth Amendment

B. The Commerce Clause

C. The Equal Protection Clause

D. The Due Process Clause of the Fourteenth Amendment

Page 143: Law Decks Flash Cards - Constitutional Law - 2007-2008

Answer: BThis statute negatively affects interstate commerce as it diverts the purchase of itemsfrom interstate to intrastate commerce. Such diverting of purchases toward intrastatecommerce discriminates against interstate commerce sales.

A is incorrect because the Privileges and Immunities Clause does not apply. TheFourteenth Amendment does not recognize the sale of products as a "privilege"protected by the Privileges and Immunities Clause.

C is incorrect as the statute applies to all businesses, such that an equal protectionclassification is not triggered.

D is incorrect as the license requirement is within the bounds of traditional state policepowers. In light of this, the state would be reasonable, with the exception of theCommerce Clause problems noted in the correct Answer B.

©2007-2008 Law Decks

Page 144: Law Decks Flash Cards - Constitutional Law - 2007-2008

Constitutional LawA state has enacted a statute requiring parents to vaccinate their children against the flueach year, or be subject to a $1,000 fine or 1 year in jail. The Smiths object to this statuteon the grounds that it violates their right to free exercise of religious beliefs under theFirst Amendment and have filed suit in Federal Court. The best argument by the state toconvict Mr. or Mrs. Smith is:

A. The Free Exercise Clause only applies to a belief not a conduct.

B. The statute is a neutral law regardless of the fact that it may burden the FirstAmendment rights of Mr. or Mrs. Smith.

C. The state has a substantial interest in the health of its citizens and this is the onlyreasonable way to foster this state interest.

D. An exception for one party would be in violation of the Establishment Clause of theFirst Amendment.

Page 145: Law Decks Flash Cards - Constitutional Law - 2007-2008

Answer: BIf the law at issue is neutral in application and uniformly applied to all parents and/or guardians thenthe Free Exercise Clause does not award a religious exemption to a single group; regardless of thefact that the law may tangentially impose a burden on that group's religious practice. The state mayallege that since the law at issue fosters the health of all residents, is uniformly applied to all parentsand/or guardians, and is not there to specifically impose upon the religious activities of one group: itis permissible.

A is incorrect as conduct is, in fact, protected under the Free Exercise Clause. This protection,however, is limited. In the present situation, in which there is conduct which equally affects religiousand non-religious groups, the Free Exercise Clause does not award the Smiths protection under thelaw.

C is incorrect as it recites the former balancing test applied by the courts in addressing whether thereligion exemption should be applied. This former balancing test required a review of the severity ofthe burden, the strength of the state's interest and the existence of alternatives. Presently, however,the balancing test is no longer applied; therefore, assuming the challenged statute is neutral, thestate need not show the above requirements.

D is incorrect. While a law giving an exemption to one religious sect absent a narrow tailoring toprotect a compelling interest would be typically held invalid, the state may argue the following: Thelaw at issue does not appear to have a sect preference, but possesses a secular purpose that neitheradvances not hinders religious expression, while simultaneously avoiding excessive governmententanglement. In light of this, there would not be a violation of the Establishment Clause.

©2007-2008 Law Decks

Page 146: Law Decks Flash Cards - Constitutional Law - 2007-2008

Constitutional LawSuppose, faced with staggering unemployment, Ohio offers a $100,000 prize to anyone who cansubmit a plan to develop 1,000 jobs within the state. Aaron, an engineer hiking through an Ohiostate park, notices a dark rock that he picks up and discovers is coal. He takes the rock backhome with him and the following day takes it to the local government, explaining how heproposes to mine the area and create the 1,000 jobs for the prize. The state government loveshis idea and he becomes an instant celebrity carried on the local news. A disgruntled parkranger (whose wife left him for an engineer), sees Aaron's TV coverage and instantly arrests himin violation of a federal law making removal of "plants, minerals, or animals" from park lands apunishable offense. Aaron is fined $10,000 and convicted. He then appeals, claiming the fineunconstitutional. How should the court rule?

A. For Aaron, since removing the coal has no effect on interstate commerce as it was solelyintrastate.

B. For the government, since the federal statute providing for the fine is constitutional under theCommerce Clause.

C. For Aaron, since the state has a compelling interest in reducing unemployment and thefederal statute unreasonably interferes with the state interest. '

D. For the government, as Property Clause of Article IV, Section 3 of the federal Constitutionsupports the federal statute preventing removal of rocks from the park.

Page 147: Law Decks Flash Cards - Constitutional Law - 2007-2008

Answer: DThe fine is constitutional under the Property Clause which gives Congress the power to"make all needful rules and regulations respecting the territory Property Clause of ArticleIV, Section 3 of the federal Constitution or other property belonging to the United States."

B is not as good an answer as Answer C because the Commerce Clause is not asapplicable as the Property Clause in the present set of facts.

A is wrong because the fine could probably be upheld under the Commerce Clause.The Commerce Clause allows Congress to regulate any act (even intrastate activities)that may itself, or in combination with other activities, have a substantial effect oninterstate commerce. Arguably, if every visitor removed a rock from federal land, therewould be substantial effect on interstate commerce.

C is incorrect because, as the Supremacy Clause provides, any act of Congresssupersedes any state or local action that actually conflicts with the federal rule. Ohio'scompelling interest is inapplicable to the facts at hand in light of a Supremacy Clauseanalysis.

©2007-2008 Law Decks

Page 148: Law Decks Flash Cards - Constitutional Law - 2007-2008

Constitutional LawA state has enacted a statute requiring parents to vaccinate their children against the flueach year, or be subject to a $1000 fine or 1 year in jail. The Smiths object to this statuteon the grounds that it violates their right to free exercise of religious beliefs under theFirst Amendment, and have filed suit in federal court. The best argument by the stateagainst the Smiths' position is:

A. A declaratory judgment is the incorrect vehicle.

B. There is no substantial threat that the statute will be enforced.

C. State courts are given the first opportunity to construe the statute, and federal courtsshould, therefore, abstain.

D. Bills to repeal the statute have been repeatedly defeated.

Page 149: Law Decks Flash Cards - Constitutional Law - 2007-2008

Answer: BIf there is no threat that the statute will be enforced, then there are no constitutionalissues that are ripe for review. A federal court may not decide the matter, absent ashowing that the person (should s/he engage in the prohibited activity) is likely to beadversely affected, and is in real and immediate danger due to the statute. If the statuteis not likely to be enforced; it is impossible to provide a showing of real or immediatedanger and the case is not ripe for federal review.

A is incorrect. Should there be a showing of ripeness, a declaratory judgment would bean appropriate vehicle.

C is incorrect. Abstention should only occur when a constitutional claim is based uponan unsettled question of state law. Here, there exists no unsettled question of state law;thereby, precluding an abstention by the federal court.

D is also incorrect. A showing that the state has refused to repeal the statute indicatesthat the state intends to enforce the statute. Such a showing supports the Smiths' caseas it tends to show that the issue at hand is, in fact, ripe for review.

©2007-2008 Law Decks

Page 150: Law Decks Flash Cards - Constitutional Law - 2007-2008

Constitutional LawSuppose that DELIVERME transports packages in the state of Pennsylvania using afleet of trucks that it owns. The trucks are registered in Pennsylvania and never leave itsboarders. These trucks were purchased from Pennsylvania car dealers and only pick uppackages for companies that are based in Pennsylvania. From here they take the boxesto an express delivery terminal at Pittsburgh International Airport where commercialairline employees load them onto the plans. DELIVERME charges each person thatships a 2% fee for handling their packages. Pennsylvania seeks to impose a 5% tax onthese collected fees garnished from Pennsylvania shippers. The tax will likely be found:

A. Constitutional. The tax is imposed prior to the packages entering the stream ofcommerce.

B. Constitutional. The tax may be severed from any effect it has on interstate commerce.

C. Unconstitutional. The packages have already entered the stream of commerce and itis too late to tax them.

D. Unconstitutional, because the tax may result in multiple taxation of DELIVERME.

Page 151: Law Decks Flash Cards - Constitutional Law - 2007-2008

Answer: BThe tax here occurs in a manner that does not burden interstate commerce and is not discriminatory.What we have here is a transactions tax, which essentially is a tax on the privilege to do business inthe state of Pennsylvania. The tax is permissible by meeting the following criteria: it does notdiscriminate against interstate commerce; the tax activity has a substantial nexus to the taxing state;the tax is fairly apportioned; and the tax fairly relates to services provided by the taxing state. Here,there is no discrimination as there are no facts to indicate that the tax is being imposed solely oninterstate commerce shipments or solely on interstate shippers. There is a substantial nexus betweenthe taxed activity and the state as the transaction tax is on fees paid by shippers for transport of thepackages within Pennsylvania. The tax is fairly apportioned since it is only 5% of the fees collected byDELIVERME's local activity and has no interstate commerce implications. Finally, DELIVERME's useof Pennsylvania roads and airports indicates that the tax relates to services that the state provides.

A is wrong for setting forth the test for an ad valorem property tax. Taxing packages already in thestream of commerce is invalid. However, the question here is related to a transport tax, making the testset forth in the correct answer appropriate.

C is incorrect. As we are dealing with a tax on a shipping transaction, whether the packages are in theinterstate commerce stream is not the factor that concerns us here. As set forth in the correct answer,the test in Answer B is appropriate, and a determination if the item is in interstate commerce isinapplicable.

D is wrong because the tax here is only related to local activities. Under the proposed tax regime, thereis no means by which DELIVERME can be exposed to multiple taxations.

©2007-2008 Law Decks

Page 152: Law Decks Flash Cards - Constitutional Law - 2007-2008

Constitutional LawSuppose that with crime soaring across the country, Congress creates the National Door LockAgency (NDLA) that sets regulations for all locksmiths and further requires that they all belicensed. All existing locksmiths are grandfathered in, while those that decide to go into businessafter the passage of the law must show two consecutive years experience with a licensedcompany. Installation by those without licenses will result in a fine. Sidney has been installingdoor locks for two decades, but sells her company to Lockright six months before the regulationby Congress. She continues to work for Lockright two days a week to remain busy. Six monthslater, she feels that she is too young to semi-retire and decides to go back to work for herself.She applies for a license and is denied, as she has only been a locksmith employed by Lockrightfor a total of six months since the regulation. Can Sidney by fined?

A. Yes, because the NDLA was established under congressional general welfare powers, andCongress may take whatever steps are necessary and proper to enforce its laws.

B. No, because the regulation interferes with Sidney's fundamental right to earn a living and soviolates the Privileges and Immunities Clause of Article IV, Section 2.

C. No, because the government agency cannot itself levy fines for a violation of its regulations.

D. Yes, because the regulation falls within the scope of congressional commerce powerallowing Congress to regulate as it has done here.

Page 153: Law Decks Flash Cards - Constitutional Law - 2007-2008

Answer: DSidney can be fined. Congress has the power to regulate in this area under theCommerce Clause, as Congress has the power to regulate any local or interstate activitythat has an effect on interstate commerce, either in itself or in combination with otheractivities. The establishment of NDLA is valid since Congress is free to delegate itslegislative power broadly.

A is incorrect as Congress does not have the power to legislate for the general welfare:only to spend for the general welfare.

B is incorrect because the Privileges and Immunities Clause of Article IV restricts states,not the federal government.

D is incorrect because Congress is free to impose fines for violation of an agency'sregulations.

©2007-2008 Law Decks

Page 154: Law Decks Flash Cards - Constitutional Law - 2007-2008

Constitutional LawSuppose that New Mexico's public schools are based primarily on property tax revenues.Some districts that have property tax revenues below what is essential to run their schools,and which happen to be in primarily Latino areas, receive a state subsidy. In 2004, facing astate-wide economic crisis, these subsidies are cancelled completely via a new state statute.A Latino family in a district that used to receive a subsidy decides to sue, claiming a violationof the Equal Protection Clause of the Fourteenth Amendment. Which of the following is theappropriate standard the court will apply in reviewing the case?

A. Faced with state discrimination against a suspect class due to the statute, New Mexico willhave to demonstrate that the statute is necessary to vindicate a compelling state interest.

B. Because right to education is not a fundamental right, the parents will have to demonstratethat the statute is not substantially related to an important state interest.

C. Because no suspect class or fundamental right is burdened by this new statute, the parentswill have to demonstrate that the statute is not rationally related to any legitimate stateinterest.

D. Because the state statute is not discriminatory in intent, the state will have to demonstrateonly that the statute is rationally related to a legitimate state interest.

Page 155: Law Decks Flash Cards - Constitutional Law - 2007-2008

Answer: CThe rational basis test is the appropriate standard with which to evaluate. Under that test, alaw is presumed to be valid unless the challenger can make a showing that it is not rationallyrelated to a legitimate state interest. This is a notoriously difficult burden for a challenger tomeet. Here, the statute did not target a suspect class and did not burden a fundamentalright, so the rational basis test applies.

A is incorrect as showing that legislation has a discriminatory effect on a suspect class isnot enough. One must show that there is intent to discriminate. When dealing with racial,national origin, or ethnicity classification, the challenger must show that: 1) the racialclassification appears in the law itself; 2) the law was applied in a purposefully discriminatorymanner; and 3) the law was enacted or maintained for discriminatory purpose. None ofthese situations apply to the current statute.

B is incorrect because it states the wrong standard. The Supreme Court has not heldeducation to be a fundamental right under the Due Process Clause, nor has it found wealthclassifications to require a strict scrutiny analysis. The standard in Answer B is anintermediate scrutiny test that is applied to gender and legitimacy classifications.

D is wrong for burdening the wrong party with the burden of proof. When evaluating astatute that does not discriminate against a suspect class, the plaintiff bears the burden ofproof.

©2007-2008 Law Decks

Page 156: Law Decks Flash Cards - Constitutional Law - 2007-2008

Constitutional LawVin, an automotive engineer, develops a carburetor that will enable cars to achieve 150miles per gallon of fuel. One of Vin's former employees brings an action to prohibit himfrom displaying the carburetor at an upcoming industry show, claiming that Vin hasstolen his design. The court grants a temporary restraining order prohibiting Vin fromdisclosing any details of his carburetor and orders a hearing to be held in one week. Atthe hearing it will be essential to reveal the intricacies of the design, so Vin seeks to havethe hearing closed. The court grants the request. Terry, a reporter for Car EngineeringWeekly, files an action to have the hearing opened so that she can attend and reportabout this new invention in the upcoming magazine. What is Terry's best argument foropening the hearing?

A. Closure is unnecessary to preserve an overriding interest in this situation.

B. Closure, here, amounts to a prior restraint.

C. Under the Fairness Doctrine, Car Engineering Weekly will be required to give eachlitigant an opportunity to present his or her side of the case.

D. The right of freedom of the press is extensive and allows the press to attend ahearing such as this, since it is of great interest to the public.

Page 157: Law Decks Flash Cards - Constitutional Law - 2007-2008

Answer: ATerry's best argument is that the closure here is not necessary to preserve an overridinginterest, as trials and pretrial hearings are generally open to the public. In criminal cases,the Supreme Court has held that a closed trial will only occur if necessary to preserve anoverriding interest and if the closure order is additionally narrowly tailored. While notformally addressed by the Supreme Court, it is well established that a civil hearing suchas this will more likely than not follow the court guideline when dealing with a criminalmatter.

B is not as good an argument. While closure, here, would amount to a prior restraint, thisrestraint would be justified if it was narrowly tailored to achieve a compelling interest. Asset forth in Answer A, this narrow tailoring is necessary; therefore, Answer B is not as goodas Answer A.

C is incorrect. The Fairness Doctrine is irrelevant to the issue of whether a hearing shouldbe open to the public. The Fairness Doctrine simply applies to political candidate coverageas promulgated by the FCC.

D is a false statement of the law and, therefore, incorrect. Freedom of the press does notallow the press unlimited access and is not absolute. Courts can close hearings inaccordance with Answer A in situations where there is an overriding interest in protectingthe privacy of the parties.

©2007-2008 Law Decks

Page 158: Law Decks Flash Cards - Constitutional Law - 2007-2008

Constitutional LawSuppose that Congress is concerned about pesticide runoff into rivers and waterways fromlawns and golf courses. In response to this, they adopt a law prohibiting the use ofpesticides in regions that meet a population density and are near waterways likely to beinfected with pesticide runoff. Suppose Baton Rouge is suffering through a West Nileepidemic and passes a law, after consulting with the state department of health, allowing theapplication of Agent Apricot (a pesticide) to combat the mosquito population in deference tothe congressional law which would ordinarily prevent them from pesticide application. KaraChambers, a Baton Rouge environmentalist, sues seeking to overturn the state law.Assuming that Chambers has standing, the court will most likely find the city ordinance:

A. Valid, because controlling health hazards is an integral governmental function.

B. Invalid, because laws passed by states are superseded by the power of Congress toadopt laws to protect the health, safety and welfare of citizens.

C. Valid, because under the police power, cities have a compelling interest in law designedto protect the health, safety and welfare of their citizens.

D. Invalid, because it conflicts with a federal law that Congress had the power to make underthe Commerce Clause.

Page 159: Law Decks Flash Cards - Constitutional Law - 2007-2008

Answer: DCongress has broad power when regulating commerce, allowing them to regulate anymatter that has substantial economic effect on interstate commerce. If Congress hasfound pesticide runoff into waterways (an instrument of interstate commerce) to have anoverall detrimental impact on the environment, then regulation is permissible underSupreme Court standards.

A is incorrect because state and local government activities that affect governmentalfunction may be regulated by a general law as long as there is a constitutional basis forthe law.

B is incorrect because Congress does not have a general "police power" to adopt lawson health and safety. Public health-related laws passed by Congress are generallypassed based on the congressional interstate commerce powers.

C is incorrect. While the power to regulate health and safety (police power) ispermissible by a state; their power, if it conflicts with a federal law, becomes invalidunder the Supremacy Clause.

©2007-2008 Law Decks

Page 160: Law Decks Flash Cards - Constitutional Law - 2007-2008

Constitutional LawA private high school that has 40% religious curriculum applies for state-offered modellesson plans for various other subjects. These model lesson plans are freely offered toany school or district within the state who requests them. What is the best argument ofthe state providing these model lesson plans to this religious-affiliated private school?

A. The teaching of the 40% religious curriculum is not constitutionally prohibited.

B. The providing of model lesson plans to schools within the state is only for secularpurposes, such as improving the learning experience of children, and has noreligious purpose.

C. Private religious institutions provide a necessary and beneficial purpose.

D. Should the state refuse to provide these plans, they would be in violation of the FreeExercise Clause of the First Amendment in light of the discrimination of these privatereligious school students over private secular students.

Page 161: Law Decks Flash Cards - Constitutional Law - 2007-2008

Answer: BAs long as the government's actions when dealing with a religious school do not havean expressed religious purpose, and neither detrimental nor beneficial effect on religion;they are permitted. Additionally, the government's actions in dealing with the privatehigh school do not result in excessive governmental entanglement with religion. So,again, the state-offered model lesson plans would be permissible.

A is incorrect as it is immaterial to the question at hand. The teaching of religious contentin a private school is not prohibited and the percentage taught not regulated.

C is incorrect as the necessary and beneficial purpose, while it may be true, is unrelatedto the matter at hand.

D is incorrect as the Establishment Clause prohibits the state from providing certainkinds of aid to religious institutions. If a state were to elect not to aid a religious schoolin these situations; it would not be a violation of the Free Exercise Clause.

©2007-2008 Law Decks

Page 162: Law Decks Flash Cards - Constitutional Law - 2007-2008

Constitutional LawMeta, a House Ways and Means Committee member, seeks to raise additional revenuefor the government by adding a surcharge to all airline tickets purchased for flight out ofthe nation's ten busiest airports. The tax passes. The airlines flying from these ten citiesare outraged and protest loudly. They challenge the validity of the tax in federal court.The tax will most likely be held:

A. Unconstitutional, because it is not rationally related to raising revenue within the USas travel from these airports will likely decrease.

B. Unconstitutional. The tax unfairly discriminates against large departure cities bytaxing them, and not taxing other similar departure airports.

C. Constitutional. Congress has plenary power to impose taxes in order to raiserevenue.

D. Unconstitutional, because it makes intrastate travel more difficult.

Page 163: Law Decks Flash Cards - Constitutional Law - 2007-2008

Answer: CCongressional taxing power will likely allow this tax to be constitutional. Article II,Section 2 taxations are typically upheld if they bear some reasonable relationship torevenue production, or if Congress has the power to regulate the taxed activity. Here,the tax does appear to be related to revenue generation.

A is incorrect because whether the tax actually does have a revenue-enhancing effectis irrelevant. Merely alleging that a revenue generation stream exists is enough.

D is incorrect because the right to travel has only been defined to include the right totravel from state to state and may not be deterred by state legislation. Federalgovernment actions such as these do not affect the right to travel from state to state andare typically viewed constitutional by the Supreme Court.

B is incorrect. The discrimination, here, is not unfair and not in violation of the DueProcess Clause of the Fifth Amendment, as there is no suspect class or fundamentalright at issue. The tax merely needs to pass a rationally related test that identifies alegitimate governmental interest. The government has a legitimate interest in taxing, andso the tax will be upheld.

©2007-2008 Law Decks

Page 164: Law Decks Flash Cards - Constitutional Law - 2007-2008

Constitutional LawSuppose, faced with staggering unemployment, Ohio offers a $100,000 prize to anyone who cansubmit a plan to develop 1,000 jobs within the state. Aaron, an engineer hiking through an Ohiostate park, notices a dark rock that he picks up and discovers is coal. He takes the rock backhome with him, and the following day takes it to the local government, explaining how heproposes to mine the area and create the 1,000 jobs for the prize. The state government loveshis idea and he becomes an instant celebrity carried on the local news. Aaron is hired by thefederal government to build a coal refining plant in accordance with federal standards on federalland. There is an Ohio statute stating that all refineries must be state-inspected. Upon inspectionthe plant fails to pass, as Ohio standards are higher than those imposed by the federalgovernment. Aaron continues to operate the plant, even without state approval, and is finedheavily. Which of the following is Aaron's best defense?

A. The state lacks a compelling interest in regulating the plant as it is within a federal park.

B. Congress has preempted the field of pollution control; therefore, the state regulation isinvalid since areas in which Congress has elected to act cannot be subsequentlylegislated on by the state.

C. The state pollution regulation is invalid since it is inconsistent with Ohio's compellinginterest in providing jobs.

D. The state law violates the principles of inter-governmental immunity as applied to an engineer.

Page 165: Law Decks Flash Cards - Constitutional Law - 2007-2008

Answer: D

The states have no power to regulate the activities of the federal government unlessCongress consents to the regulation. Here, there has been no consent by Congress forthe regulation. Even the allowance of a state inspector to evaluate the plant fails toindicate a congressional consent, as there is nothing in the fact pattern stating thatCongress has consented.

A is factually wrong and irrelevant. Arguably, the state interest in pollution is compelling,but it is irrelevant as Ohio is simply not allowed to interfere with federal activities onfederal land.

B is incorrect because not enough facts are given to make this determination. While theSupremacy Clause prohibits states from adopting laws that interfere with federal lawswhen Congress has preempted the field; a factual determination is necessary toevaluate if Congress has, in fact, preempted the field. Here, there are simply not enoughfacts.

C is incorrect as irrelevant. There is no requirement in the Constitution for a state to favorone compelling interest over another. States are free to adopt laws as they please, aslong as they are not arbitrary (a violation of substantive due process). The laws here donot appear to be arbitrary, and so Answer C does not offer a viable defense.

©2007-2008 Law Decks

Page 166: Law Decks Flash Cards - Constitutional Law - 2007-2008

Constitutional LawGarden State University has a longstanding practice of allowing students to use classrooms fornon-school-related meetings as long as they are not currently occupied by established classes.Virtus, a religious student group that believes males are a superior race and do not allow womento join, signs up to use one of the classrooms for its weekly meetings in which they discussbusiness matters of the religion as well as the religious tenets themselves. They are denied bythe university. GSU claims that Virtus's religious meetings would be in violation of a state statuteforbidding the use of public facilities by those who discriminate on the basis of race or gender.Virtus brings an action in federal court challenging application of the statute to them. Assumingthe court finds the actions of GSU valid, it would most likely be because:

A. Permitting a religious group to hold the meeting in a public school facility would violate theEstablishment Clause under the Fourteenth Amendment.

B. The right of freedom of association does not apply to groups involved in business andcommercial activities.

C. Allowing student groups to use unoccupied classrooms is not a Fourteenth Amendment stateaction.

D. The statute, by forbidding discriminatory groups from using public facilities, is the leastrestrictive means of advancing the state's compelling interest in ending discrimination ofwomen.

Page 167: Law Decks Flash Cards - Constitutional Law - 2007-2008

Answer: DGarden State University will prevail on the basis that the statute, by forbidding discriminatory groupsfrom using public facilities, is the least restrictive means of advancing the state's compelling interestin ending discrimination. Traditionally, schools are not viewed as open public forums, but in light ofGSU's policy allowing students to use unoccupied classrooms, the school is deemed a public forum.Once a public forum, there is a clear First Amendment implication which may be violated if a schoolrestricts use based on content of speech. The only manner in which the school may justify thecontent-based regulation of speech is by showing a compelling state interest that can not be satisfiedby less restrictive means. Additionally, the state may justify the denial of a right to an associationbased upon a showing of a compelling state interest which can not be achieved through meanssignificantly less restrictive. In light of the facts, the state restrictions can be viewed as compellingin order to prevent gender-based discrimination.

A is incorrect. A violation of the Establishment Clause does not occur when a school permits areligious group the same after-class access to its facilities that other student groups enjoy.

B is incorrect as the First Amendment right of groups to join together to express their opinions andaddress business matters is applicable to the matter at hand. The weekly meetings in which businessis discussed is related to Virtus's religion. Therefore, we have both a freedom of religion question aswell as a freedom of expression question at hand. In light of this, the state is burdened with showinga compelling interest in quashing the group's right to freedom of speech and expression.

C is wrong because the acts of GSU in allowing or denying access to classrooms is prima facia stateaction.

©2007-2008 Law Decks

Page 168: Law Decks Flash Cards - Constitutional Law - 2007-2008

Constitutional LawWhich of the following defendants could be punished under a New York state statute thatmakes the "incitement of others to perform any criminal act" a criminal act?

A. Joey, who tells a rally of several hundred college students that "the currentadministration can only be cleansed by guns and terror, and I will lead you in thiscleansing!"

B. Jack, who declares to several thousand protesters that "the war in Iraq isimmoral and in revolt we should march on the White House and burn it down!"

C. Jen, who tells a meeting of six of her followers that "I believe all 'Afro-Americans' arethe devil's children and should be systematically eradicated by whites!"

D. Andy, a member of the BBB, which has a manifesto stating "each member of the BBBwill take constant action to overturn the government of the United States and toassassinate all members of the government." Andy has been a BBB member for oneyear and has only been to one meeting.

Page 169: Law Decks Flash Cards - Constitutional Law - 2007-2008

Answer: BJack is the only person advocating imminent illegal action that is likely to produce suchaction. Prohibition of statements advocating the use of force or a violation of the law canonly occur if the advocating: 1) is directed to producing or indicting imminent lawlessaction; and 2) is likely to produce or incite such action. In Answer C, Jack is advocatingimmediate illegal action and his words are directed to producing such action. BecauseJack is speaking to thousands who are already protesting, it is likely that at least a few willact.

A is incorrect as these words simply relate her belief and are indefinite as to when thealleged cleansing will occur (lead by Joey). These words do not incite immediate action butspeak to action some time in the future.

C is incorrect. Jen is expressing her belief that African-Americans should be "eradicated."While these racist words do advocate force, there is no indication that this force should beapplied instantly. Additionally, there is no indication that this audience will follow thesewords and actually act.

D is incorrect because there is neither an advocacy for the use of force nor an indicationthat there will be immediate action. Additionally, it does not appear that Andy is aparticipating member. Simply being a member of an organization with a manifesto suchas this is not enough, you need continual action by the member.

©2007-2008 Law Decks

Page 170: Law Decks Flash Cards - Constitutional Law - 2007-2008

Constitutional LawSuppose that Arizona, faced with high unemployment and an ever increasing illegalimmigrant population, decides to pass a statute hiring 200,000 new state employees inan attempt to decrease both problems. The statute provides a hiring bias toward thosewho can show at least two years of experience in the customer service industry andhave been recently laid off. It additionally stipulates that aliens can only be hired as stateemployees if there are no available citizens with a service industry background. Anillegal alien rights group challenges the second tier hiring as dictated by the state statute.Which of the following will be the most helpful to their case?

A. The Privileges and Immunities Clause of the Fourteenth Amendment

B. The reserved powers of the state under the Tenth Amendment

C. The Equal Protection Clause of the Fourteenth Amendment

D. The Fourteenth Amendment Due Process Clause

Page 171: Law Decks Flash Cards - Constitutional Law - 2007-2008

Answer: CAlienage restriction results in a suspect classification under the Equal Protection Clauseand is subject to strict judicial scrutiny.

A is wrong. The Privileges and Immunities Clause of the Fourteenth Amendmentprotects the privileges and immunities of United States citizens and does not addressthe rights of aliens.

B is wrong because, even if applicable, the Tenth Amendment could only help the state;it does not carry any prohibitions.

D is wrong because the alienage restriction results in a suspect class; making an equalprotection analysis more applicable than a burden on a fundamental right analysis.

©2007-2008 Law Decks

Page 172: Law Decks Flash Cards - Constitutional Law - 2007-2008

Constitutional LawSuppose that Arizona, faced with high unemployment and an ever increasing illegalimmigrant population, decides to pass a statute hiring 200,000 new state employees inan attempt to decrease both problems. The statute provides a hiring bias toward thosewho can show at least two years of experience in the customer service industry andhave been recently laid off. It additionally stipulates that aliens can only be hired as stateemployees if there are no available citizens with a service industry background. TheArizona Supreme Court declares the statute unconstitutional as conflicting with theSupremacy Clause of the Constitution, as well as the Equal Protection Clause of theirstate constitution. Will the US Supreme Court review the decision of the ArizonaSupreme Court?

A. No. It does not meet the requirements of certiorari.

B. Yes. It meets the requirements of appeal.

C. Yes. The Supreme Court has original jurisdiction of all cases in which a state is aparty.

D. No, because of the "adequate and independent state ground" theory.

Page 173: Law Decks Flash Cards - Constitutional Law - 2007-2008

Answer: DThe Supreme Court would not review the state supreme court's decision based on theEqual Protection Clause violation of the Arizona Constitution. Therefore, even if thefederal issue was resolved, there would still exist an adequate and independent stateground upon which the decision would rest.

A is wrong because if not for the adequate and independent state ground, this type ofchallenge could be heard by the Court at its discretion.

B is wrong because this type of case would not meet the very narrow requirements ofappeal.

C is wrong because this is a question of appellate jurisdiction, not original jurisdiction.

©2007-2008 Law Decks

Page 174: Law Decks Flash Cards - Constitutional Law - 2007-2008

Constitutional LawFearing an influx of South African Water Frogs into the Great Lakes, and a subsequentdevastation of the interstate delivery of goods by ship, Congress passes a statute requiringthat all boats using the Great Lakes be coated with Frog Fatalis, a toxic chemical compound.This coating will be applied to all private and public ships. A Great Lakes naturalist grouphas also studied the problem and has found Pomum, a nontoxic citrus-based product, thatworks as well and is much cheaper. Those states located on the Great Lakes pass lawspermitting state-based boats to use Pomum as opposed to the toxic federally-mandatedproduct. Can they be prosecuted for violating federal law?

A. No. The Tenth Amendment prevents Congress interfering with government functionsdeemed integral.

B. No, because the municipalities are acting in accordance with the spirit and purpose ofthe federal law.

C. Yes, because Congress is in a better position to regulate the entire Great Lakes regionthan the neighboring states.

D. Yes, because Congress may adopt laws regulating navigable waters.

Page 175: Law Decks Flash Cards - Constitutional Law - 2007-2008

Answer: DThe states can be prosecuted because state laws that conflict with valid federal laws areinvalid under the Supremacy Clause. The federal law at issue is valid under a commercepower analysis or under the admiralty power of Congress. State actions directly inconflict with the federal law could, therefore, be stopped.

A is also incorrect. The Tenth Amendment is a poor analysis for the present question.The Supreme Court has held that states' Tenth Amendment rights are best protected bythe states' representation in Congress. Therefore, Congress is free to legislate on thematter at hand.

B is incorrect. The fact that Pomum may be as effective as the federally-mandatedcoating is negligible

C is incorrect as it is irrelevant. Here, the federal law is superior to the states' lawssimply because it is within Congress's power to pass legislation in this area. The fact thatCongress is in a better position than the states to adopt the legislation is meaningless.

©2007-2008 Law Decks

Page 176: Law Decks Flash Cards - Constitutional Law - 2007-2008

Constitutional LawA state actively participates in the sale of its natural resources on the open market whilesimultaneously selling these resources to state manufacturers at a price well below thatsupported by the market. An out-of-state company sues the state, alleging that thesebelow market price sales to resident manufacturers provide an unfair advantage.Assuming jurisdiction exists, the preferential pricing would be found:

A. Constitutional, as the state is selling natural resources at market prices outside thestate.

B. Constitutional, as the state is a market participant and free to charge out-of-stateresidents more.

C. Unconstitutional, as preferential pricing to in-state manufacturers violates thePrivileges and Immunities Clause.

D. Unconstitutional, as these preferential prices are discriminatory to nonresidents andin violation of the Commerce Clause.

Page 177: Law Decks Flash Cards - Constitutional Law - 2007-2008

Answer: BThe preferential pricing scheme will be found constitutional, on the basis that the stateis awarded the market participant exemption. While the Commerce Clause prohibitsdiscriminatory preferential pricing schemes; the fact that the state is a market participantin the sale of natural resources awards them additional protection. As this is the sale ofgoods, the state may charge whatever it wants to residents and nonresidents alike.

A is incorrect. The fact that the state is actively participating in the sale of naturalresources at market price outside of the state is irrelevant, because, absent the marketparticipant exception, the state's actions would be deemed unconstitutional in violationof the Commerce Clause.

C is incorrect. Although the Privileges and Immunities Clause does award an individualcitizen certain privileges and immunities, these privileges and immunities are notapplicable to a corporation such as the one bringing suit.

D is incorrect as set forth in the explanation for correct Answer B.

©2007-2008 Law Decks

Page 178: Law Decks Flash Cards - Constitutional Law - 2007-2008

Constitutional LawSuppose Congress attempts to prevent the loss of prior Supreme Court decisionsregarding individuals that have committed crimes by the current Supreme Court andhave decided to pass a law that restructures Supreme Court jurisdiction. The newSupreme Court jurisdiction will no longer include any case in which a state supremecourt has decided that a defendant's federal constitutional rights have been violated.Assume that the statute is held unconstitutional. Which is the most likely reason?

A. Congress only has power to limit jurisdiction in cases originating in federal courts.

B. Determination of the extent of constitutional rights is solely within the realm ofSupreme Court decisions.

C. Congress is not empowered to eliminate Supreme Court review of issues that arewithin the rights vested in federal courts.

D. To be effective here, the action taken by Congress would require a constitutionalamendment.

Page 179: Law Decks Flash Cards - Constitutional Law - 2007-2008

Answer: B

Article II, Section 2 provides that the Supreme Court shall have the appellate jurisdictionunder any regulations Congress shall make. Additionally, Congress may not limit allavenues for Supreme Court review of federal constitutional issues. In light of thisproposed statute being unconstitutional, none of the other opinions presented are viableanswers.

A is incorrect because congressional power is limited to regulating the Supreme Court'sappellate jurisdiction, not restricting or enlarging the Court's original jurisdiction.

C is incorrect as it is not the best argument. As set forth in the Answer A, Congress hasthe power to regulate the appellate jurisdiction of the Supreme Court. Additionally, theSupreme Court's primary role as an appellate court is determining constitutional issues.Therefore, Congress can eliminate cases involving constitutional issues from theSupreme Court's jurisdiction, as long as jurisdiction remains in some lower federalcourts.

D is incorrect. Although Congress is given the power to modify appellate jurisdictionthrough new laws, any modification of the Court's original jurisdiction would require aconstitutional amendment.

52007-2008 Law Decks

Page 180: Law Decks Flash Cards - Constitutional Law - 2007-2008

Constitutional LawTrying to raise revenue in a desperate economy, a city decides to sell advertising spaceon government buildings and vehicles, and passes an appropriate ordinance.Advertising is open to anyone as long as the ad has "nothing to do with politics orreligion," in an attempt to avoid legal headaches and allegations of entanglement ofreligion with state politics. Zeus and Apollo seek to rent space for theirinterdenominational store that caters to those belonging to all religions. Their ad reads:"Let Zeus and Apollo's Religious Store help you on your path to God!" The city refusestheir request. If Zeus and Apollo sue in district court alleging a First Amendmentviolation, they will most likely:

A. Win, because sides of civic buildings are now viewed as public forums.

B. Lose, because there is a viewpoint of neutral restriction that is reasonably relatedto a legitimate government purpose.

C. Win, because commercial speech restrictions must be narrowly tailored to serve asubstantional government interest directly advanced by the restriction.

D. Lose, because the city must avoid excessive entanglement with religion.

Page 181: Law Decks Flash Cards - Constitutional Law - 2007-2008

Answer: BThe restriction will be upheld because it is viewpoint neutral and reasonably related to alegitimate government purpose. The city rule distinguishes between broad categories ofspeech while refraining from distinguishing based on content within a category. This isreasonably related to the legitimate government interest of avoiding controversy.

A is incorrect because the Supreme Court has held that allowing advertising ongovernment-owned property is deemed a commercial forum, not a public forum.

C is incorrect because it merely states the general rule for regulation and commercialspeech. As mentioned, here we are dealing with a proprietary venture by the city andthe advertising is not deemed a public forum.

D is incorrect because nothing in the city policy causes excessive entanglement withreligion and actually seeks to avoid excessive entanglement.

©2007-2008 Law Decks

Page 182: Law Decks Flash Cards - Constitutional Law - 2007-2008

Constitutional LawThe media breaks several stories chronicling the President's violation of campaignfunding requirements, prompting a grand jury investigation and a Senate subcommitteeinvestigation. The subcommittee has subpoenaed documents and records fromnumerous executive branch officers, but the President flexes his executive privilegerights and orders all executive officials to refuse to turn over materials. Which of thefollowing statements is most accurate?

A. The subpoena violates separation of powers.

B. The President has absolute executive privilege in all manners except impeachment.

C. The presidential papers are assumed privileged, but this privilege will be overriddenupon a showing of specific need for evidence.

D. The President's executive privilege only applies to proceedings by Congress, not thecourts.

Page 183: Law Decks Flash Cards - Constitutional Law - 2007-2008

Answer: CUnder executive privilege, presidential documents and conversations are presumptivelyprivileged until a showing of need for the evidence in a criminal case is established.While not specifically addressed by the Supreme Court, it is likely that this privilege willalso fair in cases of pending legislative proceedings that require the privileged materials.

A is too broad. Precedent states that a subpoena to the President in a criminal case isnot a separation of powers principle violation. Therefore, allowing this power ofCongress to investigate a Senate subcommittee is likely permissible.

B is also too broad. As set forth in the correct answer, presidential privilege is notabsolute.

D is incorrect as executive privilege is overridden only upon a specific showing of needfor specific information, regardless of whether it is a congressional need or a need of theSupreme Court.

©2007-2008 Law Decks

Page 184: Law Decks Flash Cards - Constitutional Law - 2007-2008

Constitutional LawSuppose Denver passes a bill requiring all bodyguards to be licensed. Licenses requirea bond to be posted, a background check, and a bodyguarding test. Upon passingthrough these requirements a guard must swear a loyalty oath stating that he will be aloyal US citizen and a membership oath stating that he will not join any unlawful groups.Todd applies for a license to guard a persecuted Indian Muslim writer visiting fromEngland. He passes all the requirements, but for personal reasons will not swear to theoaths and is denied a license. He files suit disputing the licensing process. The court willfind that:

A. Both the loyalty oath and the membership oath are constitutional.

B. The loyalty oath is constitutional, but the membership oath is unconstitutional.

C. The membership oath is constitutional, but the loyalty oath is unconstitutional.

D. Neither the loyalty oath, nor the membership oath are constitutional.

Page 185: Law Decks Flash Cards - Constitutional Law - 2007-2008

Answer: BThe membership oath will be considered unconstitutional under the First Amendmentrights of association and speech. For the state to limit these rights, they must show acompelling state interest, unrelated to the suppression of ideas, which is not included inthe facts. As this is a private job, not a public employment, the state must be careful notto infringe unnecessarily upon the rights of an individual. The loyalty oath has been heldconstitutional on numerous occasions as the state clearly has a compelling interest in itscitizens remaining loyal. The membership oath is overly broad and would be struckdown as there is no compelling interest in the state for preventing people fromassociating freely. Therefore, B is correct while Answers A, C and D are incorrect.

©2007-2008 Law Decks

Page 186: Law Decks Flash Cards - Constitutional Law - 2007-2008

Constitutional LawSuppose District 1, which includes the wealthiest town in Utah, is funded at close to twice asmuch as District 10, which includes one of the poorest Utah towns. The reasoning behind this isthat funding is related to taxes collected, and the wealthier residents if District 1 pay more taxesthan those in District 10. The system has no racial or discriminatory motives. Regardless, thishas led to numerous complaints from state residents. In response, Utah decides to fund schoolson an equal per capita basis. Chelsea, a resident of District 1, brings suit in state court lookingto have the per capita statute declared unconstitutional. The highest court finds that thelegislation is permissible, citing an equal protection type clause in Utah's constitution. Followingthis ruling, James, from the neighboring state of Wyoming, files suit seeking to get Wyomingschools put on a per capita funding regiment. James's case reached the United States SupremeCourt, who rules against per capita funding on the basis that the Fourteenth Amendment doesnot compel equal funding absent a showing of racial discrimination. James learns of theSupreme Court decision and petitions for a writ of certiorari having the Utah decision overturned.James will:

A. Prevail under the Supremacy Clause which renders the state decision invalid.

B. Prevail because the issue is res judicata.

C. Fail because the decision in Utah is based on state law grounds.

D. Fail because James lacks standing.

Page 187: Law Decks Flash Cards - Constitutional Law - 2007-2008

Answer: CThe Supreme Court will only hear a state case if it turned on federal grounds. If thereare adequate and independent state grounds, it will refuse to take the case. Here, thedetermination in Utah was based on the state constitution. Since the state court decisionwas based on state interpretation of a state law and not an interpretation of a federallaw: the Supreme Court will determine that there is no jurisdiction.

A is incorrect as the Supremacy Clause only applies to a situation in which state lawsconflict with federal laws, or if Congress has preempted the field. Here, there is neithera conflict nor a preemption. Therefore, the state is free to grant its citizens a broaderinterpretation of the Equal Protection Clause as compared to the interpretation that thefederal court has decided when reviewing the state constitution.

B is incorrect as res judicata is not a constitutional issue.

D is also incorrect. James clearly has a concrete stake in the outcome of a controversybased upon an injury in fact caused by government action that can be remedied by adecision in his favor. Therefore, James clearly has standing.

©2007-2008 Law Decks

Page 188: Law Decks Flash Cards - Constitutional Law - 2007-2008

Constitutional LawSuppose the state of North Carolina has a statute stating that land may not be ownedby an alien regardless of how the alien comes to possess the land (i.e.— inheritance, gift,purchase). Farhana, a Saudi US resident, looks to buy herself a beach house from Karain the Outer Banks. If the seller, Kara, brings suit to test the statute, the best argumentis that the statute:

A. Denies the equal protection of the law to aliens.

B. Unconstitutionally impairs the right to contract.

C. Is a direct restraint on the alienation of real property.

D. Deprives the seller of a property right without due process of law.

Page 189: Law Decks Flash Cards - Constitutional Law - 2007-2008

Answer: AKara is free to assert the violation of the equal protection rights of another. Here, Farhana, thelegal alien, and Kara, the seller, both have a concrete stake in the outcome of the controversy.The governmental action at issue also impairs Kara's rights, as she is prohibited from contractingto sell land to Farhana, a legal alien. Therefore, she has standing. Additionally, the seller's rightsare adversely affected due to the prohibition of sale of property to aliens, indirectly violating hisequal protection rights.

B is incorrect. The Contract Clause prohibits states from passing any law impairing the obligationof contracts. Farhana and Kara entered into the contract after the statute was in effect. As thelaw was in effect before the entry into the contract, and the law was not retroactive, the statutehere does not violate the Contract Clause.

C is also incorrect. There is insufficient restraint to be deemed a constitutional violation. Here,the restraint is simply against sales to aliens. It is not a complete restraint as a seller is free tosell to any other party.

D is incorrect. There are no facts indicating a due process violation. Due process taking involves

individualized takings, such as a person's house in the path of a highway being built. Here, weare faced with a general taking, not a specific taking applied to Kara only. Additionally,substantive due process is not applicable as we are dealing with a non-fundamental rightrequiring a rational basis test. This test places a high hurdle for Kara to overcome as she isforced to assume the burden of proving the law without merit.

©2007-2008 Law Decks

Page 190: Law Decks Flash Cards - Constitutional Law - 2007-2008

Constitutional LawSuppose California enacts the "Resident Alien Act" to encourage long-time resident aliensto become American citizens. The act states that a variety of state and municipal jobs willbe denied to those who have been resident aliens for more than five years. Furthermore,resident aliens in the state have to apply for citizenship within a year of the law passing.Mestrovic, a 50 year old legal alien, has been a resident for 15 years. He did not file forcitizenship within the one-year grace period, as he hoped to return to Croatia. Looking for anew job, Mestrovic applied for a computer job at the local police department. Althoughqualified for the job, the police department, under the new law, is forced to withdraw hisapplication. Mestrovic speaks to his attorney who should advise him that the state law is:

A. Unconstitutional. There is not a compelling reason in applying the law advanced by thestate.

B. Constitutional. The Privileges and Immunities Clause does not apply to aliens in a state.

C. Constitutional. The state law does not discriminate by classifying resident aliens by raceor ethnicity, and the police department is an integral governmental organization.

D. Unconstitutional because the law does not apply to aliens.

Page 191: Law Decks Flash Cards - Constitutional Law - 2007-2008

Answer: AThe law is unconstitutional as applied. An equal protection issue is involved. The EqualProtection Clause states that state classifications based on alienage are subject to strictscrutiny and so must serve a compelling interest to be constitutional. Here, there is nocompelling purpose present.

B is incorrect. While it is true that aliens are not entitled to the privileges and immunitiesof state citizenship, the law at issue is in violation of the Equal Protection Clause whichmay be applied to aliens.

C is incorrect based on a misapplication of the exception allowing discrimination whendealing with "self government" processes. While there is an exception from the strictscrutiny standard for a state or local government discriminating against aliens whenhiring for certain "self government" applications; here, the job at issue is simply acomputer position unrelated to the self-government process. Furthermore, the statutehere applies to all positions, not just to jobs involving only the self-government process.

D is incorrect. Regardless of the application to all aliens equally, the discrimination wouldstill remain unconstitutional.

©2007-2008 Law Decks

Page 192: Law Decks Flash Cards - Constitutional Law - 2007-2008

Constitutional LawMoneybox Co. sells lottery supplies to the state of Connecticut. As there are oftenmillions at stake, the state requires Moneybox Co. to forward the resumes of all potentialemployees so that the state may investigate their background. Hudson is an existingemployee at will with a mediocre performance history. Hudson's boss instantly fires himupon discovering that Hudson accidentally numbered all of the balls "4." There is a statelaw in effect that states no state employee may be fired without a hearing. Hudson suesin the federal district court stating that he was constitutionally entitled to a hearing todetermine whether his firing was appropriate. If the court rules correctly, it will probablyfind his termination:

A. Constitutional, because no hearing was required since his boss supervised hismisconduct.

B. Constitutional, because Moneybox Co. is free to fire employees as it pleases.

C. Unconstitutional, because it violates Hudson's right to procedural due process.

D. Unconstitutional, because of the state's regulation of the hiring process.

Page 193: Law Decks Flash Cards - Constitutional Law - 2007-2008

Answer: B

Moneybox Co. was free to fire Hudson as he was an employee at will, and a privateemployer usually can fire an employee for any reason (or no reason at all), absent acontract providing otherwise. Here, there was clearly no contract prohibiting the firing.

A is incorrect. Whether Hudson's boss supervised the mistake or not has no bearing onthe necessity of a public hearing. If this was a state employee, a hearing would berequired regardless of the supervisor's presence or knowledge of mistakes, because apublic employee who is subject to "cause" has a property interest in his job; thereby,requiring a hearing prior to its taking.

Answers C and D are related and both incorrect. Most constitutional guarantees prohibitstate action and do not impose duties on private parties. State action can only be foundif the acts of a private party serve exclusive public functions, or if there is significant stateinvolvement in the activities. Here, there is no state action since running a lottery is notan exclusive public function. Selling lottery equipment is also not significant stateinvolvement.

©2007-2008 Law Decks

Page 194: Law Decks Flash Cards - Constitutional Law - 2007-2008

Constitutional LawA state has enacted a statute requiring parents to vaccinate their children against the flueach year or be subjected to a $1,000 fine or 1 year in jail. The Smiths object to thisstatute on the grounds that it violates their right to free exercise of religious beliefs underthe First Amendment, and have filed suit in federal court. If the state prosecutes theSmiths criminally, the state court may constitutionally inquire whether:

A. The Smiths honestly and sincerely believe in the tenets of their religion.

B. The religion at issue is an established one.

C. The tenets of the family's religion are true.

D. The Smiths believe the tenets of their religion are derived from the teachings of awise prophet or are internally derived.

Page 195: Law Decks Flash Cards - Constitutional Law - 2007-2008

Answer: AThe Free Exercise Clause, applicable to all states through the Fourteenth Amendment,provides protection from punishment due to a person's religious beliefs. Therefore, acourt potentially awarding protection under the Fourteenth Amendment may inquire asto the sincerity of the person's religious beliefs in order to determine if they are eligiblefor protection.

B is incorrect. The Free Exercise Clause protects all sincerely held religious beliefs,regardless of whether they are well established or not.

C is incorrect. The Free Exercise Clause prohibits a court from passing judgment on thetruth or validity of a religious belief, but allows a court to inquire whether the belief is heldby a person sincerely.

D is incorrect. Religious beliefs may be any belief sincerely held by an individual. Thesebeliefs do not need to be theistic in nature, but rather may be internally formed, from thenatural world, etc...

C.2007-2008 Law Decks

Page 196: Law Decks Flash Cards - Constitutional Law - 2007-2008

Constitutional LawSuppose New York City requires all foreign currency exchange kiosks at the city's harborfront to have licenses issued by the city, and they are only yearly renewable uponreviews of customer satisfaction surveys. Mike has a New York license, as well as oneissued by the federal government for all vendors operating at international port locations,which New York harbor is. Mike's New York license comes up for review and he isdenied renewal due to negative reviews. He is given a hearing to review the statementsbut fails to attend. Mike continues to operate and is approached by a city licensingofficer, who asks to see his license. Mike provides his federal license, and is cited fornot having a valid state license. Mike protests. What is his best defense?

A. The city has denied him his license without due process of law.

B. New York City's licensing scheme was arbitrary and capricious.

C. New York City's licensing interferes with interstate and foreign commerce and is,

therefore, unlawful.

D. New York City's license requirement is superseded by the federal license Mike holds.

Page 197: Law Decks Flash Cards - Constitutional Law - 2007-2008

Answer: DUnder a Supremacy Clause application, federal licensing law is the supreme law of theland. In light of the licensing law conflict between federal and state, the federal licensewill supersede the New York City license law, as Congress has clearly acted under itspower to regulate interstate and foreign commerce in establishing a federal licensing law.

A is incorrect as there is no due process violation here, as all valid due processrequirements appear to have been met. Assuming a vested property right under theFourteenth Amendment Due Process Clause, there is still no indication that either theNew York City commission violated this right or that there were due process inadequacies.

B is incorrect as no facts indicate an arbitrary licensing system by New York City. If it wasarbitrary, there would be a valid argument revolving around the fact that a substantive dueprocess violation occurred, as all laws must at a minimum be rational. Additionally, the"arbitrary and capricious" language is completely inapplicable, as this standard is used bycourts reviewing "off the record" determinations of fact by administrative agencies.

C is incorrect for failure to have any facts that appear to interfere with foreign or interstatecommerce. There is no favoring of local economic interests over out-of-state or foreigninterests and, therefore, no unlawful discrimination. Additionally, the system appears toplace little if any burden on interstate or foreign commerce.

©2007-2008 Law Decks

Page 198: Law Decks Flash Cards - Constitutional Law - 2007-2008

Constitutional LawSuppose California is faced with a growing public drunkenness problem among itscitizens and decides to establish a public rehab program. To fund this program theydecide to tax all out-of-state hard alcohol and beer advertisements in locally circulatedpublications at 25%. Wine producers are exempt from this tax. A group of advertisingexecutives at locally-circulated papers and out-of-state brewers elect to challenge theconstitutionality of the tax. If the tax is declared unconstitutional, it will most likely bebecause:

A. The tax burdens interstate commerce as it exempts advertisements for local brewerswhile the ads of out-of-state brewers and distillers are subject to the tax.

B. The tax infringes on the First and Fourteenth Amendment rights of freedom of thepress.

C. The tax is unconstitutional because it is not properly apportioned.

D. The tax violates the Equal Protection Clause of the Fourteenth Amendment becauseit does not treat all alcoholic products equally.

Page 199: Law Decks Flash Cards - Constitutional Law - 2007-2008

Answer: BThe tax unconstitutionally burdens all freedom of the press as a tax applicable only tothe press industry, or an industry based on the content of a publication, is impermissibleabsent a compelling justification. Revenue generation may not be deemed a compellinginterest.

A is incorrect because there is no burden on interstate commerce. Since both the localand out-of-state businesses are subject to the tax equally, there is no protecting of localbusinesses as opposed to out-of-state companies. Exempting wine producers ispermissible as they are viewed as a separate business endeavor and may be treateddifferently.

C is incorrect. Sales tax on an interstate sale does not result in an apportionmentproblem as no other state has sufficient nexus to the transaction allowing the taxationelsewhere.

D is incorrect as there is no equal protection violation here. There is no suspect class orfundamental right at issue, nor is a quasi-suspect class involved. Therefore, the tax isconstitutionally permissible as long as it is rationally related to a legitimate governmentpurpose. Here, the tax is clearly related to raising revenue.

©2007-2008 Law Decks

Page 200: Law Decks Flash Cards - Constitutional Law - 2007-2008

Constitutional LawSuppose that the state of Connecticut looks to cut back unnecessary spending in stateelections, and requires all those running for elected positions to obtain signatures from10% of all eligible voters in the state before being added to the ballot. Benson Burns isan independent running a low budget campaign with a small group of volunteers whowere only able to gain 1% of the required signatures. Burns is denied admission to theballot and sues alleging that the law is unconstitutional.

A. Burns will have to show that the petition requirement is not rationally related to alegitimate state interest.

B. Burns will have to show that the petition requirement is not achieving a compellingstate interest.

C. Burns will have to show that the petition requirement is not narrowly tailored onachieving a compelling state interest.

D. Burns will have to show that the petition requirement is rationally related to alegitimate state interest.

Page 201: Law Decks Flash Cards - Constitutional Law - 2007-2008

Answer: CA compelling state interest test will be applied requiring a showing of the narrowlytailored application of the law. Election process regulation is closely reviewed by theSupreme Court. If they find a severe First Amendment restriction, a narrowly tailoredreason to achieve a compelling interest must be shown. In this case, the 10% signaturerequirement will be viewed as a severe First Amendment restriction.Tip: Be careful of answers where the signature requirement is low, like 1%, as this maynot be held as a severe restriction.

Answers A and D are incorrect as a rational basis test is incorrect in light of the correctanswer.

C is incorrect as it shifts the burden the of proof to the wrong party. In a compellinginterest situation the burden would be on the state, not Burns.

S2007-2008 Law Decks

Page 202: Law Decks Flash Cards - Constitutional Law - 2007-2008

Constitutional LawBARPREP, a Nevada corporation with its business headquarters in New York City, NY,offers a law bar prep course for recent graduates in at least one location in each of the50 states. Preparation materials are printed by Lynnell's Printing in New Jersey, whothen ships the materials to each office location in the various states. BARPREP'sinstructors are independent contractors paid annually and are responsible for their ownpreparation of the allotted subject matter. Some professors lecture only in one statewhile others, like Sebastian who is a resident of California, make their living traveling tomany states giving bar review lectures. Which of the following taxes is most likely valid?

A. A California income tax on all of Sebastian's income.

B. A Nevada income tax on all of Sebastian's income.

C. A New Jersey use tax on instructional materials shipped to and sold in California.

D. A Nevada ad valorem property tax on each piece of instructional material received inthat state.

Page 203: Law Decks Flash Cards - Constitutional Law - 2007-2008

Answer: DAn ad valorem tax based on a percentage of the assessed value of the property in question ismost likely to be held valid. To prevent a Commerce Clause violation where goods are incorrectlytaxed while in transit, this tax may only be applied when the goods arrive at their final destinationin California.

Answers A and B are incorrect as they fail to meet the three-part test for evaluating if anondiscriminatory state tax affecting interstate commerce is valid. The factors are: 1) asubstantial nexus between the activity or property taxed, and the taxing state; 2) a fairapportionment based on the extent of the taxable property or activity in the state; and 3) a fairrelationship between the tax and the services or benefits provided by the state. In Answer A,Sebastian is a resident of California and lectures in that state. Thus, a substantional nexus existsbetween income-producing activity and the taxing state. However, Sebastian also producesincome in numerous other states. Therefore, a tax in California of his entire income in not acorrectly apportioned tax. The tax in Answer B is also poorly apportioned as it taxes all ofSebastian's income and fails to show a substantial nexus between Nevada and Sebastian'sincome-producing activity. From the facts, there is no info noting that Sebastian has any incomegenerated from lecturing in Nevada.

C is incorrect as it is a misapplication of a use tax. A use tax is applied by the state in which thebuyer resides on goods purchased out of state. In the present case, the publisher and printerare both located in New Jersey, and have no valid right to impose a tax on goods used in anotherstate. Therefore, Answer C is incorrect.

@2007-2008 Law Decks

Page 204: Law Decks Flash Cards - Constitutional Law - 2007-2008

Constitutional LawSuppose Minneapolis seeks to build a homeless shelter and decides to solicit bids from privatecompanies interested in building and subsequently operating the center. The Church of Bill, whoruns several shelters in the area, submits the lowest bid. These shelters serve food, counsel thehomeless and offer religious materials relating to Bill, their god. They do not hold mass or preachto the homeless. The second lowest bidder, Turn Around, is a nonprofit secular organization.The difference in their bids was due to a tax imposed on secular institutions running a shelter,which Turn Around must pay. Turn Around challenges the tax, seeking to gain the same savingsas the religious group. Which of the following statement is most correct?

A. Imposing the tax on the Church of Bill's bid improperly inhibits their free exercise of religion.

B. Maintaining the tax exemption for the Church of Bill's bid is a valid way to avoid excessivegovernment entanglement with religion.

C. Permitting a tax exemption for churches, or Church of Bill, and not for other nonprofitinstitutions, or Turn Around, results in an advancing of religion which is unconstitutional.

D. The tax exempting the church-run shelters is an Equal Protection Clause of the FourteenthAmendment violation, as there is not a state compelling interest justifying the taxation of someorganizations and the exemption of others.

Page 205: Law Decks Flash Cards - Constitutional Law - 2007-2008

Answer: CThere is an Establishment Clause violation in favoring a religious institution over a secularnonprofit organization. In order to prevent an Establishment Clause violation, a law favoringa religion institution must have: 1) a secular purpose; 2) a primary effect that neitheradvances nor inhibits religion; and 3) must not produce excessive governmententanglement with religion. Here, providing the tax exemption only to the Church of Billadvances religion by giving them an advantage over Turn Around.

A is incorrect. The broad Establishment Clause exemption from property taxes for religious,educational, or charitable organizations is not applicable, here, since the exemption isspecifically for religious groups opening a shelter, which unlawfully discriminates againstthose shelters operated by secular nonprofit organizations. If the state had provided a taxbreak to all charitable organizations operating homeless shelters then the law would beokay.

B is also wrong. Avoiding government entanglement is clearly a valid point, but givingreligious shelters exemptions while not exempting secular nonprofit-run shelters results inan Establishment Clause test violation.

D is incorrect. A compelling interest is not necessary of a state when they do notdiscriminate against a suspect clause or burden a fundamental right. The tax break herewould be tested with a rational relationship to a legitimate government interest test.

©2007-2008 Law Decks

Page 206: Law Decks Flash Cards - Constitutional Law - 2007-2008

Constitutional LawThe state of North Carolina has a statute stating that land may not be owned by an alienregardless of how the alien comes to possess the land (i.e.—inheritance, gift, purchase).Farhana, a Saudi US resident, looks to buy herself a beach house from Kara in theOuter Banks. If Farhana brings the suit to test the statute, she will probably:

A. Win, because the statute serves to limit Congress's power to legislate with respect toaliens and foreign affairs.

B. Win, because the statute is a Privileges and Immunities Clause violation underArticle IV of the Constitution.

C. Win, because the statute denies her equal protection of the laws.

D. Lose, because she is an alien.

Page 207: Law Decks Flash Cards - Constitutional Law - 2007-2008

Answer: C

Farhana, as a legal alien, falls within the protection of the Equal Protection Clause andthe Fourteenth Amendment which prevents discrimination among persons, of whichaliens are included. The statute is based solely on the fact that Farhana is an alien and,therefore, deemed the equal protection violation.

A is incorrect. No congressional power regarding aliens or foreign affairs is limited asnone of Congress's powers over airlines, like naturalization or deportation, are infringedupon. Furthermore, there are no ties in the statute between its acts and foreign affairs.

B is wrong. The Article IV Privileges and Immunities Clause does not protect the rightsof aliens as it only concerns itself with citizens of the states. Aliens or corporations arenot included within the Privileges and Immunities Clause.

D incorrectly states that Farhana's legal alien status automatically excludes her from anyconstitutional protections. This is clearly wrong as correct Answer C sets forth the factthat she is entitled to Equal Protection Clause benefits.

©2007-2008 Law Decks

Page 208: Law Decks Flash Cards - Constitutional Law - 2007-2008

Constitutional LawSuppose Illinois learns that tractor trailers operating on their roads and weighing over 30,000lbs cause excessive road problems: rutting, potholes, frequent jackknifing accidents. Thestate decides to pass a $1,000 registration fee for any vehicle that uses Illinois roads and a$110 per mile fee for vehicles weighing over 30,000 lbs. Joe owns a trucking company in IowaCity. All of his trucks are over 30,000 lbs and travel from Iowa through Illinois in 50% of theirtravels. The new fee and mileage requirements would result in Joe handling over about 40%of his annual income to Illinois. Rather than pay, Joe elects to sue, seeking a declaration bythe federal district court that the statute is unconstitutional. At trial the state of Illinois producesdata from traffic engineers noting the accident and road wear statistics. If the court finds thetax unconstitutional, it will most likely be because:

A. Through taxation, Illinois seeks to regulate matters that the state is not free to do otherwise—namely, to regulate interstate commerce.

B. The state's interest in highway preservation and decreasing accidents are outweighed bythe interference with interstate transportation of goods.

C. The tax violates Joe's equal protection rights.

D. A use tax on companies engaged in interstate commerce violates the Commerce Clause.

Page 209: Law Decks Flash Cards - Constitutional Law - 2007-2008

Answer: BHere, Illinois is enacting a law that does not discriminate against interstate commerce, as thestate has shown that 30,000 lb trucks do cause more accidents and increase roadway repair.However, Joe has additionally shown that the state law makes it prohibitively expensive tooperate his trucks in Illinois. In light of this, the court is free to find that the added expense oninterstate carriers is not outweighed by the benefits to the citizens of Illinois.

A is incorrect as it assumes that the states cannot regulate interstate commerce in general ortrucks in particular. States can, in fact, regulate safety aspects of interstate commerce as longas that regulation does not discriminate against, or unduly burden, interstate commerce. Bythis logic, if the state has the power to regulate: they have the power to tax.

C is incorrect because the Equal Protection Clause is not violated here. The Equal ProtectionClause states that a law regulating economic interests, absent a suspect class or fundamentalright violation, will be upheld as long as the classification is rationally-related to a legitimategovernmental interest. Preventing accidents and saving money on road repair are legitimategovernment interests: the law here is valid.

D is also incorrect. States can impose use taxes as long as: the taxed activity has asubstantial nexus to the taxing state when taxing interstate matters; the tax is fairlyapportioned; the tax does not discriminate against interstate commerce; and the tax is fairlyrelated to the services provided by the state. The tax here meets all of these conditions.

@2007-2008 Law Decks

Page 210: Law Decks Flash Cards - Constitutional Law - 2007-2008

Constitutional LawSuppose that under contract with the federal government, Wodowski owns a food car andtourist trap type store located in Big Sur National Park which is located in the bounds ofCalifornia. Wodowski pays no California property taxes as the land she works on is ownedby the federal government. California passes a new land users tax on companies that arelocated in California but pay no taxes. Wodowski is expected to pay. She protests and takesthe case to federal court. What is Wodowski's best argument?

A. Wodowski has a privileges and immunities of national citizenship violation as the statetax impairs her fundamental right to conduct business on federal lands.

B. The tax is unconstitutional because it interferes with interstate commerce (tourism) andis in violation of the federal commerce power.

C. Wodowski has an equal protection violation as she operates on federal lands and thoseoperating on federal lands in other states are not taxed.

D. The tax is unconstitutional as the property power allows the federal government toregulate federal lands.

Page 211: Law Decks Flash Cards - Constitutional Law - 2007-2008

Answer: DThe best argument is that the tax interferes with Congress's power to regulate federal lands underArticle IV, Section 3 of the Constitution. The power to regulate federal lands, combined with theSupremacy Clause, prevents any state taxation of companies on federal land absent congressionalconsent.

A is not as good an argument. The Privileges and Immunities Clause only protects a narrow set ofprivileges, such as the right to enter onto federal lands. It does not protect the right to conductbusiness on federal lands. Here, Wodowski's rights fall outside the scope and are not protectedunder the Privileges and Immunities Clause.

B is incorrect. The commerce power does not preempt the tax. States can adopt taxes that on theirface interfere with interstate commerce as long as: 1) the tax does not discriminate againstinterstate commerce; 2) there is a substantial nexus between the state and the taxpayer; 3) the taxis fairly apportioned; and 4) there is a fair relationship between the tax and the benefits providedby the state. Here, all four parts are met so the tax is permissible and does not violate theCommerce Clause.

C is also incorrect. The facts do not indicate an equal protection violation as the Equal ProtectionClause merely prohibits the states from treating similar people in a dissimilar manner absent a validreason. The present law does not involve a suspect or quasi-suspect class or a fundamental right.Therefore, the law will be upheld upon a showing of a rational basis. Here, the acceptable rationalbasis (assuming that the law discriminates) is a rational relationship to raising tax-based revenue.

©2007-2008 Law Decks

Page 212: Law Decks Flash Cards - Constitutional Law - 2007-2008

Constitutional LawSuppose that the FCC passes legislation relating to vehicle radar detectors, attemptingto limit their operation frequencies so as not to interfere with police communication andaircraft autopilot landings. Freddie owns a detector that is FCC approved, and he usesit on his daily trip from his home state of Alabama to see his aging mother in Florida.Freddie is pulled over and ticketed for speeding by a police officer that gauges his speedby following him from afar. The officer sees the detector and issues him another ticketfor violating a Florida state statute making possession of a radar detector illegal.Freddie pleads guilty for speeding, but goes to federal court to challenge the detectorticket. The court is likely to rule:

A. Freddie wins because the regulation of radio transmissions is within the FCC'spowers, and preempts any state laws that attempt to regulate such devices as radardetectors.

B. Freddie wins, because the Florida statute burdens interstate commerce.

C. Freddie loses, because the Florida statute does not conflict with the FCC regulations.

D. Freddie loses, because the state has a legitimate interest in regulating the use ofradar detectors in order to promote safe driving.

Page 213: Law Decks Flash Cards - Constitutional Law - 2007-2008

Answer: DStates may regulate local aspects of interstate commerce as long as that regulation does notconflict with, or become preempted by, a federal regulation. Additionally, the regulationcannot discriminate against out-of-state competition in order to benefit local economicinterests, and the burden on interstate commerce may not outweigh the local benefits of theregulation. Here, the regulation is not discriminatory against out-of-state products as it bansall detectors regardless of where they were made. The second part of the test, a balancingtest, is also met: the ban clearly promotes the state's legitimate interest in highway safetysince banning detectors makes it harder for drivers to avoid getting caught for speeding. Thelocal safety benefits arguably outweigh the burden on interstate commerce.

A is incorrect. A field is considered preempted only when the federal statute is socomprehensive that it is implied that Congress intended to occupy the entire field. Here, weonly have a federal regulation concerning frequencies and their effects on aircraft and TVwith no mention of speeding.

B is too broad. Following the evaluation of the law in light of a balancing test, some laws thatburden interstate commerce may be found permissible.

C is incorrect for failing to adequately address the applicable issue. Regardless of the factthat the local statute does not conflict with federal regulations, we must apply the two-parttest outlined in the correct answer to determine if the statute is permissible.

©2007-2008 Law Decks

Page 214: Law Decks Flash Cards - Constitutional Law - 2007-2008

Constitutional LawSuppose Georgia has a statute stating that all civil servants who have worked for the state for over ayear may only be dismissed for "misconduct" in accordance with the state issued guidelines for theparticular agency. The guidelines for roadway custodial civil servants require that a written notice ofany grounds for dismissal be provided to an employee prior to their dismissal. Additionally, theemployee must be granted a post-dismissal hearing within three months after the dismissal takeseffect. A dismissed employee may introduce evidence at his hearing, and will be reinstated and backpaid if the hearing board decides that the city has not shown, by a preponderance of the evidence, thedismissal to be justified. Kyle, a three-year veteran of the highway service, is notified of his dismissalby a registered letter alleging that he has taken numerous bribes from non-city employees andinformed of his right to a hearing. Kyle instantly requests a hearing. After hearing his testimony theboard agrees with the state and dismisses him based upon a preponderance of the evidence. If Kylefiles suit in federal court, challenging his dismissal on constitutional grounds, will he likely prevail?

A. Yes, because Kyle had a right to a pre-termination hearing at which he could presentwitnesses to support his side of the story.

B. Yes, because Kyle had a right to have an opportunity to respond to the charges prior to hisdismissal.

C. No. The state may establish the required procedures for terminating an interest it created bystatute.

D. No. The procedures taken for termination of Kyle's job satisfied due process requirements.

Page 215: Law Decks Flash Cards - Constitutional Law - 2007-2008

Answer: BKyle will prevail as he did not receive his due process rights in the dismissal hearing.Under due process of the Fourteenth Amendment a public employee who is subject toremoval only for "cause" has a property interest in continued employment. The Courtstates that these employees must be given notice of the charges as well as a chance torespond to the charges before their termination; and if they are terminated, allowed anevidentiary hearing that could potentially allow their reinstatement. In the present case,there was no pre-termination hearing: therefore, Kyle's termination did not satisfy dueprocess.

A is incorrect. A full formal hearing before termination is not necessary so long as the oralor written notice of charges is accompanied by an explanation of the evidence, and givesan opportunity to respond.

C is wrong. State workers' property interests are determined by constitutional standards.The state cannot define what procedures may be used to terminate the property interest.

D is also incorrect as there is a due process violation here (as outlined above). The Courthas held that an employee can only be suspended from his job without a prior hearing, ifthere is a significant reason for removing them and a post termination hearing is later held.Here, there is no significant reason to remove Kyle's pre-termination hearing right.

©2007-2008 Law Decks

Page 216: Law Decks Flash Cards - Constitutional Law - 2007-2008

Constitutional LawState Ranch Insurance Co. is a state-run insurance company that uses a fixed feeschedule for calculating premiums. Angry City has a premium 30% higher than itsneighbors using this predetermined fee schedule. A group of Angry City residents bringssuit in state court requiring State Ranch Insurance Co. to make the premiums equal foreveryone. Which of the following would be the most likely result?

A. The residents will prevail, unless State Ranch shows a compelling reason for thediscrimination.

B. State Ranch will prevail, unless the residents show that there is no rational basis forhigher premiums.

C. The suit will be dismissed, because State Ranch is organized as a private businessand, thus, acting as a market participant.

D. The suit will be dismissed. State Ranch is an instrument of the state, and, thus,immune under the Eleventh Amendment from suits by citizens of the state.

Page 217: Law Decks Flash Cards - Constitutional Law - 2007-2008

Answer: BHere, we are dealing with economic legislation that will be upheld as long as there is arational basis for the legislation. The burden of proving this rational basis falls on theresidents filing suit.

A is incorrect for using the wrong standard. The compelling interest standard is not usedto judge economic legislation.

C is incorrect. The market participant rule is only applicable to the Commerce Clauseand acts which burden interstate commerce. Here, as this involves intrastate actionsonly, we do not have an interstate commerce question. The insurance company will,therefore, be held to the same equal protection standards as any other arm of the state.

D is not applicable as the Eleventh Amendment refers to suits in federal court and thefacts, here, are state court based.

©2007-2008 Law Decks

Page 218: Law Decks Flash Cards - Constitutional Law - 2007-2008

Constitutional LawState Ranch Insurance Co. is a state-run insurance company that uses a fixed fee schedulefor calculating premiums. Angry City has a premium 30% higher than its neighbors usingthis predetermined fee schedule. A group of Angry City residents brings suit in state courtrequiring State Ranch Insurance Co. to make the premiums equal for everyone. Angry Cityis 50% Latin-American, which is 20% higher than the rest of the state. If the Latin-Americancitizens of Angry City sue alleging that State Ranch's rate structure violates the EqualProtection. Clause, which of the following would be the most likely result?

A. The citizens will prevail, because the higher rates have the effect of discriminating againstthem based on their ethnicity.

B. The citizens will prevail unless State Ranch shows a compelling reason for discrimination.

C. State Ranch will prevail unless the citizens can show that they pay higher rates thansimilarly situated cities with a smaller Latin-American population or no Latin-Americanpopulation at all.

D. State Ranch will prevail because discriminatory economic regulations are not a suspectclassification.

Page 219: Law Decks Flash Cards - Constitutional Law - 2007-2008

Answer: CState Ranch will prevail unless the citizens can show that State Ranch charges Latin-American citizens higher rates than other citizens of Angry City. Just having adiscriminatory effect is not sufficient to trigger strict scrutiny. There must be a showingof intent by the government to discriminate. Only if the citizens can show that StateRanch is charging them higher rates than other similarly situated citizens, thus, showingan intent to discriminate, will the strict scrutiny test be utilized.

A is incorrect because a showing of discriminatory effects is necessary for the Angry CityLatin-American citizens to prevail. Strict scrutiny will only be applied if intent todiscriminate is established, which can only be shown by: 1) racial discrimination, 2)discriminatory application, or 3) discriminatory motive.

B is incorrect as a compelling interest is the standard for a strict scrutiny test and strictscrutiny can only be triggered if intent to discriminate is shown. Here, there is noshowing of intent to discriminate. The alleged discrimination effect is not sufficient.

D is incorrect as a false fact. Government actions that improperly discriminate against asuspect class, even those deemed "economic" in nature, may not violate equalprotection.

©2007-2008 Law Decks

Page 220: Law Decks Flash Cards - Constitutional Law - 2007-2008

Constitutional LawSuppose Congress passed the Mail Standardization Act (MSA) setting rules regardingthe solicitation of business using the postal service. The MSA sets forth a list ofprerequisites that must be met by a potential solicitor before they can mail their firstsolicitation. This set of rules would substantially impact the solicitation of charities by theTemple of Dog, who raises virtually all of its revenue via US mail solicitations. Assumingthey file suit seeking a declaratory judgement; which is the applicable burden that theymust meet?

A. The federal government must demonstrate that this law helps to achieve a legitimateinterest when applied to religious and secular solicitations.

B. The federal government must demonstrate that this statute is necessary to advancea compelling governmental interest.

C. The Church of Dog must show a specific congressional purpose that inhibits theorganization's religious objectives.

D. The Church of Dog must show that no reasonable legislator could believe that theset of MSA laws accomplishes a legitimate governmental objective.

Page 221: Law Decks Flash Cards - Constitutional Law - 2007-2008

Answer: CHere, the Church of Dog must show a specific intention of the law to limit their religiousactivities and objectives. As long as this was not the purpose of the legislation, the stateis free to regulate conduct that has a tangential effect on a person's religious beliefs.The Church of Dog cannot rely on the Free Exercise Clause as there is no informationindicating that the law was intended to interfere with religion.

A is incorrect as it places the burden on the wrong parties. The burden is on the churchto show that the law was motivated by illegitimate reasons.

B is incorrect for stating the wrong standard. The compelling state interest application isonly applicable to a setting in which a law is purposefully designed to inhibit religiousconduct simply because it is religious in nature.

D is also incorrect. We are dealing with a law regulating general (i.e.—secular) conduct.Therefore, the Free Exercise Clause is not applicable. Here, the church must show thatthe legislation intended to interfere with religion.

52007-2008 Law Decks

Page 222: Law Decks Flash Cards - Constitutional Law - 2007-2008

Constitutional LawLee's minor son is murdered in a gruesome stabbing. The killer is subsequently caught and sentencedto death by a jury trial. The state supreme court overturns the conviction ten years later after they findthat the police who initially arrested the attacker coerced his confession by using a blunt butter knife anda gerbil named Tim. Lee is furious, and speaks to the District Attorney who refuses to prosecute thecase. The DA reasons that it's a terrible case as there is no longer any admissible evidence. (Tim thegerbil was no longer available for comment.) Lee, while interviewed by a reporter, states that his newcitizens' group will try to change the state constitution allowing further admissibility of various facts. Heinvites anyone interested to attend the group's first rally on the steps of the state supreme court building.A huge turnout arrives and Lee, somewhat emotional, calls out: "All of these judges who allowed myson's killer to go free are murderers themselves! If I had a gun I would shoot each of them myself' Leethen goes on to describe the aims of his new organization. A state statute, on the books since 1900,makes "the public declaration of blasphemy or sacrilege" a criminal offense. Lee becomes the fifthperson ever brought up on these charges. Which constitutional defenses would be the least beneficialto Lee?

A. Application of the statute is a freedom of speech violation of the Fourteenth Amendment.

B. Application of the statute is a denial of equal protection of the law in violation of the FourteenthAmendment.

C. Application of the statute is a violation of the Establishment Clause.

D. Application of the statute violated the Due Process Clause as applied to First Amendment-protectedfree speech.

Page 223: Law Decks Flash Cards - Constitutional Law - 2007-2008

Answer: BEqual protection is only applicable when a law treats a person (or class) differently thanothers. Here, the statute makes no person or class distinctions, and although infrequentlyapplied, there is no indication that the application to Lee was discriminatory.

A is incorrect because content-based prohibitions on speech are presumptivelyunconstitutional, requiring the government to show that the statute was necessary toserving a compelling state interest and narrowly drawn to achieve that end. Therefore, afree speech based defense would be a strong argument.

C is incorrect as a statute challenged under the Establishment Clause is valid when: 1) ithas a secular purpose, 2) it has a primary effect that neither advances nor inhibits religion,and 3) it does not produce excessive government entanglement with religion. Arguably, astatute prohibiting "blasphemy" and "sacrilege" is a clear violation of the above.

D is incorrect. The Due Process Clause provides that a criminal law cannot fail to givereasonable notice as to what is prohibited. In light of the First Amendment free speechissues at stake, a due process application will be strictly evaluated in order to protect anyfreedom of speech rights. Here, a defense based upon the argument that the statute isunconstitutionally vague as it does not define the prohibited speech explicitly (i.e. Whatis "blasphemous" language?) is justified.

©2007-2008 Law Decks

Page 224: Law Decks Flash Cards - Constitutional Law - 2007-2008

Constitutional LawLee's minor son is murdered in a gruesome stabbing. The killer is subsequently caught andsentenced to death by a jury trial. The state supreme court overturns the conviction ten years laterafter they find that the police who initially arrested the attacker coerced his confession by using ablunt butter knife and a gerbil named Tim. Lee is furious, and speaks to the District Attorney whorefuses to prosecute the case. The DA reasons that it's a terrible case as there is no longer anyadmissible evidence. (Tim the gerbil was no longer available for comment.) Lee, while interviewedby a reporter, states that his new citizens' group will try to change the state constitution allowingfurther admissibility of various facts. He invites anyone interested to attend the group's first rally onthe steps of the state supreme court building. A huge turnout arrives and Lee, somewhat emotional,calls out: "All of these judges who allowed my son's killer to go free are murderers themselves! If Ihad a gun I would shoot each of them myself' Lee then goes on to describe the aims of his neworganization, hands out leaflets and then heads home to watch a ballgame. There is a statute thatstates "making threats directed to judges is unlawful and criminally punishable." Which of thefollowing is true in regards to this statute?

A. Lee can be punished under the statute only if the state court judges heard the threats he made.

B. Lee could constitutionally be punished under the statute.

C. Lee could not be constitutionally punished under these circumstances, yet the statute isconstitutional on its face.

D. The statute is unconstitutional on its face.

Page 225: Law Decks Flash Cards - Constitutional Law - 2007-2008

Answer: C

Content-based restrictions on speech are permissible only when the speech creates aclear and present danger of imminent lawless action. The state can make advocating theuse of force or violating the law unlawful if there is a showing that such advocacy is: 1)directed to producing or inciting imminent lawless action and 2) is likely to produce orincite such action. The statute itself is constitutional (making Answer D incorrect).However, there is little likelihood that Lee's speech meets the requirements of 1 and 2,making him free from judgement.

A is wrong. The judges actually hearing the words of Lee is a moot point. The statutemakes spoken words actionable; it is not dependent upon the targets of those wordshearing them.

B is incorrect. Lee was not actually threatening the judges with harm or inciting thecrowd but rather venting his angry grief and went home after handing out his leaflets.There was no indication that Lee's words were inciting imminent lawless action or werelikely to produce such action.

©2007-2008 Law Decks

Page 226: Law Decks Flash Cards - Constitutional Law - 2007-2008

Constitutional LawLee's minor son is murdered in a gruesome stabbing. The killer is subsequently caught and sentenced todeath by a jury trial. The state supreme court overturns the conviction ten years later after they find thatthe police who initially arrested the attacker coerced his confession by using a blunt butter knife and agerbil named Tim. Lee is furious, and speaks to the District Attorney who refuses to prosecute the case.The DA reasons that it's a terrible case as there is no longer any admissible evidence. (Tim the gerbil wasno longer available for comment.) Lee, while interviewed by a reporter, states that his new citizens' groupwill try to change the state constitution allowing further admissibility of various facts. He invites anyoneinterested to attend the group's first rally on the steps of the state supreme court building. A huge turnoutarrives and Lee, somewhat emotional, calls out: "All of these judges who allowed my son's killer to go freeare murderers themselves! If I had a gun I would shoot each of them myself!" Lee then goes on todescribe the aims of his new organization, hands out leaflets and then heads home to watch a ballgame.There is a statute making "all speechmaking on the steps of the state supreme court during court businesshours...illegal." Assuming that Lee is charged with violation of this statute, which is the applicable burdenof proof?

A. The state must show a compelling need for the statute and that there are no less restrictive alternativesexisting to meet this need.

B. The state must show a narrow tailoring of the statute to serve a significant government interest and thatalternative channels of communication remain available.

C. Lee must show that there is a less restrictive alternative that the state could have used and that thereis no compelling need for the statute.

D. Lee must show that there is no reasonable basis for the statute.

Page 227: Law Decks Flash Cards - Constitutional Law - 2007-2008

Answer: DA showing that the statute was not reasonably related to a legitimate governmentpurpose is necessary in the given set of facts. Most public property is considered anonpublic forum, and the steps of the court building are also considered a nonpublicforum. (In comparison, public forums generally are limited to streets, sidewalks andareas such as parks.) The government can regulate speech in these forums as long asthe regulation is: 1) viewpoint neutral, and 2) reasonably related to a legitimategovernment purpose. Here, the prohibiting of gatherings on the courthouse steps duringbusiness hours is viewed as reasonable, viewpoint-neutral and aids in maintaining orderon the government property, thereby preserving the property for its intended use. In lightof this, the burden of proving that there is no reasonable basis for the statute falls to Lee.

Answers A and C are incorrect for applying the wrong standard. Strict scrutiny is onlyapplied in situations where there is content-based regulation of speech, unlike therestrictions set forth here which were not content-based, but rather time and locationbased.

B is incorrect for stating the wrong test: namely the test for public forums as opposed tothe nonpublic forum at issue here.

©2007-2008 Law Decks

Page 228: Law Decks Flash Cards - Constitutional Law - 2007-2008

Constitutional LawFederal Judge Turpis is accused of misconduct, impeached following a trial by theHouse of Representatives, and subsequently removed from office following a Senateinvestigation. Appalled by Turpis's actions, the President asks the Attorney General tocharge him criminally. A grand jury investigation results in the institution of criminalproceedings against Judge Turpis. At the opening of his trial, Judge Turpis moves todismiss. Most likely the trial judge would:

A. Dismiss, as the President had told the Attorney General to prosecute.

B. Dismiss, the criminal proceeding is a Fifth Amendment double jeopardy violation.

C. Deny the dismissal, because the federal grand jury issued the indictment.

D. Deny the dismissal, as Judge Turpis has not been criminally tried.

Page 229: Law Decks Flash Cards - Constitutional Law - 2007-2008

Answer: D

The dismissal will likely be denied as the Fifth Amendment right against double jeopardy(as applied to the same offense) covers subsequent criminal proceedings, but does notcover civil actions or impeachment proceedings. Under Article 1, Section 3 of theConstitution, a conviction by impeachment does not prevent the defendant from furthertrial or judgement. In light of this, Answer B is clearly wrong.

A is incorrect. There are no facts in the question indicating that the Attorney General wasacting outside the rights awarded to his office, regardless of the fact that it was thePresident who requested the trial.

C is incorrect. The fact that the indictment was issued by the grand jury is irrelevant. Ifdouble jeopardy was applicable, or if the Attorney General had been acting outside hisprosecutorial discretion, the mere fact that the grand jury issued an indictment is mootand does not result in the dismissal of the indictment.

©2007-2008 Law Decks

Page 230: Law Decks Flash Cards - Constitutional Law - 2007-2008

Constitutional LawUlrika is a foreign exchange student who entered the US on a student visa three years ago. Hervisa expires resulting in the Immigration and Naturalization Service (INS) informing her that shemay be deported. Federal law states that an alien subject to deportation has a constitutionalright to a hearing in front of an Attorney General-appointed administrative officer who has theright to make a deportation ruling. Ulrika has her hearing and the administrative officer statesthat Ulrika may remain in the US as a legal alien. There is a congressional act, however, thatallows the House of Representative to deport those they deem "incorrigible aliens" of which theyfind Ulrika. Ulrika, about to be deported, petitions the federal court to declare this congressionalact invalid. The court should find the resolution:

A. Valid, Congress is granted plenary power in regards to aliens and naturalization.

B. Valid, because aliens are not "citizens" under the Fourteenth Amendment.

C. Invalid, as the INS law removed congressional power over aliens, and the resolution passedby the House of Representatives is, therefore, in violation of the Doctrine of Separation ofPowers.

D. Invalid, Ulrika was denied due process since she was not given a hearing before the Houseof Representatives.

Page 231: Law Decks Flash Cards - Constitutional Law - 2007-2008

Answer: CUnder the Separation of Powers Doctrine, Congress cannot attempt to control power thatit has already delegated to other agencies absent a passage by both houses of Congress(bicameralism). Here, the federal law gave the administrative officer the power todetermine deportation of an individual, and Congress essentially relinquished any controlit may have had in matters such as this. Therefore, the House resolution is anunconstitutional legislative veto, a violation of the Separation of Powers Doctrine.

A is wrong. Congress has given up control over aliens and deportation in the present setof facts, in light of the law providing the administrative officer review power overdeportation matters.Note: In most cases Congress does traditionally have plenary power over aliens asrelated to immigration and naturalization.

B is incorrect as it is irrelevant. The fact that aliens are not citizens has no bearing onwhether the House resolution is in fact a constitutional violation.

D is incorrect. A resident alien simply needs a hearing before deportation. It is notessential that this hearing be conducted by the body that is actually deporting them.Here, Ulrika has a valid hearing under the administrative office which suffices. In light ofthis, a separation of powers argument is the stronger answer.

S2007-2008 Law Decks

Page 232: Law Decks Flash Cards - Constitutional Law - 2007-2008

Constitutional LawRiver defeats Hoffer for State Truck Drivers Union president. On the day after River'sswearing in, Hoffer learns that River was a member of the communist party 20 years agowhile attending college. This violates a state statute prohibiting a member of thecommunist party from holding elected office. Hoffer instantly brings this to the union'sattention and River is promptly removed from office. River sues challenging theconstitutionality of the statute. The court will most likely hold the statute:

A. Constitutional, because the statute bears directly on the internal security of theUnited States.

B. Constitutional, upon a showing of a rational basis for the adoption of the statute.

C. Unconstitutional, because the federal government may never discriminate amongpersons on the basis of political affiliation.

D. Unconstitutional, as a bill of attainder.

Page 233: Law Decks Flash Cards - Constitutional Law - 2007-2008

Answer: DA bill of attainder is a legislative act that inflicts punishment on a person, designed eitherby name or by past conduct, without a judicial trial. In this present set of facts, theprohibition against bills of attainder requires that River is given a chance in front of ajudiciary. Therefore, the bill here is deemed a bill of attainder and is unconstitutional.

A is incorrect. Internal security concerns are inapplicable to the present case in light ofthe bill of attainder information set forth in the correct answer. Under the present facts,River is entitled to a judicial trial prior to his removal.

B is wrong in light of the bill of attainder argument of the correct answer. A rational basistest is moot if the statute is a bill of attainder, as a bill of attainder cannot constitutionallybe upheld.

C is incorrect as being too broad. A showing of a compelling governmental interest, mayallow for the suppression of a person's right to join with others for political activity.

S2007-2008 Law Decks

Page 234: Law Decks Flash Cards - Constitutional Law - 2007-2008

Constitutional LawSuppose Minnesota faces diminished beaver populations due to over-hunting and rapidly expandingsuburbs, which is especially problematic as beaver pelts are a great revenue generator. Both the federaland state government decide to take action, resulting in the passage of the following legislation. The statestatute requires: 1) A minimum beaver length of 12 inches is strictly enforced, and all beavers under 12inches must be released unharmed. 2) A tax of $5 per beaver is applied to all state-trapped beaver, theproceeds of which are used to build beaver dam communities. 3) All trappers must report the number offoxes trapped to their local fish and game officer, who is charged with collecting the tax. The federalgovernment legislation provides $3 million in funding to investigate how best to reestablish the beaverpopulation and ascertain the number of beavers living in harmony with humans. Additionally, a $50 tax isapplied to any trapped beaver that is later sold in interstate commerce.

A consumer group sues in federal court to stop the expenditure of the $3 million. They deem it an improperexercise of governmental power and feel the project itself is unnecessary and extravagant. Which of thefollowing defenses raised by the federal government are valid?

I. The legislation is a valid exercise of the federal taxing and spending powers.

H. The Doctrine of Separation of Powers prevents the federal court from enjoining these expendituresvoted by Congress as they lack jurisdiction.

III. A consumer group lacks standing to sue.

A. I and II only B. I and Ill only C. I only D. III only

Page 235: Law Decks Flash Cards - Constitutional Law - 2007-2008

Answer: BConsumer groups have standing and the legislation is, in fact, a valid expenditure ofCongress as they have the power to lay and collect taxes, and to spend in order toprovide for the general welfare. A tax is generally permissible as long as it bears somereasonable relationship to revenue production, or if Congress has the power to regulatethe taxed activity. Additionally, Congress is empowered to spend for any public purpose.In this case, the excise tax and the appropriation are valid and within the scope ofCongressional power. Therefore, Statement I is a valid defense. Furthermore,Statement III is valid as the consumer group has standing to sue on behalf of itsmembers only if the individual members would have a right to sue on their own behalf.Generally, a taxpayer has no standing to challenge the way their tax dollars are spent,the only exception being in cases where Congress's spending results in anEstablishment Clause violation. Here, there is no Establishment Clause violation atissue. Therefore, there is no applicable standing.

Statement II is incorrect. It is well established that the principle of judicial review allowsthe federal courts to review, and possibly enjoin, expenditures voted by Congress.

©2007-2008 Law Decks

Page 236: Law Decks Flash Cards - Constitutional Law - 2007-2008

Constitutional LawSuppose Minnesota faces diminished beaver populations due to over-hunting and rapidly expandingsuburbs, which is especially problematic as beaver pelts are a great revenue generator. Both the federaland state government decide to take action, resulting in the passage of the following legislation. The statestatute requires: 1) A minimum beaver length of 12 inches is strictly enforced, and all beavers under 12inches must be released unharmed. 2) A tax of $5 per beaver is applied to all state-trapped beaver, theproceeds of which are used to build beaver dam communities. 3) All trappers must report the number offoxes trapped to their local fish and game officer, who is charged with collecting the tax. The federalgovernment legislation provides $3 million in funding to investigate how best to reestablish the beaverpopulation and ascertain the number of beavers living in harmony with humans. Additionally, a $50 tax isapplied to any trapped beaver that is later sold in interstate commerce.

Karin is a hunter and resident of a neighboring state with no trapping restrictions. While tracking a beaver,Karin accidentally crosses the state line entering Minnesota from the west. She is promptly arrested by afish and wildlife officer patrolling the border for possessing a beaver less than 12 inches long. Assumethat Karin challenges the constitutionality of the Minnesota statute. Which of the following is the mostlikely result?

A. The statute is a violation of the Privileges and Immunities Clause and is invalid.

B. The statute is invalid because it has been preempted by the federal legislation.

C. The statute is permissible as Congress is free to allow the state to adopt legislation that wouldotherwise violate the Commerce Clause.

D. The statute is permissible as it is not unduly burdensome on interstate commerce.

Page 237: Law Decks Flash Cards - Constitutional Law - 2007-2008

Answer: DIn a setting in which Congress has not preempted the area of regulation, a state is free toregulate local aspects of interstate commerce as long as their regulation: 1) does notdiscriminate against out-of-state competition to benefit local economic interests, and 2) is notunduly burdensome (a burden vs. benefit analysis). Here, the federal legislation is not in conflictwith the state law, nor does it preempt state regulation of beaver trapping. Additionally, the statelaw does not discriminate against interstate commerce, or out-of-state competition, since itapplies to all hunting of beavers within the state. Finally, there is no undue burden as the stateinterest, trying to preserve beavers (a natural resource), outweighs the incidental burden oninterstate commerce.

A is incorrect. The Privileges and Immunities Clause prohibits state discrimination against non-residents engaging in commercial activities. Here, the law is not discriminatory since it isuniformly applied to all hunters in Minnesota regardless of residency.

B is incorrect. The federal legislation has not preempted state law in the area regarding thetrapping of beaver.

C is incorrect. The statute does not discriminate, nor is it a violation of the Commerce Clauseeven in the absence of congressional regulation on the topic. Here, the Congressional excisetax reinforces state laws. It does not grant the states Congress's power over interstatecommerce, regardless of the fact that Congress may allow states to pass regulations that wouldotherwise be viewed as violations of the Commerce Clause.

©2007-2008 Law Decks

Page 238: Law Decks Flash Cards - Constitutional Law - 2007-2008

Constitutional LawSuppose Minnesota faces diminished beaver populations due to over-hunting and rapidly expandingsuburbs, which is especially problematic as beaver pelts are a great revenue generator. Both the federaland state government decide to take action, resulting in the passage of the following legislation. The statestatute requires: 1) A minimum beaver length of 12 inches is strictly enforced, and all beavers under 12inches must be released unharmed. 2) A tax of $5 per beaver is applied to all state-trapped beaver, theproceeds of which are used to build beaver dam communities. 3) All trappers must report the number offoxes trapped to their local fish and game officer, who is charged with collecting the tax. The federalgovernment legislation provides $3 million in funding to investigate how best to reestablish the beaverpopulation and ascertain the number of beavers living in harmony with humans. Additionally, a $50 tax isapplied to any trapped beaver that is later sold in interstate commerce.

David, a resident of South Dakota, hunts in Minnesota and sells his furs on the internet in various states.As the $50 per beaver tax cuts into his profits, David sues challenging the constitutionality of the excisetax. Which of the following is the most likely result?

A. David will win because the tax is invalid under the Commerce Clause.

B. David will win, because the tax is invalid under the Import-Export Clause.

C. David will lose, because the tax is valid under the Commerce Clause.

D. David will lose, because the tax is valid under the Import-Export Clause.

Page 239: Law Decks Flash Cards - Constitutional Law - 2007-2008

Answer: CDavid is likely to loose as the tax is valid under the Commerce Clause, making AnswerA immediately incorrect. When dealing with taxation, a nondiscriminatory state tax thathas interstate commerce effects is viewed as an undue burden on commerce unless allthree parts of this test are met: 1) there must be a substantial nexus between the activitytaxed and the taxing state, 2) the tax must be fairly apportioned according to a rationalformula, and 3) the tax must be fairly related to the benefits or services provided by thestate. Here, the tax satisfies all three parts: the trapping of beaver within the state clearlyhas a nexus with the state, the tax is on a per beaver basis, and the tax revenue is usedfor a legitimate purpose, namely: the conservation of beavers.

Answers B and D are incorrect since the Import-Export Clause prohibits states fromimposing taxes on goods imported from or exported to foreign nations. Here, the factsgive no indication that the furs are imported or exported outside of the US.

©2007-2008 Law Decks